When an aircraft is decelerated some attitude indicators will precess and incorrectly indicate a group of answer choices?

[52]During normal operation of a vacuum-driven attitude indicator, what attitude indicationshould you see when rolling out from a 180° skidding turn to straight-and-levelcoordinated flight?A straight-and-level coordinated flight indication.A.A nose-high indication relative to level flight.B.The miniature aircraft shows a turn in the direction opposite the skid.C.Gleim FAA Test Prep:Instrument Pilot(75 questions)

Gleim's FAA Test Prep Gleim's FAA Test Prep
Instrument Pilot

[1]

On the taxi check, the magnetic compass should
  1. swing freely and indicate known headings.
  2. swing opposite to the direction of turn when turning from north.
  3. exhibit the same number of degrees of dip as the latitude.
  • Answer (A) is correct. When taxiing, the magnetic compass should be checked to make sure that the compass card is swinging freely and indicating known headings.
  • Answer (B) is incorrect because the compass exhibits turning errors only when the airplane is airborne and in a bank.
  • Answer (C) is incorrect because compass turning errors occur in flight as a result of magnetic dip.

[2]

What should be the indication on the magnetic compass as you roll into a standard-rate turn to the left from an east heading in the Northern Hemisphere?
  1. The compass will initially indicate a turn to the right.
  2. The compass will indicate the approximate correct magnetic heading if the roll into the turn is smooth.
  3. The compass will remain on east for a short time, then gradually catch up to the magnetic heading of the aircraft.
  • Answer (A) is incorrect because the compass will initially indicate a turn in the opposite direction of a bank only when the airplane is turning from a north heading.
  • Answer (B) is correct. If you roll into a smooth, coordinated, standard-rate turn to the left or right from an east or west heading, the compass will indicate the approximate correct magnetic heading. There are no turning errors on turns from east or west headings.
  • Answer (C) is incorrect because when the airplane is turning from an east heading, the compass will immediately indicate a turn, if the roll into the turn is smooth.

[3]

What should be the indication on the magnetic compass as you roll into a standard rate turn to the right from an easterly heading in the Northern Hemisphere?
  1. The compass will initially indicate a turn to the left.
  2. The compass will remain on east for a short time, then gradually catch up to the magnetic heading of the aircraft.
  3. The compass will indicate the approximate correct magnetic heading if the roll into the turn is smooth.
  • Answer (A) is incorrect because the compass will initially indicate a turn in the opposite direction of a bank only when turning from a north heading.
  • Answer (B) is incorrect because, when turning from an east heading, the compass will immediately indicate a turn if the roll into the turn is smooth.
  • Answer (C) is correct. If you roll into a smooth, coordinated, standard-rate turn to the left or right from an east or west heading, the compass will normally indicate the approximate correct magnetic heading. Acceleration, not turning, errors are found in east or west headings in the Northern Hemisphere.

[4]

What should be the indication on the magnetic compass as you roll into a standard rate turn to the right from a south heading in the Northern Hemisphere?
  1. The compass will indicate a turn to the right, but at a faster rate than is actually occurring.
  2. The compass will initially indicate a turn to the left.
  3. The compass will remain on south for a short time, then gradually catch up to the magnetic heading of the aircraft.
  • Answer (A) is correct. In turns from a southerly heading in the Northern Hemisphere, the compass leads the turn. That is, it indicates a turn in the proper direction but at a faster rate than is actually occurring.
  • Answer (B) is incorrect because the compass will initially indicate a turn in the opposite direction if the turn is made from a north, not south, heading.
  • Answer (C) is incorrect because it describes a shallower-than-standard turn from a north, not south, heading.

[5]

On what headings will the magnetic compass read most accurately during a level 360° turn, with a bank of approximately 15°?
  1. 180° and 0°.
  2. 90° and 270°.
  3. 135° through 225°.
  • Answer (A) is incorrect because compass turning errors are greatest at north and south headings.
  • Answer (B) is correct. In a turn through north or south, the compass indication will usually be incorrect. But in a turn through east or west, the compass indication is usually accurate. Therefore, the compass should be indicating properly when passing through 90° and 270°. This holds true for only medium to shallow bank turns, e.g., 15-30°.
  • Answer (C) is incorrect because the compass leads the heading in a turn through south.

[6]

What causes the northerly turning error in a magnetic compass?
  1. Centrifugal force acting on the compass card.
  2. Coriolis force at the mid-latitudes.
  3. The magnetic dip characteristic.
  • Answer (A) is incorrect because centrifugal force is not related to compass turning errors.
  • Answer (B) is incorrect because the Coriolis force of the spinning Earth affects winds rather than compass indications.
  • Answer (C) is correct. Magnetic dip causes a compass needle to point both down toward the earth and to the magnetic pole. This characteristic causes turning errors.

[7]

What should be the indication on the magnetic compass when you roll into a standard rate turn to the left from a south heading in the Northern Hemisphere?
  1. The compass will initially indicate a turn to the right.
  2. The compass will indicate a turn to the left, but at a faster rate than is actually occurring.
  3. The compass will remain on south for a short time, then gradually catch up to the magnetic heading of the aircraft.
  • Answer (A) is incorrect because the compass will initially indicate a turn in the opposite direction if the turn is made from a north, not south, heading.
  • Answer (B) is correct. In turns from a southerly heading in the Northern Hemisphere, the compass leads the turn. That is, it indicates a turn in the proper direction but at a rate faster than the actual rate.
  • Answer (C) is incorrect because it describes a shallower-than-standard turn from a north, not south, heading.

[8]

What should be the indication on the magnetic compass as you roll into a standard rate turn to the right from a westerly heading in the Northern Hemisphere?
  1. The compass will indicate the approximate correct magnetic heading if the roll into the turn is smooth.
  2. The compass will initially show a turn in the opposite direction, then turn to a northerly indication but lagging behind the actual heading of the aircraft.
  3. The compass will remain on a westerly heading for a short time, then gradually catch up to the actual heading of the aircraft.
  • Answer (A) is correct. If you roll into a smooth, coordinated, standard-rate turn to the left or right from an east or west heading, the compass will indicate the approximate correct magnetic heading. There are no turning errors on turns from east or west headings.
  • Answer (B) is incorrect because the compass will initially indicate a turn in the opposite direction of a bank only when turning from a north heading.
  • Answer (C) is incorrect because when turning from a westerly heading, the compass will immediately indicate a turn if the roll into the turn is smooth.

[9]

What should be the indication on the magnetic compass as you roll into a standard rate turn to the right from a northerly heading in the Northern Hemisphere?
  1. The compass will remain on north for a short time, then gradually catch up to the magnetic heading of the aircraft.
  2. The compass will initially indicate a turn to the left.
  3. The compass will indicate a turn to the right, but at a faster rate than is actually occurring.
  • Answer (A) is incorrect because it describes a shallower-than-standard turn from north.
  • Answer (B) is correct. In turns from a northerly heading in the Northern Hemisphere, the compass will initially indicate a turn to the opposite direction. Then the compass heading will lag behind the actual heading until the airplane approaches an east or west heading.
  • Answer (C) is incorrect because it describes a turn to the right from a southerly heading.

[10]

What should be the indication on the magnetic compass as you roll into a standard rate turn to the left from a west heading in the Northern Hemisphere?
  1. The compass will initially indicate a turn to the right.
  2. The compass will indicate the approximate correct magnetic heading if the roll into the turn is smooth.
  3. The compass will remain on west for a short time, then gradually catch up to the magnetic heading of the aircraft.
  • Answer (A) is incorrect because the compass will initially indicate a turn in the opposite direction of a bank only in a turn from a north heading.
  • Answer (B) is correct. If you roll into a smooth, coordinated, standard-rate turn to the left or right from an east or west heading, the compass will indicate the approximate correct magnetic heading. Turning errors appear on turns from north or south headings, not east or west.
  • Answer (C) is incorrect because in a turn from an east heading, the compass will immediately indicate a turn if the roll into the turn is smooth.

[11]

What should be the indication on the magnetic compass as you roll into a standard rate turn to the left from a north heading in the Northern Hemisphere?
  1. The compass will remain on north for a short time, then gradually catch up to the magnetic heading of the aircraft.
  2. The compass will initially indicate a turn to the right.
  3. The compass will indicate a turn to the left, but at a faster rate than is actually occurring.
  • Answer (A) is incorrect because it describes a shallower-than-standard turn from north.
  • Answer (B) is correct. In turns from a northerly heading in the Northern Hemisphere, the compass will initially indicate a turn to the opposite direction, and the compass heading will lag behind actual until the airplane approaches an east or west heading.
  • Answer (C) is incorrect because it describes a turn to the left from a southerly heading.

[12]

During flight, if the pitot tube becomes clogged with ice, which of the following instruments would be affected?
  1. The airspeed indicator only.
  2. The airspeed indicator and the altimeter.
  3. The airspeed indicator, altimeter, and Vertical Speed Indicator.
  • Answer (A) is correct. The pitot-static system is a source of pressure for the altimeter, vertical speed indicator, and airspeed indicator. The pitot tube is connected directly to the airspeed indicator, and the static vents are connected directly to all three. The pressure of air coming into the pitot tube (impact air pressure) is compared with the air pressure at the static system vents to determine airspeed.
  • Answer (B) is incorrect because the pitot tube is connected only to the airspeed indicator, not the altimeter.
  • Answer (C) is incorrect because the pitot tube is connected only to the airspeed indicator, not the altimeter or vertical speed indicator.

[13]

If both the ram air input and the drain hole of the pitot system are blocked, what reaction should you observe on the airspeed indicator when power is applied and a climb is initiated out of severe icing conditions?
  1. No change until an actual climb rate is established, then indicated airspeed will increase.
  2. The airspeed would drop to, and remain at, zero.
  3. The indicated airspeed would show a continuous deceleration while climbing.
  • Answer (A) is correct. When the airspeed indicator pitot tube and drain hole are blocked, the airspeed indicator will react as an altimeter. There will be a constant pressure within the pitot tube, and as the pressure from the static source decreases during a climb, the indicated airspeed on the altimeter will increase.
  • Answer (B) is incorrect because this reaction would occur if the ram air input, but not the drain hole, were blocked.
  • Answer (C) is incorrect because, as there is an increase in altitude, the airspeed indicator will show an increase in airspeed as the air pressure within the pitot tube is relatively greater than that of the static air.

[14]

What indication should a pilot observe if an airspeed indicator ram air input and drain hole are blocked?
  1. The airspeed indicator will show a decrease with an increase in altitude.
  2. No airspeed indicator change will occur during climbs or descents.
  3. The airspeed indicator will react as an altimeter.
  • Answer (A) is incorrect because indicated airspeed will increase, not decrease, with increases in altitude.
  • Answer (B) is incorrect because differential pressure between the pitot tube and static air source changes, and so does indicated airspeed.
  • Answer (C) is correct. When the airspeed indicator pitot tube and drain hole are blocked, the airspeed indicator will react as an altimeter. There will be a constant pressure within the pitot tube, and as the pressure from the static source decreases during a climb, the indicated airspeed on the altimeter will increase.

[15]

If both the ram air input and drain hole of the pitot system are blocked, what airspeed indication can be expected?
  1. Constant indicated airspeed during a descent.
  2. No variation of indicated airspeed in level flight even if large power changes are made.
  3. Decrease of indicated airspeed during a climb.
  • Answer (A) is incorrect because indicated airspeed would change with changes in altitude.
  • Answer (B) is correct. If both the pitot tube input and the drain hole on the pitot system are blocked, the airspeed indication will be constant at any given altitude.
  • Answer (C) is incorrect because, during a climb, the airspeed indicator will indicate an increase, not decrease, due to the stronger differential pressure in the blocked pitot tube relative to the static vents.

[16]

If the outside air temperature increases during a flight at constant power and at a constant indicated altitude, the true airspeed will
  1. increase and true altitude will decrease.
  2. decrease and true altitude will increase.
  3. increase and true altitude will increase.
  • Answer (A) is incorrect because true altitude increases, not decreases.
  • Answer (B) is incorrect because TAS increases, not decreases.
  • Answer (C) is correct. As temperature increases, pressure levels rise farther above sea level and true altitude increases given a constant indicated altitude. As altitude increases with constant power, true airspeed increases.

[17]

If, while in level flight, it becomes necessary to use an alternate source of static pressure vented inside the airplane, which of the following should the pilot expect?
  1. The vertical speed to momentarily show a descent.
  2. The altimeter to read higher than normal.
  3. The vertical speed to show a climb.
  • Answer (A) is incorrect because the VSI indication will be a momentary climb, not a descent.
  • Answer (B) is correct. Most aircraft equipped with a pitot-static system are provided with an alternate source of static pressure for emergency use. This source is usually vented inside the cabin. The pressure within an unpressurized cockpit is slightly lower than the pressure outside the airplane because of the Venturi effect of the air moving past the outside of the cockpit. When the alternate static source is used, the altimeter and airspeed indicator will read higher than actual, and the vertical speed indicator will momentarily show a climb.
  • Answer (C) is incorrect because the VSI will show only a momentary, not a steady-state, climb.

[18]

If while in level flight, it becomes necessary to use an alternate source of static pressure vented inside the airplane, which of the following variations in instrument indications should the pilot expect?
  1. The altimeter will read higher than normal, airspeed greater than normal, and the VSI will momentarily show a climb.
  2. The altimeter will read lower than normal, airspeed lower than normal, and the VSI will momentarily show a descent.
  3. The altimeter will read lower than normal, airspeed greater than normal, and the VSI will momentarily show a climb and then a descent.
  • Answer (A) is correct. Most aircraft equipped with a pitot-static system are provided with an alternate source of static pressure for emergency use. This source is usually vented inside the cabin. The pressure within an unpressurized cockpit is slightly lower than the pressure outside the airplane because of the Venturi effect of the air moving past the outside of the cockpit. When the alternate static source is used, the altimeter will read higher than actual, and the vertical speed indicator will momentarily show a climb.
  • Answer (B) is incorrect because the altimeter and airspeed indicators will read higher, not lower, than normal, and the vertical speed indicator will momentarily show a climb, not a descent.
  • Answer (C) is incorrect because the altimeter will read higher, not lower, than normal, and the vertical speed indicator will show level flight, not a descent, after the momentary climb.

[19]

If, while in level flight, it becomes necessary to use an alternate source of static pressure vented inside the airplane, which of the following should the pilot expect?
  1. The gyroscopic instruments to become inoperative.
  2. The vertical speed to momentarily show a climb.
  3. The altimeter and airspeed indicator to become inoperative.
  • Answer (A) is incorrect because the gyroscopic instruments are not affected by changes in the pitot-static system.
  • Answer (B) is correct. Most aircraft equipped with a pitot-static system are provided with an alternate source of static pressure for emergency use. This source is usually vented inside the cabin. The pressure within an unpressurized cockpit is slightly lower than the pressure outside the airplane because of the Venturi effect of the air moving past the outside of the cockpit. When the alternate static source is used, the altimeter and airspeed indicator will read higher than actual, and the vertical speed indicator will momentarily show a climb.
  • Answer (C) is incorrect because the altimeter and airspeed indicator will read higher than actual, not become inoperative.

[20]

What would be the indication on the VSI during entry into a 500 FPM actual descent from level flight if the static ports were iced over?
  1. The VSI pointer would remain at zero regardless of the actual rate of descent.
  2. The indication would be in reverse of the actual rate of descent (500 FPM climb).
  3. The initial indication would be a climb, then descent at a rate in excess of 500 FPM.
  • Answer (A) is correct. The vertical speed indicator operates from the static pressure source and indicates change in pressure. Thus, if the static pressure ports became iced over, there would be no change in the static pressure, and there would be no indication of descent or climb.
  • Answer (B) is incorrect because, with no change in pressure, the indication would be level flight, not a climb.
  • Answer (C) is incorrect because, with no change in pressure, the indication would be level flight, not a climb or descent.

[21]

You check the flight instruments while taxiing and find the vertical speed indicator (VSI) indicates a descent of 100 feet per minute. In this case, you
  1. may take off and use 100 feet descent as the zero indication.
  2. must return to the parking area and have the instrument corrected by an authorized instrument repairman.
  3. may not take off until the instrument is corrected by either the pilot or a mechanic.
  • Answer (A) is correct. The needle of the vertical speed indicator (VSI) should indicate zero when the aircraft is on the ground or is maintaining a constant pressure level in flight. If it does not indicate zero, you must allow for the error when interpreting the indications in flight. Since the VSI is not a required instrument, you may fly when it is out of adjustment.
  • Answer (B) is incorrect because the VSI can be used by making allowances in flight.
  • Answer (C) is incorrect because the VSI can be used by making allowances in flight.

[22]

What information does a Mach meter present?
  1. The ratio of aircraft true airspeed to the speed of sound.
  2. The ratio of aircraft equivalent airspeed, corrected for installation error, to the speed of sound.
  3. The ratio of aircraft indicated airspeed to the speed of sound.
  • Answer (A) is correct. The Mach meter indicates the ratio of aircraft true airspeed to the speed of sound at flight altitude. It uses the pressure differential between the impact and static air sources and corrects automatically for temperature and altitude.
  • Answer (B) is incorrect because equivalent airspeed is the calibrated airspeed of an aircraft corrected for compressibility flow at a particular altitude. The Mach meter uses true airspeed.
  • Answer (C) is incorrect because the Mach meter uses true airspeed, not indicated airspeed.

[23]

How should you preflight check the altimeter prior to an IFR flight?
  1. Set the altimeter to the current altimeter setting. The indication should be within 75 feet of the actual elevation for acceptable accuracy.
  2. Set the altimeter to 29.92" Hg. With current temperature and the altimeter indication, determine the true altitude to compare with the field elevation.
  3. Set the altimeter first with 29.92" Hg and then the current altimeter setting. The change in altitude should correspond to the change in setting.
  • Answer (A) is correct. Set the altimeter to the local altimeter setting before taking off, and verify that the indication is within 75 ft. of the actual elevation. If the difference is greater than 75 ft., you should consult an instrument repair shop.
  • Answer (B) is incorrect because setting the altimeter to 29.92 and adjusting for temperature gives density altitude, not true altitude.
  • Answer (C) is incorrect because only indicated altitude, not change in altitude, is checked.

[24]

How should you preflight check the altimeter prior to an IFR flight?
  1. Set the altimeter to the current altimeter setting. The indication should be within 75 feet of the actual elevation for acceptable accuracy.
  2. Set the altimeter to the current temperature. With current temperature and the altimeter indication, determine the calibrated altitude to compare with the field elevation.
  3. Set the altimeter first with 29.92" Hg and then the current altimeter setting. The change in altitude should correspond to the change in setting.
  • Answer (A) is correct. Set the altimeter to the local altimeter setting before taking off, and verify that the indication is within 75 ft. of the actual elevation. If the difference is greater than 75 ft., you should consult an instrument repair shop.
  • Answer (B) is incorrect because an altimeter can be adjusted for nonstandard pressure, not temperature.
  • Answer (C) is incorrect because only indicated altitude, not change in altitude, is checked.

[25]

Pressure altitude is the altitude read on your altimeter when the instrument is adjusted to indicate height above
  1. sea level.
  2. ground level.
  3. the standard datum plane.
  • Answer (A) is incorrect because indicated altitude, not pressure altitude, is the altitude read on your altimeter (when set to the correct altimeter setting) to indicate the approximate height above mean sea level.
  • Answer (B) is incorrect because the height above ground (absolute altitude) may be indicated directly on a radar altimeter, not by adjusting a pressure altimeter to 29.92 in.
  • Answer (C) is correct. Pressure altitude is the altitude read on your altimeter when the instrument is adjusted to indicate height above the standard datum plane, i.e., an altimeter setting of 29.92 in. The standard datum plane is a theoretical level where the weight of the atmosphere is 29.92 in. of Hg.

[26]

The local altimeter setting should be used by all pilots in a particular area, primarily to provide for
  1. the cancellation of altimeter error due to nonstandard temperatures aloft.
  2. better vertical separation of aircraft.
  3. more accurate terrain clearance in mountainous areas.
  • Answer (A) is incorrect because the altimeter setting does not compensate for nonstandard temperatures aloft. Rather, all altimeters in the area will reflect the same uncompensated amount.
  • Answer (B) is correct. Because the altimeter in each airplane is equally affected by temperature and pressure variation errors, use of the local altimeter setting in a given area provides for better vertical separation of aircraft.
  • Answer (C) is incorrect because temperatures aloft must also be considered to assure terrain clearance.

[27]

(Refer to figure 84.) Which altimeter depicts 8,000 feet?
When an aircraft is decelerated some attitude indicators will precess and incorrectly indicate a group of answer choices?

When an aircraft is decelerated some attitude indicators will precess and incorrectly indicate a group of answer choices?


Figure 84
  • Answer (A) is incorrect because, while the short hand is pointing to 8 and the long hand is pointing to 0 (i.e., 8,000), the long thin needle with the flared tip (10,000-ft. indicator) is on 0, not on the fourth tick mark from the 0 toward the 1. Altimeter 3 is either broken or indicating 108,000 ft.
  • Answer (B) is correct. When indicating 8,000 ft., an altimeter has the short hand (1,000-ft. intervals) on 8, the long hand (100-ft. intervals) on zero, and the thin hand with flared tip (10,000-ft. intervals) just below 1, as shown in altimeter 2.
  • Answer (C) is incorrect because, in altimeter 1, both the thin hand and the short hand are on 8, which would indicate something in the vicinity of 88,000 ft.

[28]

(Refer to figure 83.) Which altimeter depicts 12,000 feet?
When an aircraft is decelerated some attitude indicators will precess and incorrectly indicate a group of answer choices?

Figure 83
  • Answer (A) is correct. When indicating 12,000 ft., an altimeter has the long hand (100-ft. intervals) on 0, the short hand (1,000-ft. intervals) on 2, and the thin hand with flared tip (10,000-ft. intervals) just over 1, as shown in altimeter 4.
  • Answer (B) is incorrect because, in altimeter 3, the short hand is on 1, the long hand is on 2, and the thin hand is above 1, indicating about 11,200 ft.
  • Answer (C) is incorrect because, in altimeter 2, the thin hand is above 2 and the short hand is on 1, indicating 21,000 ft.

[29]

Altimeter setting is the value to which the scale of the pressure altimeter is set so the altimeter indicates
  1. true altitude at field elevation.
  2. pressure altitude at sea level.
  3. pressure altitude at field elevation.
  • Answer (A) is correct. The altimeter setting is the value which permits the altimeter to indicate the true altitude at field elevation. Within the vicinity of a particular airport, the altimeter setting provides an accurate means of separating traffic vertically and also provides obstruction clearance. Note that the use of the word "true" here is not completely accurate because the altimeter setting does not correct for nonstandard temperature.
  • Answer (B) is incorrect because the altimeter setting causes the altimeter to indicate true altitude, not pressure altitude, at field elevation, not sea level.
  • Answer (C) is incorrect because the altimeter setting causes the altimeter to indicate true altitude, not pressure altitude.

[30]

Altimeter setting is the value to which the scale of the pressure altimeter is set so the altimeter indicates
  1. pressure altitude at sea level.
  2. true altitude at field elevation.
  3. pressure altitude at field elevation.
  • Answer (A) is incorrect because the altimeter setting causes the altimeter to indicate true altitude, not pressure altitude, at field elevation, not sea level.
  • Answer (B) is correct. The altimeter setting is the value which permits the altimeter to indicate the true altitude at field elevation. Within the vicinity of a particular airport, the altimeter setting provides an accurate means of separating traffic vertically and also provides obstruction clearance. Note that the use of the word "true" here is not completely accurate because the altimeter setting does not correct for nonstandard temperature.
  • Answer (C) is incorrect because the altimeter setting causes the altimeter to indicate true altitude, not pressure altitude.

[31]

If you are departing from an airport where you cannot obtain an altimeter setting, you should set your altimeter
  1. on the current airport barometric pressure, if known.
  2. to the airport elevation.
  3. on 29.92" Hg.
  • Answer (A) is incorrect because the altimeter setting is the current barometric pressure adjusted to sea level.
  • Answer (B) is correct. If you cannot obtain an altimeter setting at an airport, adjust your altimeter so it reads the current airport elevation.
  • Answer (C) is incorrect because setting the altimeter to 29.92 would provide only pressure altitude.

[32]

Which altitude is indicated when the altimeter is set to 29.92" Hg?
  1. Pressure.
  2. Density.
  3. Standard.
  • Answer (A) is correct. Pressure altitude is indicated when the altimeter is set to 29.92.
  • Answer (B) is incorrect because density altitude is pressure altitude corrected for nonstandard temperature.
  • Answer (C) is incorrect because standard altitude is a nonsense concept in this context.

[33]

The pressure altitude at a given location is indicated on the altimeter after the altimeter is set to
  1. the field elevation.
  2. 29.92" Hg.
  3. the current altimeter setting.
  • Answer (A) is incorrect because you set the altimeter to a barometric pressure, not an elevation.
  • Answer (B) is correct. Pressure altitude by definition is indicated altitude when the altimeter setting is 29.92. This is true regardless of area or true altitude.
  • Answer (C) is incorrect because the altitude indicated on the altimeter after it is set to the current altimeter setting is the indicated altitude, not pressure altitude.

[34]

How can you determine the pressure altitude on an airport without a tower or FSS?
  1. Use your computer and correct the field elevation for temperature.
  2. Set the altimeter to 29.92" Hg and read the altitude indicated.
  3. Set the altimeter to the current altimeter setting of a station within 100 miles and correct this indicated altitude with local temperature.
  • Answer (A) is incorrect because correcting field elevation for temperature is a nonsense concept.
  • Answer (B) is correct. The pressure altitude is determined by setting your altimeter to 29.92 and reading the altitude indicated. No matter what altitude or location, the altimeter reads pressure altitude when the barometric window is set to 29.92" Hg.
  • Answer (C) is incorrect because it describes true altitude.

[35]

How can you obtain the pressure altitude on flights below 18,000 feet?
  1. Use your computer to change the indicated altitude to pressure altitude.
  2. Set your altimeter to 29.92" Hg.
  3. Contact an FSS and ask for the pressure altitude.
  • Answer (A) is incorrect because it is much easier to just change the altimeter to 29.92.
  • Answer (B) is correct. Pressure altitude can be determined anywhere by setting the altimeter to 29.92.
  • Answer (C) is incorrect because pressure altitude is determined by setting your altimeter to 29.92, not by contacting an FSS.

[36]

At an altitude of 6,500 feet MSL, the current altimeter setting is 30.42" Hg. The pressure altitude would be approximately
  1. 6,500 feet.
  2. 7,500 feet.
  3. 6,000 feet.
  • Answer (A) is incorrect because pressure altitude is equal to actual altitude only when the current altimeter setting is 29.92.
  • Answer (B) is incorrect because a pressure change of 0.5" Hg results in an altitude change of 500 ft., not 1,000 ft.
  • Answer (C) is correct. Rotating the altimeter's setting knob to a higher (or lower) barometric setting moves the hands to higher (or lower) indicated altitude at the rate of 1" Hg to 1,000 ft. of altitude. Given the current altimeter setting of 30.42 and the indicated altitude of 6,500 ft., resetting the window to 29.92 would lower indicated altitude by 500 ft. (30.42 - 29.92 = 0.5" Hg = 500 ft.). The new indicated altitude would be 6,000 ft. MSL, which would be pressure altitude.

[37]

How does a pilot normally obtain the current altimeter setting during an IFR flight in Class E airspace below 18,000 feet?
  1. ATC periodically advises the pilot of the proper altimeter setting.
  2. The pilot should contact ARTCC at least every 100 NM and request the altimeter setting.
  3. FSS's along the route broadcast the weather information at 15 minutes past the hour.
  • Answer (A) is correct. During IFR flight in Class E airspace below 18,000 ft. MSL, ATC periodically provides altimeter settings. Thus, you will continually hear the altimeter settings given to other pilots as you monitor ATC.
  • Answer (B) is incorrect because ATC provides the altimeter setting without being asked.
  • Answer (C) is incorrect because FSSs do not broadcast the altimeter setting at 15 min. past the hour.

[38]

Which of the following defines the type of altitude used when maintaining FL 210?
  1. Indicated.
  2. Pressure.
  3. Calibrated.
  • Answer (A) is incorrect because indicated altitude is the altitude shown on the altimeter after it is set to the current altimeter setting. Pressure altitude, not indicated altitude, is used at or above 18,000 ft. MSL.
  • Answer (B) is correct. Above 18,000 ft. MSL (FL 180), pressure altitude is used to separate traffic. Pressure altitude is the altitude indicated on an altimeter when it is set to 29.92.
  • Answer (C) is incorrect because all altimeters are calibrated.

[39]

What is the procedure for setting the altimeter when assigned an IFR altitude of 18,000 feet or higher on a direct flight off airways?
  1. Set the altimeter to the current reported setting for climb-out and 29.92" Hg upon reaching 18,000 feet.
  2. Set the altimeter to the current altimeter setting until reaching the assigned altitude, then set to 29.92" Hg.
  3. Set the altimeter to 29.92" Hg before takeoff.
  • Answer (A) is correct. When you are at an altitude of 18,000 ft. MSL (FL 180) or higher, the altimeter should be set to 29.92, which is pressure altitude. It does not matter whether you are on an airway.
  • Answer (B) is incorrect because you should set the altimeter to 29.92 upon reaching FL 180, not upon reaching your assigned altitude.
  • Answer (C) is incorrect because indicated altitude (using local altimeter settings) should be used up to FL 180.

[40]

While you are flying at FL 250, you hear ATC give an altimeter setting of 28.92" Hg in your area. At what pressure altitude are you flying?
  1. 26,000 feet.
  2. 24,000 feet.
  3. 25,000 feet.
  • Answer (A) is incorrect because FL 250 by definition means a pressure altitude of 25,000 ft., not 26,000 ft.
  • Answer (B) is incorrect because FL 250 by definition means a pressure altitude of 25,000 ft., not 24,000 ft.
  • Answer (C) is correct. The pressure altitude is 25,000 ft. in your area because at FL 250 your altimeter should be set at 29.92. A flight level (FL) by definition means a pressure altitude.

[41]

En route at FL 290, the altimeter is set correctly, but not reset to the local altimeter setting of 30.57" Hg during descent. If the field elevation is 650 feet and the altimeter is functioning properly, what is the approximate indication upon landing?
  1. 1,300 feet.
  2. 715 feet.
  3. Sea level.
  • Answer (A) is incorrect because 1,300 ft. is obtained by adding 650 ft. rather than subtracting 650 ft.
  • Answer (B) is incorrect because 715 ft. is obtained by adding 65 ft. rather than subtracting 650 ft.
  • Answer (C) is correct. One in. of pressure equals approximately 1,000 ft. of altitude. If an altimeter should be set to 30.57 but is set to 29.92, it is set .65" Hg too low and thus will indicate 650 ft. (1,000 ft. x .65) less than actual altitude. Thus, if the airplane lands at an airport with a field elevation of 650 ft., the altimeter will indicate sea level, i.e., 650 ft. elevation minus the 650 ft. altimeter setting error.

[42]

En route at FL 290, your altimeter is set correctly, but you fail to reset it to the local altimeter setting of 30.26" Hg during descent. If the field elevation is 134 feet and your altimeter is functioning properly, what will it indicate after landing?
  1. 206 feet below MSL.
  2. 474 feet MSL.
  3. 100 feet MSL.
  • Answer (A) is correct. One in. of pressure equals approximately 1,000 ft. of altitude. If an altimeter should be set to 30.26 but is set to 29.92, it is set .34" Hg too low and thus will indicate 340 ft. (1,000 ft. x .34) less than actual altitude. Thus, if the airplane lands at an airport with a field elevation of 134 ft., the altimeter will indicate 206 ft. below sea level (134 ft. airport elevation - 340 ft. altimeter setting error).
  • Answer (B) is incorrect because 474 ft. is obtained by adding 340 ft. rather than subtracting 340 ft.
  • Answer (C) is incorrect because 100 ft. is obtained by subtracting 34 ft. rather than subtracting 340 ft.

[43]

Under which condition will pressure altitude be equal to true altitude?
  1. When standard atmospheric conditions exist.
  2. When the atmospheric pressure is 29.92" Hg.
  3. When indicated altitude is equal to the pressure altitude.
  • Answer (A) is correct. Pressure altitude will equal true altitude when standard atmospheric conditions exist at the current altitude, i.e, 29.92" Hg and 15°C at sea level.
  • Answer (B) is incorrect because, to get true altitude from an altimeter, both temperature and pressure must be standard.
  • Answer (C) is incorrect because nonstandard temperatures will make indicated altitude deviate from true altitude.

[44]

Under what condition is pressure altitude and density altitude the same value?
  1. When indicated, and pressure altitudes are the same value on the altimeter.
  2. At standard temperature.
  3. When the altimeter setting is 29.92" Hg.
  • Answer (A) is incorrect because neither indicated nor pressure altitude is adjusted for nonstandard temperature.
  • Answer (B) is correct. Density altitude, by definition, is pressure altitude adjusted for nonstandard temperature. Accordingly, pressure altitude will equal density altitude at standard temperature.
  • Answer (C) is incorrect because pressure altitude has not been adjusted for nonstandard temperature.

[45]

Under what condition will true altitude be lower than indicated altitude with an altimeter setting of 29.92" Hg?
  1. In colder than standard air temperature.
  2. When density altitude is higher than indicated altitude.
  3. In warmer than standard air temperature.
  • Answer (A) is correct. When temperature lowers en route, you are lower than your altimeter indicates. Similarly, when you are in colder-than-standard air temperatures, true altitude is lower than pressure altitude.
  • Answer (B) is incorrect because, assuming an altimeter setting of 29.92, if density altitude is higher than indicated altitude, the air is thinner and thus warmer. When you are in warmer-than-standard air, the true altitude is above indicated altitude.
  • Answer (C) is incorrect because, when you are in warmer-than-standard air, the true altitude is above pressure altitude.

[46]

When an altimeter is changed from 30.11" Hg to 29.96" Hg, in which direction will the indicated altitude change and by what value?
  1. Altimeter will indicate 150 feet higher.
  2. Altimeter will indicate 150 feet lower.
  3. Altimeter will indicate 15 feet lower.
  • Answer (A) is incorrect because, when the altimeter setting is lowered, indicated altitude decreases, not increases.
  • Answer (B) is correct. Altimeter settings vary approximately 1" Hg for each 1,000 ft. of altitude. When the altimeter setting is changed from 30.11 to 29.96, it is lower by .15" Hg. Thus, the altimeter will indicate 150 ft. lower (1,000 ft. x .15). Remember that the indicated altitude and altimeter setting vary directly; i.e., when the altimeter setting is adjusted up, indicated altitude increases and vice versa.
  • Answer (C) is incorrect because .15" Hg is equal to 150 ft., not 15 ft.

[47]

Which condition would cause the altimeter to indicate a lower altitude than actually flown (true altitude)?
  1. Air temperature lower than standard.
  2. Air temperature warmer than standard.
  3. Atmospheric pressure lower than standard.
  • Answer (A) is incorrect because in colder-than-standard air, pressure levels drop. Thus, the altimeter reads higher, not lower, than true altitude.
  • Answer (B) is correct. When temperatures are warmer than standard, the pressure levels are raised. That is, the altimeter will indicate an altitude lower than that at which the aircraft is actually flying.
  • Answer (C) is incorrect because using the correct altimeter setting adjusts for nonstandard pressure.

[48]

Which practical test should be made on the electric gyro instruments prior to starting an engine?
  1. Check that the electrical connections are secure on the back of the instruments.
  2. Turn on the electrical power and listen for any unusual or irregular mechanical noise.
  3. Check that the attitude of the miniature aircraft is wings level before turning on electrical power.
  • Answer (A) is incorrect because pilots should not be handling or touching electrical connections behind the instrument panel.
  • Answer (B) is correct. Electric gyro instruments can be checked for irregular noises by listening to them after the battery is turned on but before the engine is started. You should notice any bearing clatters, clicks, or other unusual noises. Additionally, you can look at the instruments to see if they are in their expected positions. There are additional tests to pursue after the engine is started.
  • Answer (C) is incorrect because, due to the construction of the turn coordinator, the miniature aircraft will be wings level before turning on the electrical power. Thus, this indication would not be a practical test for the electric gyro.

[49]

One characteristic that a properly functioning gyro depends upon for operation is the
  1. deflecting force developed from the angular velocity of the spinning wheel.
  2. resistance to deflection of the spinning wheel or disc.
  3. ability to resist precession 90° to any applied force.
  • Answer (A) is incorrect because the deflective force is applied to, not developed by, the gyro.
  • Answer (B) is correct. Newton's second law of motion states that the deflection of a moving body is proportional to the deflective force applied and is inversely proportional to its weight and speed. For a gyro, the resistance to deflection is proportional to the deflective force. This property is used by the attitude indicator.
  • Answer (C) is incorrect because precession is the name for the reaction to deflection. The gyro uses precession, not resists it.

[50]

What pre-takeoff check should be made of a vacuum-driven heading indicator in preparation for an IFR flight?
  1. After 5 minutes, check that the heading indicator card aligns itself with the magnetic heading of the aircraft.
  2. Determine that the heading indicator does not precess more than 2° in 5 minutes of ground operation.
  3. After 5 minutes, set the indicator to the magnetic heading of the aircraft and check for proper alignment after taxi turns.
  • Answer (A) is incorrect because non-slaved heading indicators must be manually set to the correct magnetic heading.
  • Answer (B) is incorrect because a precession error of no more than 3° in 15 min., not 2° in 5 min., is acceptable for normal operations.
  • Answer (C) is correct. Vacuum-driven gyros take several minutes to get up to speed. After about 5 min., set the heading indicator to the correct magnetic heading. After taxiing to the runup area, verify that the heading indicator still indicates the correct magnetic heading.

[51]

(Refer to figure 143.) The heading on a remote indicating compass is 120³ and the magnetic compass indicates 110³. What action is required to correctly align the heading indicator with the magnetic compass?
  1. Select the free gyro mode and depress the counter-clockwise heading drive button.
  2. Select the free gyro mode and depress the clockwise heading drive button.
  3. Select the slaved gyro mode and depress the clockwise heading drive button.
When an aircraft is decelerated some attitude indicators will precess and incorrectly indicate a group of answer choices?

Figure 143
  • Answer (A) is incorrect because using the counter-clockwise drive button would increase the indicated headings, e.g., from 120° to 130° indicated.
  • Answer (B) is correct. Some remote indicating compasses (RIC) have a separate slaving control and compensator unit with a deviation meter. The instrument shows any difference between the displayed heading and the magnetic heading. It also provides the controls shown in Fig. 143 to correct any errors. To correct from 120° indicated on the RIC to the magnetic heading of 110°, select the free gyro mode and depress the clockwise heading drive button until 110° heading on RIC is indicated and the deviation meter is centered.
  • Answer (C) is incorrect because the RIC must be in the free gyro mode to use the heading drive button.

[52]

(Refer to figure 143.) When the system is in the free gyro mode, depressing the clockwise manual heading drive button will rotate the remote indicating compass card to the
  1. right to eliminate right compass card error.
  2. left to eliminate left compass card error.
  3. right to eliminate left compass card error.
When an aircraft is decelerated some attitude indicators will precess and incorrectly indicate a group of answer choices?

Figure 143
  • Answer (A) is incorrect because you adjust to the right for left, not right, error.
  • Answer (B) is incorrect because the clockwise manual heading drive button rotates the compass card to the right, not left.
  • Answer (C) is correct. A left compass card error means that the card has rotated too far to the left; i.e., it reads too high a heading. The clockwise adjustment rotates the indicating compass card to the right to eliminate left compass card error. For example, this would change a 120° heading to 110°.

[53]

(Refer to figure 143.) The heading on a remote indicating compass is 5° to the left of that desired. What action is required to move the desired heading under the heading reference?
  1. Select the free gyro mode and depress the counter-clockwise heading drive button.
  2. Select the free gyro mode and depress the clockwise heading drive button.
  3. Select the slaved gyro mode and depress the clockwise heading drive button.
When an aircraft is decelerated some attitude indicators will precess and incorrectly indicate a group of answer choices?

Figure 143
  • Answer (A) is correct. If the heading on the compass is left of that desired, the compass has rotated too far to the right, which is a right compass error. Thus, the slaving control must be placed in the free mode, and the counterclockwise drive button must be depressed.
  • Answer (B) is incorrect because the clockwise heading drive button would correct a left, not a right, card error.
  • Answer (C) is incorrect because the RIC must be in the free gyro mode to use the heading drive button and the counterclockwise heading drive button should be depressed.

[54]

What pre-takeoff check should be made of the attitude indicator in preparation for an IFR flight?
  1. The miniature airplane should erect and become stable within 5 minutes.
  2. The horizon bar does not vibrate during warmup.
  3. The horizon bar should erect and become stable within 5 minutes.
  • Answer (A) is incorrect because the miniature airplane can be adjusted manually but otherwise remains stationary. It is the horizon bar that moves within the instrument face.
  • Answer (B) is incorrect because the horizon bar usually vibrates during warm-up as the gyro gets up to speed.
  • Answer (C) is correct. The pretakeoff check for attitude indicators is that, within the 5-min. warm-up, the horizon bar should erect to the horizontal position and remain at the correct position. It should remain stable during straight taxiing and taxi turns.

[55]

Which condition during taxi is an indication that an attitude indicator is unreliable?
  1. The horizon bar vibrates during warmup.
  2. The horizon bar does not align itself with the miniature airplane after warmup.
  3. The horizon bar tilts more than 5° while making taxi turns.
  • Answer (A) is incorrect because the horizon bar will vibrate as the gyros get up to speed during warm-up.
  • Answer (B) is incorrect because the horizon bar aligns itself with the center of the dial, indicating level flight. Then the miniature airplane must be adjusted to align with the horizon bar from the pilot's perspective.
  • Answer (C) is correct. The horizon bar in an attitude indicator should not tilt more than 5° during a taxi turn. If it does, it is unreliable and should not be used for IFR flight.

[56]

During coordinated turns, which force moves the pendulous vanes of a vacuum-driven attitude indicator resulting in precession of the gyro toward the inside of the turn?
  1. Deceleration.
  2. Centrifugal.
  3. Acceleration.
  • Answer (A) is incorrect because deceleration is a force that induces climb/descent errors in the attitude indicator.
  • Answer (B) is correct. The attitude indicator normally erects itself by discharging air equally through four exhaust ports, each of which is partially covered by a pendulous vane. During coordinated (and skidding) turns, centrifugal force moves the vanes from their vertical position, precessing the gyro toward the inside of the turn.
  • Answer (C) is incorrect because acceleration is a force that induces climb/descent errors in the attitude indicator.

[57]

If a 180° steep turn is made to the right and the aircraft is rolled out to straight-and-level flight by visual references, the attitude indicator
  1. may show a slight climb and turn.
  2. should immediately show straight-and-level flight.
  3. show a slight skid and climb to the right.
  • Answer (A) is correct. In 180° coordinated steep turns, when the airplane is rolled out to straight-and-level flight, the attitude indicator will indicate a turn to the opposite direction along with a slight climb.
  • Answer (B) is incorrect because, when the airplane is rolled out of a coordinated 180° steep turn to straight-and-level flight, the attitude indicator will indicate a turn to the opposite direction with a slight climb; it will not immediately show straight-and-level flight.
  • Answer (C) is incorrect because evidence of a skid error cannot be read from the attitude indicator, and the turn error would be to the left, not to the right.

[58]

During normal coordinated turns, what error due to precession should you observe when rolling out to straight-and-level flight from a 180° steep turn to the right?
  1. The miniature aircraft would show a slight turn indication to the left.
  2. A straight-and-level coordinated flight indication.
  3. The miniature aircraft would show a slight descent and wings-level attitude.
  • Answer (A) is correct. In 180° coordinated steep turns, when the airplane is rolled out to straight-and-level flight, the attitude indicator will indicate a turn to the opposite direction along with a slight climb. Thus, the indicated turn would be to the left if the steep turn were made to the right.
  • Answer (B) is incorrect because the attitude indicator will show a slight climb to the left, not straight-and-level flight, and it does not indicate coordinated flight.
  • Answer (C) is incorrect because it will indicate a slight climb, not a descent, and it will indicate a turning error.

[59]

During normal operation of a vacuum-driven attitude indicator, what attitude indication should you see when rolling out from a 180° skidding turn to straight-and-level coordinated flight?
  1. A straight-and-level coordinated flight indication.
  2. The miniature aircraft shows a turn in the direction opposite the skid.
  3. A nose-high indication relative to level flight.
  • Answer (A) is incorrect because the attitude indicator will show a slight turn in the opposite direction, not straight-and-level flight, and it does not indicate coordinated flight.
  • Answer (B) is correct. Similarly to coordinated turns, the attitude indicator will show a bank in the opposite direction when the aircraft is rolled out of a skidding turn. However, there will be no nose-up indication.
  • Answer (C) is incorrect because a nose-high indication will result only when the aircraft is rolling out of a coordinated turn.

[60]

Errors in both pitch and bank indication on an attitude indicator are usually at a maximum as the aircraft rolls out of a
  1. 360° turn.
  2. 180° turn.
  3. 270° turn.
  • Answer (A) is incorrect because pitch and bank errors are usually the greatest after 180°, not 360°, of turn.
  • Answer (B) is correct. Errors for both pitch and bank indication on an attitude indicator are greatest when rolling out of a 180° turn. This precession error, normally between 3° and 5°, is self-correcting by the part of the heading indicator called the erecting mechanism.
  • Answer (C) is incorrect because pitch and bank errors are usually the greatest after 180°, not 270°, of turn.

[61]

When an aircraft is decelerated, some attitude indicators will precess and incorrectly indicate a
  1. climb.
  2. descent.
  3. left turn.
  • Answer (A) is incorrect because acceleration, not deceleration, may result in precession-related errors indicating a climb.
  • Answer (B) is correct. Deceleration affects some attitude indicators through precession to incorrectly indicate a descent.
  • Answer (C) is incorrect because acceleration and deceleration result in erroneous pitch readings, not turning errors.

[62]

When an aircraft is accelerated, some attitude indicators will precess and incorrectly indicate a
  1. right turn.
  2. climb.
  3. descent.
  • Answer (A) is incorrect because acceleration and deceleration result in erroneous pitch readings, not turning errors.
  • Answer (B) is correct. Acceleration affects some attitude indicators through precession to incorrectly indicate a climb.
  • Answer (C) is incorrect because deceleration, not acceleration, may result in precession-related errors indicating a descent.

[63]

Prior to starting an engine, you should check the turn-and-slip indicator to determine if the
  1. needle indication properly corresponds to the angle of the wings or rotors with the horizon.
  2. needle is approximately centered and the tube is full of fluid.
  3. ball will move freely from one end of the tube to the other when the aircraft is rocked.
  • Answer (A) is incorrect because the needle corresponds to rate and direction of turn, not wing or rotor angle.
  • Answer (B) is correct. Prior to starting an engine, you should determine that the needle of the turn-and-slip indicator is approximately centered and that the tube is full of fluid with the ball also approximately centered if the airplane is on a level surface.
  • Answer (C) is incorrect because rocking the aircraft is not necessary. Freedom of movement of the ball is checked during taxi.

[64]

What indications should you observe on the turn-and-slip indicator during taxi?
  1. The ball moves freely opposite the turn, and the needle deflects in the direction of the turn.
  2. The ball deflects opposite the turn, but the needle remains centered.
  3. The needle deflects in the direction of the turn, but the ball remains centered.
  • Answer (A) is correct. During taxi, the turn-and-slip indicator ball should move freely opposite the direction of any turn since centrifugal force forces the ball to the outside. Also, the rate of turn indicator should indicate a turn in the proper direction.
  • Answer (B) is incorrect because the needle deflects in the direction of the turn.
  • Answer (C) is incorrect because the ball will deflect opposite to the direction of the turn.

[65]

What indications are displayed by the miniature aircraft of a turn coordinator?
  1. Direct indication of bank angle and pitch attitude.
  2. Indirect indication of bank angle and pitch attitude.
  3. Rate of roll and rate of turn.
  • Answer (A) is incorrect because the turn coordinator only indirectly indicates bank angle and has no relationship to pitch attitude.
  • Answer (B) is incorrect because the turn coordinator has no relationship to pitch attitude.
  • Answer (C) is correct. The turn coordinator indicates rate of roll and rate of turn. When the bank is constant, the rate of turn is indicated. When the bank is changing, the rate of roll is also indicated.

[66]

What does the miniature aircraft of the turn coordinator directly display?
  1. Angle of bank and rate of turn.
  2. Rate of roll and rate of turn.
  3. Angle of bank.
  • Answer (A) is incorrect because the angle of bank is not directly indicated by the turn coordinator.
  • Answer (B) is correct. The turn coordinator indicates rate of roll and rate of turn. When the bank is constant, the rate of turn is indicated. When the bank is changing, the rate of roll is also indicated.
  • Answer (C) is incorrect because the angle of bank is not directly indicated by the turn coordinator.

[67]

What indication is presented by the miniature aircraft of the turn coordinator?
  1. Indirect indication of the bank attitude.
  2. Quality of the turn.
  3. Direct indication of the bank attitude and the quality of the turn.
  • Answer (A) is correct. The miniature aircraft of the turn coordinator indicates the rate of roll when bank is changing. When the rotation around the longitudinal axis is zero, the instrument indicates the rate of turn. Thus, it provides only an indirect indication of the angle of bank.
  • Answer (B) is incorrect because the ball in the turn coordinator, not the miniature aircraft, provides information on the quality of the turn.
  • Answer (C) is incorrect because the turn coordinator does NOT provide a direct indication of bank.

[68]

The displacement of a turn coordinator during a coordinated turn will
  1. increase as angle of bank increases.
  2. remain constant for a given bank regardless of airspeed.
  3. indicate the angle of bank.
  • Answer (A) is correct. The displacement of a turn coordinator increases as angle of bank (in coordinated flight) increases because the turn coordinator shows rate of turn, which increases as angle of bank increases.
  • Answer (B) is incorrect because, when the rate of roll is zero, the turn coordinator provides information concerning the rate of turn, which in turn changes as airspeed changes given a constant bank.
  • Answer (C) is incorrect because the angle of bank is only indirectly indicated.

[69]

What indication should be observed on a turn coordinator during a left turn while taxiing?
  1. Both the miniature aircraft and the ball will remain centered.
  2. The miniature aircraft will show a turn to the left and the ball moves to the right.
  3. The miniature aircraft will show a turn to the left and the ball remains centered.
  • Answer (A) is incorrect because the miniature aircraft reacts to yaw and shows a turn to the left, and the ball reacts to centrifugal force by moving right.
  • Answer (B) is correct. On a taxiing turn to the left, the turn coordinator shows a turn to the left, and the ball moves to the right. The centrifugal force of the turn, which is not offset by bank when taxiing, forces the ball opposite to the turn.
  • Answer (C) is incorrect because the ball moves to the outside of the turn due to centrifugal force.

[70]

What force causes an airplane to turn?
  1. Vertical lift component.
  2. Rudder pressure or force around the vertical axis.
  3. Horizontal lift component.
  • Answer (A) is incorrect because the vertical component of lift counteracts weight and thus affects altitude.
  • Answer (B) is incorrect because the rudder pressure coordinates flight only when the airplane is banked.
  • Answer (C) is correct. An airplane, like any object, requires a sideward force to make it turn. This force is supplied by banking the airplane so that lift is separated into two components at right angles to each other. The lift acting upward and opposing weight is the vertical lift component, and the lift acting horizontally and opposing centrifugal force is the horizontal lift component. The horizontal lift component is the sideward force that causes an airplane to turn.

[71]

The rate of turn at any airspeed is dependent upon
  1. centrifugal force.
  2. the vertical lift component.
  3. the horizontal lift component.
  • Answer (A) is incorrect because centrifugal force acts against the horizontal lift component, thus acting against turning the airplane.
  • Answer (B) is incorrect because the vertical component of lift determines altitude and change in altitude.
  • Answer (C) is correct. At a given airspeed, the rate at which an airplane turns depends upon the amount of the horizontal component of lift.

[72]

What is the relationship between centrifugal force and the horizontal lift component in a coordinated turn?
  1. Horizontal lift and centrifugal force are equal.
  2. Centrifugal force exceeds horizontal lift.
  3. Horizontal lift exceeds centrifugal force.
  • Answer (A) is correct. When a turn is coordinated, horizontal lift equals centrifugal force. This is indicated when the ball on the turn coordinator or turn-and-slip indicator is centered.
  • Answer (B) is incorrect because, when centrifugal force exceeds horizontal lift, there is a skidding turn.
  • Answer (C) is incorrect because, when horizontal lift exceeds centrifugal force, there is a slipping turn.

[73]

The primary reason the angle of attack must be increased, to maintain a constant altitude during a coordinated turn, is because the
  1. thrust is acting in a different direction, causing a reduction in airspeed and loss of lift.
  2. vertical component of lift has decreased as the result of the bank.
  3. use of ailerons has increased the drag.
  • Answer (A) is incorrect because thrust is always a for ward-acting force. The reduction in airspeed (assuming constant power) is due to an increase in angle of attack to compensate for the loss of vertical lift in a turn, i.e., to maintain altitude.
  • Answer (B) is correct. In comparison to level flight, a bank results in the division of lift between vertical and horizontal components. To provide a vertical component of lift sufficient to maintain altitude in a level turn, an increase in the angle of attack is required.
  • Answer (C) is incorrect because in a coordinated turn the ailerons are streamlined and no aileron drag exists. When entering or recovering from turns, you can counteract the adverse yaw caused by aileron drag by use of the rudder.

[74]

(Refer to figure 144.) What changes in control displacement should be made so that "2" would result in a coordinated standard-rate turn?
  1. Increase left rudder and decrease rate of turn.
  2. Decrease left rudder and decrease angle of bank.
  3. Increase left rudder and increase rate of turn.
When an aircraft is decelerated some attitude indicators will precess and incorrectly indicate a group of answer choices?

Figure 144
  • Answer (A) is incorrect because the rate of turn must be increased, not decreased, to establish a standard-rate turn.
  • Answer (B) is incorrect because left rudder pressure must be increased, not decreased, in a slip to the left.
  • Answer (C) is correct. Illustration 2 in Fig. 144 indicates a slip, in which the rate of turn is too slow for the angle of bank, and the lack of centrifugal force causes the ball to move to the inside of the turn. To return to a coordinated standard-rate turn, you should increase left rudder (i.e., step on the ball) and increase the rate of turn. A standard-rate turn is indicated when the needle is on the "doghouse" (i.e., standard rate) mark. It is presently indicating a half-standard-rate turn.

[75]

(Refer to figure 144.) Which illustration indicates a coordinated turn?
When an aircraft is decelerated some attitude indicators will precess and incorrectly indicate a group of answer choices?

Figure 144
  • Answer (A) is incorrect because illustration 2 shows a slipping turn, in which centrifugal force is less than the horizontal component of lift.
  • Answer (B) is incorrect because illustration 1 shows a skidding turn, in which centrifugal force exceeds the horizontal component of lift.
  • Answer (C) is correct. A coordinated turn is one in which the ball is centered as indicated in illustration 3. The horizontal component of lift equals centrifugal force.

[76]

(Refer to figure 144.) What changes in control displacement should be made so that "1" would result in a coordinated standard rate turn?
  1. Increase right rudder and decrease rate of turn.
  2. Increase right rudder and increase rate of turn.
  3. Decrease right rudder and increase angle of bank.
When an aircraft is decelerated some attitude indicators will precess and incorrectly indicate a group of answer choices?

Figure 144
  • Answer (A) is incorrect because the rate of turn must be increased, not decreased.
  • Answer (B) is correct. Illustration 1 in Fig. 144 indicates a skid, in which the rate of turn is too great for the angle of bank, and excessive centrifugal force causes the ball to move to the outside of the turn. To return to coordinated flight, you should increase right rudder (i.e., step on the ball) to center the ball. A standard-rate turn is indicated when the needle is on the doghouse mark. Thus, the rate of turn must be increased to result in a standard-rate turn.
  • Answer (C) is incorrect because right rudder must be increased, not decreased.

[77]

(Refer to figure 144.) Which illustration indicates a skidding turn?
When an aircraft is decelerated some attitude indicators will precess and incorrectly indicate a group of answer choices?

Figure 144
  • Answer (A) is correct. A skidding turn occurs when centrifugal force is greater than horizontal lift. As shown by illustration 1, the ball is outside the turn.
  • Answer (B) is incorrect because illustra tion 3 shows a coordinated turn.
  • Answer (C) is incorrect because illustration 2 shows a slipping turn.

[78]

(Refer to figure 144.) Which illustration indicates a slipping turn?
When an aircraft is decelerated some attitude indicators will precess and incorrectly indicate a group of answer choices?

Figure 144
  • Answer (A) is incorrect because illustration 3 shows a coordinated turn.
  • Answer (B) is incorrect because illustration 1 shows a skidding turn.
  • Answer (C) is correct. A slipping turn is one in which the centrifugal force is less than horizontal lift. As shown by illustration 2, the ball is inside the turn.

[79]

During a skidding turn to the right, what is the relationship between the component of lift, centrifugal force, and load factor?
  1. Centrifugal force is less than horizontal lift and the load factor is increased.
  2. Centrifugal force is greater than horizontal lift and the load factor is increased.
  3. Centrifugal force and horizontal lift are equal and the load factor is decreased.
  • Answer (A) is incorrect because a slipping, not skidding, turn occurs when centrifugal force is less than horizontal lift.
  • Answer (B) is correct. In skidding turns, centrifugal force is greater than horizontal lift. The load factor increases in level turns.
  • Answer (C) is incorrect because centrifugal force and horizontal lift are equal in a coordinated, not skidding, turn, and in a level turn the load factor is increased, not decreased.

[80]

When airspeed is increased in a turn, what must be done to maintain a constant altitude?
  1. Increase the angle of bank and/or decrease the angle of attack.
  2. Decrease the angle of bank.
  3. Decrease the angle of attack.
  • Answer (A) is correct. To compensate for added lift, which would result if airspeed were increased during a turn, the angle of attack must be decreased and the angle of bank increased if a constant altitude is to be maintained.
  • Answer (B) is incorrect because the angle of bank must be increased, not decreased.
  • Answer (C) is incorrect because, as an alternative, the angle of bank can be increased.

[81]

When airspeed is decreased in a turn, what must be done to maintain level flight?
  1. Decrease the angle of bank and/or increase the angle of attack.
  2. Increase the angle of attack.
  3. Increase the angle of bank and/or decrease the angle of attack.
  • Answer (A) is correct. To compensate for the decreased lift resulting from decreased airspeed during a turn, the angle of bank must be decreased and/or the angle of attack increased.
  • Answer (B) is incorrect because the angle of bank can be decreased as well as the angle of attack increased.
  • Answer (C) is incorrect because the increased vertical lift required must be obtained by a decrease, not an increase, in angle of bank and/or an increase, not a decrease, in angle of attack.

[82]

When airspeed is decreased in a turn, what must be done to maintain level flight?
  1. Decrease the angle of bank and/or increase the pitch attitude.
  2. Increase the pitch attitude and/or increase the angle of bank.
  3. Increase the angle of bank and/or decrease the pitch attitude.
  • Answer (A) is correct. To compensate for the decreased lift resulting from decreased airspeed during a turn, the angle of bank must be decreased and/or the angle of attack increased by increasing the pitch attitude.
  • Answer (B) is incorrect because the increased vertical lift required must be obtained by an increase in the pitch attitude and/or a decrease, not an increase, in the angle of bank.
  • Answer (C) is incorrect because the increased vertical lift required must be obtained by a decrease, not an increase, in the angle of bank and/or an increase, not a decrease, in the pitch attitude.

[83]

If a standard-rate turn is maintained, how much time would be required to turn to the left from a heading of 090° to a heading of 300°?
  1. 40 seconds.
  2. 50 seconds.
  3. 30 seconds.
  • Answer (A) is incorrect because, at standard-rate of turn, an airplane would turn left 120° to a heading of 330°, not 300°, in 40 sec.
  • Answer (B) is correct. A standard-rate turn means an airplane is turning at a rate of 3°/sec. A left turn from 090° to 300° is a total of 150° (90° to north and another 60° to 300°). Thus, at standard-rate, it would take 50 sec. (150° ÷ 3°/sec.).
  • Answer (C) is incorrect because, at standard-rate of turn, an airplane would turn left 90° to a heading of 360°, not 300°, in 30 sec.

[84]

If a half-standard-rate turn is maintained, how long would it take to turn 135°?
  1. 1 minute.
  2. 1 minute 20 seconds.
  3. 1 minute 30 seconds.
  • Answer (A) is incorrect because an airplane would turn 90°, not 135°, in 1 min. at half-standard-rate.
  • Answer (B) is incorrect because an airplane would turn 120°, not 135°, in 1 min. 20 sec. at half-standard-rate.
  • Answer (C) is correct. A standard-rate turn means an airplane is turning at a rate of 3°/sec. Thus, a half-standard-rate is a turn at the rate of 1.5°/sec. To turn 135° at half-standard-rate would take 90 sec. (135° ÷ 1.5°/sec.) or 1 min. 30 sec.

[85]

If a half-standard-rate turn is maintained, how long would it take to turn 360°?
  1. 1 minute.
  2. 4 minutes.
  3. 2 minutes.
  • Answer (A) is incorrect because a half-standard-rate turn would take 1 min. to turn 90°, not 360°.
  • Answer (B) is correct. A standard-rate turn (3°/sec) takes 2 min. for 360°. A half-standard-rate turn (1.5°/sec.) would thus take 4 min. for 360°.
  • Answer (C) is incorrect because a standard-rate, not half-standard-rate, turn completes 360° in 2 min.

[86]

If a standard-rate turn is maintained, how long would it take to turn 180°?
  1. 2 minutes.
  2. 3 minutes.
  3. 1 minute.
  • Answer (A) is incorrect because an airplane would turn 180° in 2 min. at a half-standard-rate, not standard-rate, turn.
  • Answer (B) is incorrect because an airplane would turn 540° in 3 min. at a standard-rate (3°/sec.)
  • Answer (C) is correct. A standard-rate turn means an airplane is turning at a rate of 3°/sec. To turn 180° at a standard-rate would take 60 sec. (180° ÷ 3°/sec.), or 1 min.

[87]

If a half-standard-rate turn is maintained, how much time would be required to turn clockwise from a heading of 090° to a heading of 180°?
  1. 1 minute.
  2. 1 minute 30 seconds.
  3. 30 seconds.
  • Answer (A) is correct. A half-standard-rate turn means an airplane is turning at a rate of 1.5°/sec. A turn clockwise from 090° to 180° is a total of 90°. Thus, at a half-standard-rate, it would take 60 sec. (90° ÷ 1.5°/sec.).
  • Answer (B) is incorrect because an airplane would turn 135°, not 90°, in 1 min. 30 sec. at a half-standard-rate of turn.
  • Answer (C) is incorrect because it would take 30 sec. to turn 90° at a standard, not half-standard, rate of turn.

[88]

During a constant-bank level turn, what effect would an increase in airspeed have on the rate and radius of turn?
  1. Rate of turn would increase, and radius of turn would increase.
  2. Rate of turn would decrease, and radius of turn would increase.
  3. Rate of turn would decrease, and radius of turn would decrease.
  • Answer (A) is incorrect because the rate of turn decreases, not increases.
  • Answer (B) is correct. The radius of turn at a constant-bank level turn varies directly with the airspeed, while the rate of turn at a constant-bank level turn also varies with airspeed. If airspeed is increased during a constant-bank level turn, the radius of turn will increase, and rate of turn will decrease.
  • Answer (C) is incorrect because the radius of the turn increases, not decreases.

[89]

Rate of turn can be increased and radius of turn decreased by
  1. increasing airspeed and increasing the bank.
  2. decreasing airspeed and increasing the bank.
  3. decreasing airspeed and shallowing the bank.
  • Answer (A) is incorrect because the airspeed should be decreased, not increased, to increase the rate of turn.
  • Answer (B) is correct. To increase the rate and decrease the radius of turn, you should decrease airspeed and increase the bank angle.
  • Answer (C) is incorrect because decreasing (shallowing) the bank decreases, not increases, the rate of turn.

[90]

If a standard-rate turn is maintained, how much time would be required to turn to the right from a heading of 090° to a heading of 270°?
  1. 3 minutes.
  2. 1 minute.
  3. 2 minutes.
  • Answer (A) is incorrect because it would take 3 min. to turn 270°, not 180°, at a half-standard, not standard, rate of turn.
  • Answer (B) is correct. A standard-rate turn means an airplane is turning at the rate of 3°/sec. A turn to the right (or left) from 090° to 270° is a total of 180°. Thus, at standard- rate, it would take 60 sec. (180° ÷ 3°/sec.), or 1 min.
  • Answer (C) is incorrect because it would take 2 min. to turn 180° at a half-standard, not standard, rate of turn.

[91]

Conditions that determine pitch attitude required to maintain level flight are
  1. airspeed, air density, wing design, and angle of attack.
  2. flightpath, wind velocity, and angle of attack.
  3. relative wind, pressure altitude, and vertical lift component.
  • Answer (A) is correct. Conditions that determine the pitch attitude required to maintain level flight are airspeed, air density, wing design, and angle of attack. At a constant angle of attack, any change in airspeed will vary the lift. Lift varies directly with changes in air density. An airplane's wing has lift characteristics that are suited to its intended uses. Lift increases with any increase in the angle of attack (up to the critical angle).
  • Answer (B) is incorrect because flight path is the direction of travel of the airplane, which in this case is level flight. Wind velocity is not considered in maintaining level flight. Angle of attack is the resultant pitch attitude to maintain level flight.
  • Answer (C) is incorrect because relative wind is the direction of airflow produced by an airplane in flight, and the vertical lift component is an aerodynamic force that acts perpendicular to the relative wind. The density, not pressure, altitude is one condition which determines the pitch attitude required to maintain level flight.

[92]

Approximately what percent of the indicated vertical speed should be used to determine the number of feet to lead the level-off from a climb to a specific altitude?
  1. 10 percent.
  2. 25 percent.
  3. 20 percent.
  • Answer (A) is correct. To level off from a climb and maintain a specific altitude, you should start the level-off before reaching the desired altitude. If your airplane is climbing at 500 fpm, it will continue to climb at a decreasing rate throughout the transition to level flight. An effective practice is to lead the altitude by 10% of the indicated vertical speed (i.e., at 500 fpm, use a 50-ft. lead).
  • Answer (B) is incorrect because you should begin to level off from a climb at approximately 10%, not 25%, of the indicated vertical speed.
  • Answer (C) is incorrect because you should begin to level off from a climb at approximately 10%, not 20%, of the indicated vertical speed.

[93]

To level off from a descent to a specific altitude, the pilot should lead the level-off by approximately
  1. 30 percent of the vertical speed.
  2. 10 percent of the vertical speed.
  3. 50 percent of the vertical speed.
  • Answer (A) is incorrect because you should begin to level off from a descent at approximately 10%, not 30%, of the indicated vertical speed.
  • Answer (B) is correct. To level off from a descent to a specific altitude, you should start the level-off before reaching the desired altitude. If your airplane is descending at 500 fpm, it will continue to descend at a decreasing rate throughout the transition to level flight. An effective practice is to lead the desired altitude by 10% of the indicated vertical speed (i.e., at 500 fpm, use a 50-ft. lead).
  • Answer (C) is incorrect because you should begin to level off from a descent at approximately 10%, not 50%, of the indicated vertical speed.

[94]

As a rule of thumb, altitude corrections of less than 100 feet should be corrected by using
  1. less than a half bar width on the attitude indicator.
  2. less than a full bar width on the attitude indicator.
  3. two bar widths on the attitude indicator.
  • Answer (A) is incorrect because altitude corrections of less than 100 ft. should be corrected by using a half-bar-width correction on the attitude indicator, not less than a half-bar-width.
  • Answer (B) is correct. As a general rule, altitude corrections of less than 100 ft. should be corrected by using a half-bar-width (i.e., less than a full-bar-width) correction on the attitude indicator.
  • Answer (C) is incorrect because altitude corrections of less than 100 ft. should be corrected by using a half-bar-, not a two-bar-, width correction on the attitude indicator.

[95]

Which instruments should be used to make a pitch correction when you have deviated from your assigned altitude?
  1. Attitude indicator, altimeter, and VSI.
  2. Altimeter and VSI.
  3. Manifold pressure gauge and VSI.
  • Answer (A) is correct. The pitch instruments are the attitude indicator, the altimeter, the vertical speed indicator, and the airspeed indicator. The attitude indicator gives you a direct indication of changes in pitch attitude when correcting for altitude variations. The rate and direction of the altimeter and vertical speed indicator confirm the correct pitch adjustment was made, and the altimeter is used to determine when you have reached your assigned altitude.
  • Answer (B) is incorrect because the question implies that you have all instruments available. Without an attitude indicator, you would use the altimeter and vertical speed indicator to make pitch corrections.
  • Answer (C) is incorrect because the manifold pressure gauge is a power, not pitch, instrument.

[96]

As a rule of thumb, altitude corrections of less than 100 feet should be corrected by using a
  1. half bar width on the attitude indicator.
  2. two bar width on the attitude indicator.
  3. full bar width on the attitude indicator.
  • Answer (A) is correct. As a general rule, altitude corrections of less than 100 ft. should be corrected by using a half-bar-width correction on the attitude indicator.
  • Answer (B) is incorrect because altitude corrections of less than 100 ft. should be corrected by using a half-bar-, not a two-bar-, width correction on the attitude indicator.
  • Answer (C) is incorrect because as a general rule, altitude corrections in excess of, not less than, 100 ft. should be corrected by an initial full-bar-width correction on the attitude indicator.

[97]

To enter a constant-airspeed descent from level-cruising flight, and maintain cruising airspeed, the pilot should
  1. first reduce power, then adjust the pitch using the attitude indicator as a reference to establish a specific rate on the VSI.
  2. first adjust the pitch attitude to a descent using the attitude indicator as a reference, then adjust the power to maintain the cruising airspeed.
  3. simultaneously reduce power and adjust the pitch using the attitude indicator as a reference to maintain the cruising airspeed.
  • Answer (A) is incorrect because airspeed will decrease if you first reduce power. You use the airspeed, not vertical speed, indicator to maintain a constant airspeed.
  • Answer (B) is incorrect because airspeed will increase if you adjust the pitch attitude first.
  • Answer (C) is correct. To enter a constant-airspeed descent from level cruising flight and maintain cruising airspeed, you should simultaneously reduce the power smoothly to the desired setting and reduce the pitch attitude slightly by using the attitude indicator as a reference to maintain the cruising airspeed.

[98]

To level off at an airspeed higher than the descent speed, the addition of power should be made, assuming a 500 FPM rate of descent, at approximately
  1. 50 to 100 feet above the desired altitude.
  2. 100 to 150 feet above the desired altitude.
  3. 150 to 200 feet above the desired altitude.
  • Answer (A) is incorrect because, to level off at descent airspeed, not a higher airspeed, lead the desired altitude by approximately 50 to 100 ft.
  • Answer (B) is correct. To level off from a descent at an airspeed higher than the descent speed, it is necessary to start the level-off before reaching the desired altitude. At 500 fpm, an effective practice is to lead the desired altitude by approximately 100 to 150 ft. above the desired altitude. At this point, add power to the appropriate level flight cruise setting.
  • Answer (C) is incorrect because 150 to 200 ft. above the desired altitude is not a lead point when descending at 500 fpm.

[99]

To level off from a descent maintaining the descending airspeed, the pilot should lead the desired altitude by approximately
  1. 60 feet.
  2. 20 feet.
  3. 50 feet.
  • Answer (A) is incorrect because you should lead the desired altitude by approximately 50 ft., not 60 ft., when leveling off from a descent at descent airspeed.
  • Answer (B) is incorrect because you should lead the desired altitude by approximately 50 ft., not 20 ft., when leveling off from a descent at descent airspeed.
  • Answer (C) is correct. To level off from a descent at descent airspeed, lead the desired altitude by approximately 50 ft., simultaneously adjusting the pitch attitude to level flight and adding power to a setting that will hold airspeed constant. Trim off the control pressures and continue with the normal straight-and-level flight cross-check.

[100]

While cruising at 160 knots, you wish to establish a climb at 130 knots. When entering the climb (full panel), it is proper to make the initial pitch change by increasing back elevator pressure until the
  1. attitude indicator, airspeed, and vertical speed indicate a climb.
  2. vertical speed indication reaches the predetermined rate of climb.
  3. attitude indicator shows the approximate pitch attitude appropriate for the 130-knot climb.
  • Answer (A) is incorrect because, for the predetermined climb speed, the adjustment should be to the climb attitude, not just a climb indication on the instruments.
  • Answer (B) is incorrect because the airspeed is predetermined, i.e., constant climb speed, not constant climb rate.
  • Answer (C) is correct. To enter a constant-airspeed climb from cruising air-speed, raise the miniature aircraft in the attitude indicator to the approximate nose-high indication appropriate to the predetermined climb speed. The attitude will vary according to the type of airplane you are flying. Apply light elevator back pressure to initiate and maintain the climb attitude. The amount of back pressure will increase as the airplane decelerates.

[101]

While cruising at 190 knots, you wish to establish a climb at 160 knots. When entering the climb (full panel), it would be proper to make the initial pitch change by increasing back elevator pressure until the
  1. airspeed indication reaches 160 knots.
  2. attitude indicator shows the approximate pitch attitude appropriate for the 160-knot climb.
  3. attitude indicator, airspeed, and vertical speed indicate a climb.
  • Answer (A) is incorrect because you make an initial pitch adjustment, not an increasing adjustment; i.e., airspeed will decrease gradually.
  • Answer (B) is correct. To enter a constant-airspeed climb from cruising air speed, raise the miniature aircraft in the attitude indicator to the approximate nose-high indication appropriate to the predetermined climb speed. The attitude will vary according to the type of airplane you are flying. Apply light elevator back pressure to initiate and maintain the climb attitude. The required back pressure will increase as the airplane decelerates.
  • Answer (C) is incorrect because, for the predetermined climb speed, you make the adjustment to the climb attitude, not just a climb indication on the instruments.

[102]

What is the first fundamental skill in attitude instrument flying?
  1. Instrument interpretation.
  2. Aircraft control.
  3. Instrument cross-check.
  • Answer (A) is incorrect because the second, not first, fundamental skill in attitude instrument flying is instrument interpretation. For each maneuver, you must know the performance to expect and the combination of instruments that you must interpret in order to control airplane attitude during the maneuver.
  • Answer (B) is incorrect because the third, not first, fundamental skill in attitude instrument flying is aircraft control. Aircraft control is composed of three components: pitch, bank, and power control.
  • Answer (C) is correct. The first fundamental skill in attitude instrument flying is instrument cross-check. Cross-checking is the continuous and logical observation of instruments for attitude and performance information.

[103]

What are the three fundamental skills involved in attitude instrument flying?
  1. Instrument interpretation, trim application, and aircraft control.
  2. Cross-check, emphasis, and aircraft control.
  3. Cross-check, instrument interpretation, and aircraft control.
  • Answer (A) is incorrect because trim application is only one aspect of aircraft control.
  • Answer (B) is incorrect because emphasis (along with fixation and omission) are common errors in instrument cross-checking.
  • Answer (C) is correct. The three fundamental skills involved in all instrument flight maneuvers are instrument cross-check, instrument interpretation, and aircraft control. Cross-checking is the continuous and logical observation of the instruments for attitude and performance information. Instrument interpretation requires you to understand each instrument's construction, operating principle, and relationship to the performance of your airplane. Aircraft control requires you to substitute instruments for outside references.

[104]

What is the third fundamental skill in attitude instrument flying?
  1. Power control.
  2. Aircraft control.
  3. Instrument cross-check.
  • Answer (A) is incorrect because power control is only one component of aircraft control.
  • Answer (B) is correct. The third fundamental skill in instrument flying is aircraft control. It consists of pitch, bank, and power control.
  • Answer (C) is incorrect because instrument cross-check is the first, not third, fundamental skill in attitude instrument flying. Cross-checking is the continuous and logical observation of instruments for attitude and per formance information.

[105]

What is the correct sequence in which to use the three skills used in instrument flying?
  1. Instrument interpretation, cross-check, and aircraft control.
  2. Aircraft control, cross-check, and instrument interpretation.
  3. Cross-check, instrument interpretation, and aircraft control.
  • Answer (A) is incorrect because instrument interpretation is the second, not first, skill and cross-check is the first, not second, skill used in instrument flying.
  • Answer (B) is incorrect because aircraft control is the third, not first, skill used in instrument flying.
  • Answer (C) is correct. The correct sequence in which to use the three fundamental skills of instrument flying is cross-check, instrument interpretation, and aircraft control. Although you learn these skills separately and in deliberate sequence, a measure of your proficiency in precision flying will be your ability to integrate these skills into unified, smooth, positive control responses to maintain any desired flight path.

[106]

As power is reduced to change airspeed from high to low cruise in level flight, which instruments are primary for pitch, bank, and power, respectively?
  1. Altimeter, heading indicator, and manifold pressure gauge or tachometer.
  2. Attitude indicator, heading indicator, and manifold pressure gauge or tachometer.
  3. Altimeter, attitude indicator, and airspeed indicator.
  • Answer (A) is correct. In straight-and-level flight, when reducing airspeed from high to low cruise, the primary instrument for pitch is the altimeter; the primary instrument for bank is the heading indicator; and the primary instrument for power is the manifold pressure gauge or tachometer.
  • Answer (B) is incorrect because the primary pitch instrument is the altimeter, not attitude indicator.
  • Answer (C) is incorrect because the primary bank instru ment is the heading indicator, not attitude indicator; and the primary power instrument is the manifold pressure gauge or tachometer, not airspeed indicator.

[107]

What instruments are considered supporting bank instruments during a straight, stabilized climb at a constant rate?
  1. Attitude indicator and turn coordinator.
  2. Heading indicator and turn coordinator.
  3. Heading indicator and attitude indicator.
  • Answer (A) is correct. During a straight, stabilized climb at a constant rate, the heading indicator is the primary instrument for bank. The supporting bank instruments are the turn coordinator and the attitude indicator.
  • Answer (B) is incorrect because the heading indicator is the primary, not supporting, bank instrument in a straight climb.
  • Answer (C) is incorrect because the heading indicator is the primary, not supporting, bank instrument in a straight climb.

[108]

Which instruments are considered primary and supporting for bank, respectively, when establishing a level standard-rate turn?
  1. Turn coordinator and attitude indicator.
  2. Attitude indicator and turn coordinator.
  3. Turn coordinator and heading indicator.
  • Answer (A) is incorrect because the turn coordinator is the primary bank instrument and the attitude indicator is the supporting bank instrument only after the standard-rate turn is established, not while entering the turn.
  • Answer (B) is correct. When you are establishing a level standard-rate turn, the attitude indicator is the primary bank instrument and is used to establish the approximate angle of bank. The turn coordinator is the supporting bank instrument as you check for the standard-rate turn indication.
  • Answer (C) is incorrect because the turn coordinator is the supporting, not primary, bank instrument and the heading indicator is neither the primary nor the supporting bank instrument when establishing a standard-rate turn.

[109]

Which instrument provides the most pertinent information (primary) for bank control in straight-and-level flight?
  1. Turn-and-slip indicator.
  2. Attitude indicator.
  3. Heading indicator.
  • Answer (A) is incorrect because the turn-and-slip indicator is a supporting, not primary, bank instrument in straight-and-level flight.
  • Answer (B) is incorrect because the attitude indicator is a supporting, not primary, bank and pitch instrument in straight-and-level flight.
  • Answer (C) is correct. The primary instrument for bank control in straight-and-level flight is the heading indicator.

[110]

Which instruments, in addition to the attitude indicator, are pitch instruments?
  1. Altimeter and airspeed only.
  2. Altimeter, airspeed indicator, and vertical speed indicator.
  3. Altimeter and VSI only.
  • Answer (A) is incorrect because it omits the vertical speed indicator and the airspeed indicator.
  • Answer (B) is correct. The pitch control instruments are the attitude indicator, altimeter, vertical speed indicator, and airspeed indicator.
  • Answer (C) is incorrect because it omits the airspeed indicator.

[111]

Which instrument provides the most pertinent information (primary) for pitch control in straight-and-level flight?
  1. Altimeter.
  2. Attitude indicator.
  3. Airspeed indicator.
  • Answer (A) is correct. The primary pitch instrument for straight-and-level flight is the altimeter.
  • Answer (B) is incorrect because the attitude indicator is a supporting, not primary, pitch instrument in straight-and-level flight.
  • Answer (C) is incorrect because the airspeed indicator is the primary power, not pitch, control instrument in straight-and-level flight.

[112]

For maintaining level flight at constant thrust, which instrument would be the least appropriate for determining the need for a pitch change?
  1. Altimeter.
  2. Attitude indicator.
  3. VSI.
  • Answer (A) is incorrect because, since level flight means a constant altitude, the altimeter is the primary pitch instrument in level flight.
  • Answer (B) is correct. To maintain level flight at constant thrust, the attitude indicator is the least appropriate for determining the need for pitch change. Until you have established and identified the level flight attitude for that airspeed, you have no way of knowing whether level flight as indicated on the attitude indicator is resulting in level flight as shown on the altimeter, vertical speed indicator, and airspeed indicator.
  • Answer (C) is incorrect because the vertical speed indicator (as a trend instrument) shows immediately the initial vertical movement of the airplane, which, disregarding turbulence, can be a reflection of pitch change at a constant thrust.

[113]

The gyroscopic heading indicator is inoperative. What is the primary bank instrument in unaccelerated straight-and-level flight?
  1. Magnetic compass.
  2. Attitude indicator.
  3. Miniature aircraft of turn coordinator.
  • Answer (A) is correct. With the gyroscopic heading indicator inoperative, the primary bank instrument in unaccelerated straight-and-level flight is the magnetic compass. Since any banking results in a turn and change in heading, the magnetic compass is the only other instrument that indicates direction.
  • Answer (B) is incorrect because, although the attitude indicator shows any change in bank, it does not provide information (i.e., heading) needed to maintain straight flight.
  • Answer (C) is incorrect because the miniature aircraft of the turn coordinator is the primary bank instrument in established standard-rate turns, not in straight flight.

[114]

What instruments are primary for pitch, bank, and power, respectively, when transitioning into a constant airspeed climb from straight-and-level flight?
  1. Attitude indicator for both pitch and bank; airspeed indicator for power.
  2. Vertical speed, attitude indicator, and manifold pressure or tachometer.
  3. Attitude indicator, heading indicator, and manifold pressure gauge or tachometer.
  • Answer (A) is incorrect because the heading indicator, not attitude indicator, is primary for bank, and the manifold pressure gauge, not airspeed indicator, is primary for power.
  • Answer (B) is incorrect because the attitude indicator, not vertical speed indicator, is the primary instrument for pitch, and the heading indicator, not attitude indicator, is primary for bank.
  • Answer (C) is correct. When you are entering a constant airspeed climb, the attitude indicator is the primary pitch instrument, the heading indicator is the primary bank instrument, and the tachometer or manifold pressure gauge is the primary power instrument.

[115]

What is the primary bank instrument once a standard-rate turn is established?
  1. Turn coordinator.
  2. Heading indicator.
  3. Attitude indicator.
  • Answer (A) is correct. After a standard-rate turn is established, the turn coordinator is the primary bank instrument.
  • Answer (B) is incorrect because the heading indicator is the primary bank instrument for straight flight.
  • Answer (C) is incorrect because the attitude indicator is the primary bank instrument in establishing a standard-rate turn but not for maintaining the turn once established.

[116]

During standard-rate turns, which instrument is considered "primary" for bank?
  1. Heading indicator.
  2. Attitude indicator.
  3. Turn and slip indicator or turn coordinator.
  • Answer (A) is incorrect because the heading indicator is the primary bank instrument for straight flight.
  • Answer (B) is incorrect because the attitude indicator is the primary bank instrument in establishing a standard-rate turn but not for maintaining the turn once established.
  • Answer (C) is correct. After a standard-rate turn is established, the turn coordinator or turn and slip indicator is the primary bank instrument.

[117]

What is the primary pitch instrument when establishing a constant altitude standard-rate turn?
  1. Altimeter.
  2. Airspeed indicator.
  3. VSI.
  • Answer (A) is correct. The primary pitch instrument in level flight, either straight or turns, is the altimeter.
  • Answer (B) is incorrect because the airspeed indicator is the primary power, not pitch, instrument when establishing a constant altitude standard-rate turn.
  • Answer (C) is incorrect because the vertical speed indicator is a supporting, not primary, pitch instrument for establishing a level standard-rate turn.

[118]

What is the initial primary bank instrument when establishing a level standard-rate turn?
  1. Turn coordinator.
  2. Attitude indicator.
  3. Heading indicator.
  • Answer (A) is incorrect because only after the turn is established does the turn coordinator become the primary bank instrument.
  • Answer (B) is correct. The initial primary bank instrument when establishing a level standard-rate turn is the attitude indicator.
  • Answer (C) is incorrect because the heading indicator is the primary bank instrument for straight flight.

[119]

What is the primary bank instrument while transitioning from straight-and-level flight to a standard-rate turn to the left?
  1. Attitude indicator.
  2. Turn coordinator (miniature aircraft).
  3. Heading indicator.
  • Answer (A) is correct. The initial primary bank instrument when establishing a level standard-rate of turn is the attitude indicator.
  • Answer (B) is incorrect because only after the turn is established does the turn coordinator become the primary bank instrument.
  • Answer (C) is incorrect because the heading indicator is the primary bank instrument for straight flight.

[120]

What instrument(s) is(are) supporting bank instrument when entering a constant airspeed climb from straight-and-level flight?
  1. Attitude indicator and turn coordinator.
  2. Turn coordinator and heading indicator.
  3. Heading indicator.
  • Answer (A) is correct. When entering a constant-airspeed climb from straight-and-level flight, the primary bank instrument is the heading indicator. Supporting bank instruments are the turn coordinator and the attitude indicator.
  • Answer (B) is incorrect because the heading indicator is the primary, not supporting, bank instrument for straight flight.
  • Answer (C) is incorrect because the heading indicator is the primary, not supporting, bank instrument for straight flight.

[121]

What is the primary pitch instrument during a stabilized climbing left turn at cruise climb airspeed?
  1. Attitude indicator.
  2. Airspeed indicator.
  3. VSI.
  • Answer (A) is incorrect because the attitude indicator is a supporting, not primary, pitch instrument in a stabilized climb.
  • Answer (B) is correct. In a climbing left turn at a constant airspeed, the airspeed indicator is the primary instrument for pitch once the climb is established.
  • Answer (C) is incorrect because the vertical speed indicator is a supporting, not primary, pitch instrument in a stabilized climb.

[122]

Which instruments are considered to be supporting instruments for pitch during change of airspeed in a level turn?
  1. Altimeter and attitude indicator.
  2. Airspeed indicator and VSI.
  3. Attitude indicator and VSI.
  • Answer (A) is incorrect because the altimeter is the primary, not supporting, pitch instrument in level flight.
  • Answer (B) is incorrect because the airspeed indicator is a supporting power, not pitch, instrument during a change of airspeed in a level turn. It becomes the primary power instrument as the desired airspeed is reached.
  • Answer (C) is correct. The supporting instruments for pitch during a change of airspeed in a level turn are the attitude indicator and the vertical speed indicator. The primary instrument is the altimeter.

[123]

Which instrument is considered primary for power as the airspeed reaches the desired value during change of airspeed in a level turn?
  1. Airspeed indicator.
  2. Altimeter.
  3. Attitude indicator.
  • Answer (A) is correct. The airspeed indicator is the primary power instrument as the airspeed reaches the desired value during a change of airspeed in a level turn.
  • Answer (B) is incorrect because the altimeter is the primary pitch instrument.
  • Answer (C) is incorrect because the attitude indicator is a supporting pitch and bank instrument.

[124]

Which instrument indicates the quality of a turn?
  1. Heading indicator or magnetic compass.
  2. Attitude indicator.
  3. Ball of the turn coordinator.
  • Answer (A) is incorrect because the heading indicator and/or magnetic compass show current direction and changes in direction, not quality of a turn.
  • Answer (B) is incorrect because the attitude indicator provides both pitch and bank information.
  • Answer (C) is correct. The quality (coordination) of a turn relates to whether the horizontal component of lift balances the centrifugal force. It is indicated by the ball of the turn coordinator or the ball in a turn-and-slip indicator. The airplane is neither slipping nor skidding when the ball is centered, indicating the desired quality of a turn.

[125]

As power is increased to enter a 500 feet per minute rate of climb in straight flight, which instruments are primary for pitch, bank, and power respectively?
  1. Attitude indicator, heading indicator, and manifold pressure gauge or tachometer.
  2. VSI, attitude indicator, and airspeed indicator.
  3. Airspeed indicator, attitude indicator, and manifold pressure gauge or tachometer.
  • Answer (A) is correct. As the power is increased to enter a constant-rate climb in straight flight, the primary pitch instrument is the attitude indicator until the vertical speed indicator stabilizes at the desired rate of climb (then the vertical speed indicator becomes primary). The primary bank instrument is the heading indicator. The primary power instrument is the manifold pressure gauge or tachometer.
  • Answer (B) is incorrect because the vertical speed indicator is the primary pitch instrument once the constant-rate climb is established. Also, the manifold pressure gauge, not the airspeed indicator, is primary for power.
  • Answer (C) is incorrect because the heading indicator, not the attitude indicator, is primary for bank in straight flight.

[126]

(Refer to figure 145.) What is the correct sequence for recovery from the unusual attitude indicated?
  1. Level the wings, raise the nose of the aircraft to level flight attitude, and obtain desired airspeed.
  2. Reduce power, level the wings, bring pitch attitude to level flight.
  3. Reduce power, increase back elevator pressure, and level the wings.
When an aircraft is decelerated some attitude indicators will precess and incorrectly indicate a group of answer choices?

Figure 145
  • Answer (A) is incorrect because the power should be reduced first.
  • Answer (B) is correct. In Fig. 145, a nose-low attitude is indicated by a negative vertical speed indicator, high airspeed (i.e., near VNE), and the airplane below the horizon on the attitude indicator. For nose-low unusual attitudes, the correct sequence for recovery is to reduce power to prevent excessive airspeed and loss of altitude; level the wings with coordinated aileron and rudder pressure to straight flight by referring to the turn coordinator; and raise the nose to level flight attitude by smooth back elevator pressure.
  • Answer (C) is incorrect because the wings should be level before you increase back pressure to decrease the load factor during leveling off.

[127]

While recovering from an unusual flight attitude without the aid of the attitude indicator, approximate level pitch attitude is reached when the
  1. airspeed and altimeter stop their movement and the VSI reverses its trend.
  2. airspeed arrives at cruising speed, the altimeter reverses its trend, and the vertical speed stops its movement.
  3. altimeter and vertical speed reverse their trend and the airspeed stops its movement.
  • Answer (A) is correct. As the rate of movement of the altimeter and airspeed indicator needles decreases, the attitude is approaching level flight. When the needles stop and reverse direction, the aircraft is passing through level flight.
  • Answer (B) is incorrect because the vertical speed indicator will be lagging, i.e., showing a decrease in vertical movement when vertical movement has stopped.
  • Answer (C) is incorrect because the rate is only slowing and has not stabilized when the altimeter reverses its trend; i.e., it must stop to indicate level flight.

[128]

(Refer to figure 147.) Which is the correct sequence for recovery from the unusual attitude indicated?
  1. Add power, lower nose, level wings, return to original attitude and heading.
  2. Level wings, add power, lower nose, descend to original attitude, and heading.
  3. Stop turn by raising right wing and add power at the same time, lower the nose, and return to original attitude and heading.
When an aircraft is decelerated some attitude indicators will precess and incorrectly indicate a group of answer choices?

Figure 147
  • Answer (A) is correct. In Fig. 147, a nose-high attitude is indicated by the increasing altitude, the rate-of-climb indication on the vertical speed indicator, and the decreasing airspeed. The correct sequence for recovery is to add power, apply forward elevator pressure to lower the nose and prevent a stall, level the wings with coordinated aileron and rudder pressure to straight flight, and return to original altitude and heading.
  • Answer (B) is incorrect because you should both add power and lower the nose before you level the wings.
  • Answer (C) is incorrect because you should both add power and lower the nose before you level the wings.

[129]

If an airplane is in an unusual flight attitude and the attitude indicator has exceeded its limits, which instruments should be relied on to determine pitch attitude before starting recovery?
  1. Airspeed and altimeter.
  2. VSI and airspeed to detect approaching VSI or VMO.
  3. Turn indicator and VSI.
  • Answer (A) is correct. If the attitude indicator is inoperative, a nose-low or nose-high attitude can be determined by the airspeed and altimeter. In a nose-high attitude, airspeed is decreasing and altimeter is increasing, and vice versa for nose-low attitudes.
  • Answer (B) is incorrect because the altimeter, not the VSI, is the primary pitch instrument. Note the FAA answer selection has VSI and VMO, which should be VS1 and VMO, respectively.
  • Answer (C) is incorrect because the turn indicator indicates nothing about pitch attitude.

[130]

Which is the correct sequence for recovery from a spiraling, nose-low, increasing airspeed, unusual flight attitude?
  1. Reduce power, correct the bank attitude, and raise the nose to a level attitude.
  2. Increase pitch attitude, reduce power, and level wings.
  3. Reduce power, raise the nose to level attitude, and correct the bank attitude.
  • Answer (A) is correct. For nose-low unusual attitudes, one should reduce the power, level the wings, and then increase the pitch to raise the nose to a level attitude.
  • Answer (B) is incorrect because the power should be decreased first, then the wings leveled.
  • Answer (C) is incorrect because the wings should be leveled before the nose is raised to minimize the load factor.

[131]

During recoveries from unusual attitudes, level flight is attained the instant
  1. a zero rate of climb is indicated on the VSI.
  2. the altimeter and airspeed needles stop prior to reversing their direction of movement.
  3. the horizon bar on the attitude indicator is exactly overlapped with the miniature airplane.
  • Answer (A) is incorrect because there is a lag or delay in the vertical speed indicator. It cannot be relied on for determining the instant level flight is attained.
  • Answer (B) is correct. In unusual attitudes, you can determine the attainment of level flight (not vertical movement) when the altimeter and airspeed needles stop prior to reversing their direction of movement.
  • Answer (C) is incorrect because the attitude indicator has a tendency to precess during an unusual attitude and may not be reliable.

[132]

(Refer to figure 146.) Identify the system that has failed and determine a corrective action to return the airplane to straight-and-level flight.
  1. Static/pitot system is blocked; lower the nose and level the wings to level-fight attitude by use of attitude indicator.
  2. Electrical system has failed; reduce power, roll left to level wings, and raise the nose to reduce airspeed.
  3. Vacuum system has failed; reduce power, roll left to level wings, and pitchup to reduce airspeed.
When an aircraft is decelerated some attitude indicators will precess and incorrectly indicate a group of answer choices?

Figure 146
  • Answer (A) is correct. In Fig. 146, the airplane is in a right turn as indicated by the attitude indicator, the heading indicator, and the turn coordinator; thus the vacuum and electrical instruments are consistent with each other. Since the attitude indicator indicates a climb, which is consistent with the altimeter and the VSI, the airspeed should not be increasing. Thus, the pitot tube ram air and drain holes are blocked, causing the airspeed indicator to react like an altimeter. To return the airplane to straight-and-level flight, you should lower the nose and level the wings to level-flight attitude by use of the attitude indicator.
  • Answer (B) is incorrect because the turn coordinator, which is normally electric, is consistent with the attitude indicator, which is normally a vacuum system instrument.
  • Answer (C) is incorrect because the attitude indicator and heading indicator are consistent with the turn coordinator.

[133]

(Refer to figure 148.) What is the flight attitude? One system which transmits information to the instruments has malfunctioned.
  1. Climbing turn to left.
  2. Level turn to left.
  3. Climbing turn to right.
When an aircraft is decelerated some attitude indicators will precess and incorrectly indicate a group of answer choices?

Figure 148
  • Answer (A) is incorrect because the turn is to the right, not the left.
  • Answer (B) is incorrect because the attitude indicator, altimeter, and vertical speed indicator all indicate a climb, and the turn is to the right, not left.
  • Answer (C) is correct. Fig. 148 illustrates a climbing turn to the right. Note that the attitude indicator shows a climbing turn to the right, the heading indicator shows a turn to the right, and both the altimeter and vertical speed indicator indicate a climb. The turn coordinator shows no turn and is malfunctioning.

[134]

(Refer to figure 149.) What is the flight attitude? One system which transmits information to the instruments has malfunctioned.
  1. Straight-and-level flight.
  2. Level turn to the right.
  3. Level turn to the left.
When an aircraft is decelerated some attitude indicators will precess and incorrectly indicate a group of answer choices?

Figure 149
  • Answer (A) is correct. In Fig. 149, the vertical speed indicator, altimeter, and turn coordinator all indicate straight-and-level flight. The heading indicator indicates a turn to the south from west, which is a turn to the left. The attitude indicator indicates a turn to the right; i.e., the attitude indicator and heading indicator are in conflict. Thus, the vacuum system must be malfunctioning, and the airplane must be in straight-and-level flight.
  • Answer (B) is incorrect because the vacuum system (i.e., the attitude and heading indicators) is inoperative, and the airplane is in straight flight, not a right turn.
  • Answer (C) is incorrect because the vacuum system (i.e., the attitude and heading indicators) is inoperative, and the airplane is in straight flight, not a left turn.

[135]

(Refer to figure 150.) What is the flight attitude? One instrument has malfunctioned.
  1. Climbing turn to the right.
  2. Climbing turn to the left.
  3. Descending turn to the right.
When an aircraft is decelerated some attitude indicators will precess and incorrectly indicate a group of answer choices?

Figure 150
  • Answer (A) is correct. In Fig. 150, the airplane is in a climb as evidenced by the vertical speed indicator, altimeter, and airspeed indicator. The heading indicator indicates a turn from west to north, which is a turn to the right. The turn coordinator also indicates a turn to the right. Thus, the airplane is in a climbing turn to the right. The attitude indicator is the instrument that is malfunctioning since it indicates a level turn to the left.
  • Answer (B) is incorrect because the attitude indicator is inoperative; thus, the airplane is turning to the right, not left.
  • Answer (C) is incorrect because the airspeed indicator, altimeter, and vertical speed indicator all show that the airplane is climbing, not descending.

[136]

(Refer to figure 151.) What is the flight attitude? One instrument has malfunctioned.
  1. Level turn to the right.
  2. Climbing turn to the right.
  3. Level turn to the left.
When an aircraft is decelerated some attitude indicators will precess and incorrectly indicate a group of answer choices?

Figure 151
  • Answer (A) is correct. The vertical speed indicator, altimeter, and attitude indicator indicate level flight. The turn coordinator, attitude indicator, and heading indicator indicate a turn to the right. Accordingly, there is a level turn to the right, and the airspeed should not be near the stall speed. Thus, the ram air inlet and the drain hole of the pitot tube are clogged. The airspeed indicator will react the same way as an altimeter, if the static port is open, by showing a decrease in airspeed as altitude decreases and an increase in speed as altitude increases.
  • Answer (B) is incorrect because flight is level, not climbing, according to the vertical speed indicator, altimeter, and attitude indicator.
  • Answer (C) is incorrect because the turn is to the right, not left.

[137]

If you fly into severe turbulence, which flight condition should you attempt to maintain?
  1. Constant airspeed (VA).
  2. Constant altitude and constant airspeed.
  3. Level flight attitude.
  • Answer (A) is incorrect because you want to maintain an airspeed at or below VA, but in severe turbulence, there will be large variations in airspeed, and you will not be able to keep it constant.
  • Answer (B) is incorrect because, in severe turbulence, you will not be able to maintain a constant altitude and/or constant airspeed.
  • Answer (C) is correct. In severe turbulence, you should attempt to maintain a level flight attitude. You will not be able to maintain a constant altitude and/or airspeed, but you should fly at or below design maneuvering speed (VA) and attempt to maintain a level flight attitude.

[138]

If severe turbulence is encountered during your IFR flight, the airplane should be slowed to the design maneuvering speed because the
  1. amount of excess load that can be imposed on the wing will be decreased.
  2. maneuverability of the airplane will be increased.
  3. airplane will stall at a lower angle of attack, giving an increased margin of safety.
  • Answer (A) is correct. Flight at or below the design maneuvering speed (VA) means that the airplane will stall before excess loads can be imposed on the wings.
  • Answer (B) is incorrect because you should slow the airspeed to reduce excessive loads, not because the airplane will be more maneuverable at a slow airspeed.
  • Answer (C) is incorrect because an airplane will always stall when the critical angle of attack is exceeded.

[139]

When a climb or descent through an inversion or wind-shear zone is being performed, the pilot should be alert for which of the following change in airplane performance?
  1. A fast rate of climb and a slow rate of descent.
  2. A sudden surge of thrust.
  3. A sudden change in airspeed.
  • Answer (A) is incorrect because a fast rate of climb and a slow rate of descent are usually not a safety problem, as the reverse could be.
  • Answer (B) is incorrect because the amount of thrust does not change as a result of wind shears.
  • Answer (C) is correct. When climbing through an inversion or wind-shear zone, the danger is a sudden change in airspeed. If the airplane were to move abruptly from a headwind to a tailwind, the airspeed would slow dramatically, and a stall or rapid descent could be induced.

[140]

Which distance is displayed by the DME indicator?
  1. Line-of-sight direct distance from aircraft to VORTAC in SM.
  2. Slant range distance in SM.
  3. Slant range distance in NM.
  • Answer (A) is incorrect because the measurement is in nautical miles, not statute miles.
  • Answer (B) is incorrect because the measurement is in nautical miles, not statute miles.
  • Answer (C) is correct. DME (distance measuring equipment) displays line-of-sight direct distance, i.e., slant range, from the aircraft to the VORTAC in nautical miles.

[141]

As a rule of thumb, to minimize DME slant range error, how far from the facility should you be to consider the reading as accurate?
  1. One or more miles for each 1,000 feet of altitude above the facility.
  2. Two miles or more for each 1,000 feet of altitude above the facility.
  3. No specific distance is specified since the reception is line-of-sight.
  • Answer (A) is correct. You should consider the DME slant range error negligible if the airplane is 1 NM or more from the ground facility for each 1,000 ft. of altitude above the elevation of the facility.
  • Answer (B) is incorrect because the accuracy is 1 NM, not 2 NM, for each 1,000 ft. AGL.
  • Answer (C) is incorrect because a specific distance is required because the reception is line-of-sight.

[142]

As a rule of thumb, to minimize DME slant range error, how far from the facility should you be to consider the reading as accurate?
  1. One or more miles for each 1,000 feet of altitude above the facility.
  2. Two miles or more for each 1,000 feet of altitude above the facility.
  3. No specific distance is specified since the reception is line-of-sight.
  • Answer (A) is correct. You should consider the DME slant range error negligible if the airplane is 1 NM or more from the ground facility for each 1,000 ft. of altitude above the elevation of the facility.
  • Answer (B) is incorrect because the accuracy is 1 NM, not 2 NM, for each 1,000 ft. AGL.
  • Answer (C) is incorrect because a specific distance is required because the reception is line-of-sight.

[143]

Where does the DME indicator have the greatest error between ground distance to the VORTAC and displayed distance?
  1. High altitudes close to the VORTAC.
  2. Low altitudes far from the VORTAC.
  3. High altitudes far from the VORTAC.
  • Answer (A) is correct. Because the DME reads slant range distance, its greatest error occurs at high altitudes very close to the VORTAC. For example, if one were at 12,000 ft. directly over the VOR, the DME would show a distance from the VOR of approximately 2 NM.
  • Answer (B) is incorrect because the DME has the greatest error at high, not low, altitudes close to, not far from, the VORTAC.
  • Answer (C) is incorrect because, as you get farther away from the station, the slant range error of the DME becomes minimal.

[144]

Which DME indication should you receive when you are directly over a VORTAC site at approximately 6,000 feet AGL?
  • Answer (A) is incorrect because the DME would only indicate zero if you were at ground level next to the VORTAC.
  • Answer (B) is correct. Because the DME indicates slant range distance, it will indicate your altitude if you are directly above the VORTAC. One nautical mile equals approximately 6,000 ft., so the DME would read 1 NM.
  • Answer (C) is incorrect because it would mean that your altitude was about 8,000 ft. AGL (6,000 x 1.3).

[145]

By which means may a pilot determine if a Loran C equipped aircraft is approved for IFR operations?
  1. Check aircraft logbook.
  2. Check the Airplane Flight Manual Supplement.
  3. Not necessary; Loran C is not approved for IFR.
  • Answer (A) is incorrect because the operational approval level may be found in the aircraft maintenance records, not necessarily the aircraft logbook.
  • Answer (B) is correct. Pilots must be aware of the authorized operational approval level (e.g., VFR or IFR) of a LORAN receiver installed in their aircraft. Approval information is contained in the Aircraft Flight Manual Supplement, on FAA Form 337, in aircraft maintenance records, or possibly on a placard installed near or on the control panel.
  • Answer (C) is incorrect because some LORAN C receivers are approved for IFR operations by the FAA.

[146]

(Refer to figure 101.) What is the magnetic bearing TO the station?
When an aircraft is decelerated some attitude indicators will precess and incorrectly indicate a group of answer choices?

Figure 101
  • Answer (A) is incorrect because 060° would be the MB FROM, not TO, the station if the MH were 330°.
  • Answer (B) is correct. Magnetic bearing TO the station is equal to the sum of magnetic heading plus relative bearing. Magnetic heading is given on the heading indicator as 350°, and the relative bearing is given as 270°. The sum is 620°. To obtain answers between 0° and 360°, you may have to add or subtract 360°. 620° - 360° = 260° magnetic bearing TO the station.
    MH + RB = MB
    620° = MB
    620° - 360° = MB
    260° = MB
  • Answer (C) is incorrect because 270° is the relative bearing, not the magnetic bearing, TO the station.

[147]

(Refer to figure 105.) If the magnetic heading shown for aircraft 7 is maintained, which ADF illustration would indicate the aircraft is on the 120° magnetic bearing FROM the station?
When an aircraft is decelerated some attitude indicators will precess and incorrectly indicate a group of answer choices?

Figure 105
  • Answer (A) is correct. On Fig. 105, aircraft 7 has a magnetic heading of 270°. To use the standard magnetic bearing formula, you must first convert to magnetic bearing TO by adding 180° to MB FROM (120° + 180° = 300° MB TO). Also note that you may have to add or subtract 360° from the final answer to arrive at a figure between 0° and 360°.
    MH + RB=MB
    270° + RB=300°
    RB=30°

    Illustration 5 indicates a 30° relative bearing.

  • Answer (B) is incorrect because illustration 2 indicates you are on the 150°, not 120°, MB FROM the NDB.
  • Answer (C) is incorrect because illustration 4 indicates you are on the 120° MB TO, not FROM, the NDB.

[148]

(Refer to figure 105.) If the magnetic heading shown for aircraft 5 is maintained, which ADF illustration would indicate the aircraft is on the 210° magnetic bearing FROM the station?
When an aircraft is decelerated some attitude indicators will precess and incorrectly indicate a group of answer choices?

Figure 105
  • Answer (A) is incorrect because illustration 2 indicates you are on the 240°, not 210°, MB TO, not FROM, the NDB.
  • Answer (B) is correct. On Fig. 105, aircraft 5 has a magnetic heading of 180°. To determine the relative bearing given a 210° magnetic bearing FROM the station, convert to MB TO by adding or subtracting 180° (210° - 180° = 30° MB TO). Then use the standard formula.
    MH + RB = MB
    180° + RB = 30°
    RB = -150°
    RB = -150° + 360°
    RB = 210°

    Illustration 4 indicates a of 210° relative bearing.

  • Answer (C) is incorrect because illustration 3 indicates you are on the 75°, not 210°, MB TO the NDB.

[149]

(Refer to figure 105.) If the magnetic heading shown for aircraft 3 is maintained, which ADF illustration would indicate the aircraft is on the 120° magnetic bearing TO the station?
When an aircraft is decelerated some attitude indicators will precess and incorrectly indicate a group of answer choices?

Figure 105
  • Answer (A) is incorrect because illustration 4 indicates you are on the 120° MB FROM, not TO, the NDB.
  • Answer (B) is incorrect because illustration 8 indicates you are on the 225°, not 120°, MB TO the NDB.
  • Answer (C) is correct. On Fig. 105, aircraft 3 has a magnetic heading of 090°. To determine the relative bearing given a 120° magnetic bearing TO the station, use the standard magnetic bearing formula to get the bearing to the station.
    MH + RB = MB
    90° + RB = 120°
    RB = 30°

    Illustration 5 indicates a 30° relative bearing.

[150]

(Refer to figure 105.) If the magnetic heading shown for aircraft 1 is maintained, which ADF illustration would indicate the aircraft is on the 060° magnetic bearing TO the station?
When an aircraft is decelerated some attitude indicators will precess and incorrectly indicate a group of answer choices?

Figure 105
  • Answer (A) is incorrect because illustration 4 indicates you are on the 210°, not 060°, MB TO the NDB.
  • Answer (B) is incorrect because illustration 5 indicates you are on the 030°, not 060°, MB TO the NDB.
  • Answer (C) is correct. On Fig. 105, aircraft 1 has a magnetic heading of 360° or 0°. To determine the relative bearing given a 060° magnetic bearing TO the station, use the standard magnetic bearing formula to get the bearing to the station.
    MH + RB = MB
    0° + RB = 60°
    RB = 60°

    Illustration 2 indicates a 60° relative bearing.

[151]

(Refer to figure 105.) If the magnetic heading shown for aircraft 2 is maintained, which ADF illustration would indicate the aircraft is on the 255° magnetic bearing TO the station?
When an aircraft is decelerated some attitude indicators will precess and incorrectly indicate a group of answer choices?

Figure 105
  • Answer (A) is incorrect because illustration 5 indicates you are on the 255° MB FROM, not TO, the NDB.
  • Answer (B) is incorrect because illustration 2 indicates you are on the 105°, not 255°, MB TO the NDB.
  • Answer (C) is correct. On Fig. 105, aircraft 2 has a magnetic heading of 045°. To determine the relative bearing given a 255° magnetic bearing TO the station, use the standard magnetic bearing formula to get the bearing to the station.
    MH + RB = MB
    45° + RB = 255°
    RB = 210°

    Illustration 4 indicates a 210° relative bearing.

[152]

(Refer to figure 105.) If the magnetic heading shown for aircraft 4 is maintained, which ADF illustration would indicate the aircraft is on the 135° magnetic bearing TO the station?
When an aircraft is decelerated some attitude indicators will precess and incorrectly indicate a group of answer choices?

Figure 105
  • Answer (A) is correct. On Fig. 105, aircraft 4 has a magnetic heading of 135°. To determine the relative bearing given a 135° magnetic bearing TO the station, use the standard magnetic bearing formula to get the magnetic bearing to the station.
    MH + RB = MB
    135° + RB = 135°
    RB =

    Illustration 1 indicates a 0° relative bearing.

  • Answer (B) is incorrect because illustration 8 indicates you are on the 270°, not 135°, MB TO the NDB.
  • Answer (C) is incorrect because illustration 4 indicates you are on the 345°, not 135°, MB TO the NDB.

[153]

(Refer to figure 105.) If the magnetic heading shown for aircraft 6 is maintained, which ADF illustration would indicate the aircraft is on the 255° magnetic bearing FROM the station?
When an aircraft is decelerated some attitude indicators will precess and incorrectly indicate a group of answer choices?

Figure 105
  • Answer (A) is incorrect because illustration 2 indicates you are on the 105°, not 255°, MB FROM the NDB.
  • Answer (B) is incorrect because illustration 5 indicates you are on the 255° MB TO, not FROM, the NDB.
  • Answer (C) is correct. On Fig. 105, aircraft 6 has a magnetic heading of 225°. To determine the relative bearing given a 255° magnetic bearing FROM the station, convert to MB TO by subtracting 180° (255° - 180° = 75° MB TO). Then use the standard formula.
    MH + RB = MB
    225° + RB = 75°
    RB = -150° + 360°
    RB = 210°

    Illustration 4 indicates a 210° relative bearing.

[154]

(Refer to figure 105.) If the magnetic heading shown for aircraft 8 is maintained, which ADF illustration would indicate the aircraft is on the 090° magnetic bearing FROM the station?
When an aircraft is decelerated some attitude indicators will precess and incorrectly indicate a group of answer choices?

Figure 105
  • Answer (A) is incorrect because illustration 3 indicates you are on the 030°, not 090°, MB FROM the NDB.
  • Answer (B) is incorrect because illustration 4 indicates you are on the 345°, not 090°, MB FROM the NDB.
  • Answer (C) is correct. On Fig. 105, aircraft 8 has a magnetic heading of 315°. To determine the relative bearing given a 090° magnetic bearing FROM the station, convert to MB TO by adding 180° (90° + 180° = 270° MB TO). Then use the standard formula.
    MH + RB = MB
    315° + RB = 270°
    RB = -45° + 360°
    RB = 315°

    Illustration 6 indicates a 315° relative bearing.

[155]

(Refer to figure 105.) If the magnetic heading shown for aircraft 5 is maintained, which ADF illustration would indicate the aircraft is on the 240° magnetic bearing TO the station?
When an aircraft is decelerated some attitude indicators will precess and incorrectly indicate a group of answer choices?

Figure 105
  • Answer (A) is incorrect because illustration 3 indicates you are on the 075°, not 240°, MB TO the NDB.
  • Answer (B) is correct. On Fig. 105, aircraft 5 has a magnetic heading of 180°. To determine the relative bearing given a 240° magnetic bearing TO the station, use the standard magnetic bearing formula to get the relative bearing to the station.
    MH + RB = MB
    180° + RB = 240°
    RB = 60°

    Illustration 2 indicates a 60° relative bearing.

  • Answer (C) is incorrect because illustration 4 indicates you are on the 030°, not 240°, MB TO the NDB.

[156]

(Refer to figure 105.) If the magnetic heading shown for aircraft 8 is maintained, which ADF illustration would indicate the aircraft is on the 315° magnetic bearing TO the station?
When an aircraft is decelerated some attitude indicators will precess and incorrectly indicate a group of answer choices?

Figure 105
  • Answer (A) is incorrect because illustration 4 indicates you are on the 165°, not 315°, MB TO the NDB.
  • Answer (B) is correct. On Fig. 105, aircraft 8 has a magnetic heading of 315°. To determine the relative bearing given a 315° magnetic bearing TO the station, use the standard magnetic bearing formula to get the bearing to the station.
    MH + RB = MB
    315° + RB = 315°
    RB TO =

    Illustration 1 indicates a 0° relative bearing.

  • Answer (C) is incorrect because illustration 3 indicates you are on the 210°, not 315°, MB TO the NDB.

[157]

(Refer to figure 102.) On the basis of this information, the magnetic bearing TO the station would be
When an aircraft is decelerated some attitude indicators will precess and incorrectly indicate a group of answer choices?

Figure 102
  • Answer (A) is correct. On Fig. 102, the aircraft has a magnetic heading of 215° and a relative bearing of 140°. To determine the magnetic bearing TO the station, use the standard magnetic bearing formula to get the bearing TO the station.
    MH + RB = MB
    215° + 140° = MB
    MB = 355°
  • Answer (B) is incorrect because 175° is the MB FROM, not TO, the NDB.
  • Answer (C) is incorrect because 255° is not a related direction in this problem.

[158]

(Refer to figure 102.) On the basis of this information, the magnetic bearing FROM the station would be
When an aircraft is decelerated some attitude indicators will precess and incorrectly indicate a group of answer choices?

Figure 102
  • Answer (A) is incorrect because 255° is not a related direction in this problem.
  • Answer (B) is incorrect because 355° is the MB TO, not FROM, the NDB.
  • Answer (C) is correct. On Fig. 102, the aircraft has a magnetic heading of 215° and a relative bearing of 140°. To determine the magnetic bearing FROM the station, use the standard magnetic bearing formula to get the magnetic bearing TO the station, and then add or subtract 180° to convert MB TO to MB FROM.
    MH + RB = MB
    215° + 140° = 355° MB TO
    MB FROM = 355° - 180° = 175°

[159]

(Refer to figure 103.) On the basis of this information, the magnetic bearing TO the station would be
When an aircraft is decelerated some attitude indicators will precess and incorrectly indicate a group of answer choices?

Figure 103
  • Answer (A) is incorrect because 060° is the MB FROM, not TO, the NDB.
  • Answer (B) is correct. On Fig. 103, the aircraft has a magnetic heading of 330° and a relative bearing of 270°. To determine the magnetic bearing TO the station, use the standard magnetic bearing formula to get the magnetic bearing TO the station.
    MH + RB = MB
    330° + 270° = MB
    MB = 600°
    600° - 360° = 240°
  • Answer (C) is incorrect because 270° is the RB, not MB, TO the NDB.

[160]

(Refer to figure 103.) On the basis of this information, the magnetic bearing FROM the station would be
When an aircraft is decelerated some attitude indicators will precess and incorrectly indicate a group of answer choices?

Figure 103
  • Answer (A) is correct. On Fig. 103, the aircraft has a magnetic heading of 330° and a relative bearing of 270°. To determine the magnetic bearing FROM the station, use the standard magnetic bearing formula to get the magnetic bearing TO the station. Then add or subtract 180° to convert MB TO to MB FROM.
    MH + RB = MB
    330° + 270° = MB
    MB = 600°
    600° - 360° = 240° MB TO
    MB FROM = 240° - 180° = 60°
  • Answer (B) is incorrect because 030° is a heading unrelated to this problem.
  • Answer (C) is incorrect because 240° is the MB TO, not FROM, the NDB.

[161]

(Refer to figure 100.) Which RMI illustration indicates the aircraft to be flying outbound on the magnetic bearing of 235° FROM the station? (Wind 050° at 20 knots.)
When an aircraft is decelerated some attitude indicators will precess and incorrectly indicate a group of answer choices?

Figure 100
  • Answer (A) is correct. A radio magnetic indicator (RMI) consists of a compass card which rotates as the airplane turns. The magnetic heading of the airplane is always directly under the index at the top of the instrument. The bearing pointer displays magnetic bearings TO the selected station. The tail of the indicator tells you which radial you are on or the magnetic bearing FROM the station. Thus, a magnetic bearing of 235° FROM the station is indicated when the tail of the needle is on 235°, as in RMI 3. The airplane's heading is also 235°, which indicates that it is tracking outbound on the 235° MB FROM. The 20-kt. wind from 50° is almost a direct tailwind, which would not require a significant wind correction.
  • Answer (B) is incorrect because RMI 2 indicates outbound on the 235° MB TO, not FROM, the station.
  • Answer (C) is incorrect because RMI 4 indicates a large wind correction to the right, e.g., to compensate for a strong crosswind, which does not exist in this question.

[162]

(Refer to figure 100.) What is the magnetic bearing TO the station as indicated by illustration 4?
When an aircraft is decelerated some attitude indicators will precess and incorrectly indicate a group of answer choices?

Figure 100
  • Answer (A) is correct. The magnetic heading of the airplane is always directly under the index at the top of the instrument. The bearing pointer displays the magnetic bearing TO the selected station. In RMI 4, the needle is pointing to 055°, which is the magnetic bearing TO the station.
  • Answer (B) is incorrect because 235° is the magnetic bearing FROM, not TO, the station.
  • Answer (C) is incorrect because 285° is the magnetic heading, not magnetic bearing.

[163]

(Refer to figure 100.) Which RMI illustration indicates the aircraft is southwest of the station and moving closer TO the station?
When an aircraft is decelerated some attitude indicators will precess and incorrectly indicate a group of answer choices?

Figure 100
  • Answer (A) is incorrect because RMI 2 shows a heading of 055° and the VOR behind the airplane, i.e., the airplane is northeast, not southwest, of the station, and moving away FROM the station.
  • Answer (B) is correct. If the airplane is to the southwest of the station and moving toward it, both the heading and the needle should be indicating northeast, which is shown in RMI 1. It indicates a magnetic bearing TO the station of 055°. The magnetic heading is also 055°, which means the airplane is flying to the station.
  • Answer (C) is incorrect because RMI 3 shows a heading of 235° and the VOR behind the airplane, i.e., the airplane moving further FROM, not TO, the station.

[164]

(Refer to figure 100.) Which RMI illustration indicates the aircraft is located on the 055° radial of the station and heading away from the station?
When an aircraft is decelerated some attitude indicators will precess and incorrectly indicate a group of answer choices?

Figure 100
  • Answer (A) is correct. A radial, or bearing FROM, is indicated by the tail of the needle. Thus, the 055° radial is indicated when the tail of the needle is on 055°, as in RMI 2. The heading is also 055°, which means you are flying toward the northeast away from the station and are on the 055° radial.
  • Answer (B) is incorrect because RMI 1 indicates flying toward the station on the 235° radial.
  • Answer (C) is incorrect because RMI 3 indicates flying away from the station on the 235° radial.

[165]

(Refer to figure 107.) Where should the bearing pointer be located relative to the wing-tip reference to maintain the 16 DME range in a right-hand arc with a right crosswind component?
  1. Behind the right wing-tip reference for VOR-1.
  2. Behind the right wing-tip reference for VOR-2.
  3. Ahead of the right wing-tip reference for VOR-2.
When an aircraft is decelerated some attitude indicators will precess and incorrectly indicate a group of answer choices?

Figure 107
  • Answer (A) is incorrect because VOR 1 is pointing toward the left, which would indicate a left-hand, not right-hand, DME arc.
  • Answer (B) is incorrect because the bearing pointer would be behind the wing-tip reference if you were crabbed away from the VORTAC such as in a left-hand crosswind and a right-hand arc.
  • Answer (C) is correct. Normally when flying a DME arc with an RMI, the RMI needle will point directly to the VORTAC, and in a no-wind situation it will be on either a 90° or 3 o'clock (for right-hand arc) or 270° or 9 o'clock (for left-hand arc) indication. Since you are flying a right-hand arc, you should be using VOR 2, which points to the right. The right crosswind component will be blowing you away from the VORTAC. You should crab to the right so the VOR 2 bearing pointer is in front of the right wing-tip reference. This indicates you are correcting back into the wind and toward the VORTAC.

[166]

(Refer to figure 108.) Where should the bearing pointer be located relative to the wing-tip reference to maintain the 16 DME range in a left-hand arc with a left crosswind component?
  1. Behind the left wing-tip reference for the VOR-2.
  2. Ahead of the left wing-tip reference for the VOR-2.
  3. Ahead of the right wing-tip reference for the VOR-1.
When an aircraft is decelerated some attitude indicators will precess and incorrectly indicate a group of answer choices?

Figure 108
  • Answer (A) is incorrect because the needle would be behind the wing-tip if you were crabbed away from, not toward, the VORTAC.
  • Answer (B) is correct. Since you are flying a left-hand arc, you should be using VOR 2, which points to the left. The left crosswind component will be blowing you away from the VORTAC. You should crab to the left so the VOR 2 bearing pointer is in front of the left wing-tip reference. This indicates you are correcting back into the wind and toward the VORTAC.
  • Answer (C) is incorrect because you should use VOR 2 as you are making a left-hand, not right-hand, turn.

[167]

(Refer to figure 104.) If the radio magnetic indicator is tuned to a VOR, which illustration indicates the aircraft is on the 010° radial?
When an aircraft is decelerated some attitude indicators will precess and incorrectly indicate a group of answer choices?

Figure 104
  • Answer (A) is incorrect because RMI 1 indicates the 115° radial, not the 010° radial.
  • Answer (B) is correct. A radio magnetic indicator (RMI) consists of a rotating compass card and one or more indicators which point to stations. The tail of the indicator tells you what radial you are on. RMI 3 shows the tail of the indicator on 010°, which means that you are on the 010° radial.
  • Answer (C) is incorrect because RMI 2 indicates the 315° radial, not the 010° radial.

[168]

(Refer to figure 104.) If the radio magnetic indicator is tuned to a VOR, which illustration indicates the aircraft is on the 115° radial?
When an aircraft is decelerated some attitude indicators will precess and incorrectly indicate a group of answer choices?

Figure 104
  • Answer (A) is correct. The tail of the RMI indicator tells you what radial you are on. RMI 1 shows the tail of the indicator on 115°, which means that you are on the 115° radial.
  • Answer (B) is incorrect because RMI 2 indicates the 315° radial, not the 115° radial.
  • Answer (C) is incorrect because RMI 3 indicates the 010° radial, not the 115° radial.

[169]

(Refer to figure 104.) If the radio magnetic indicator is tuned to a VOR, which illustration indicates the aircraft is on the 335° radial?
When an aircraft is decelerated some attitude indicators will precess and incorrectly indicate a group of answer choices?

Figure 104
  • Answer (A) is incorrect because RMI 3 indicates the 010° radial, not the 335° radial.
  • Answer (B) is incorrect because RMI 2 indicates the 315° radial, not the 335° radial.
  • Answer (C) is correct. The tail of the RMI indicator tells you what radial you are on. RMI 4 shows the tail of the indicator on 335°, which means that you are on the 335° radial.

[170]

(Refer to figure 104.) If the radio magnetic indicator is tuned to a VOR, which illustration indicates the aircraft is on the 315° radial?
When an aircraft is decelerated some attitude indicators will precess and incorrectly indicate a group of answer choices?

Figure 104
  • Answer (A) is incorrect because RMI 3 indicates the 010° radial, not the 315° radial.
  • Answer (B) is incorrect because RMI 4 indicates the 335° radial, not the 315° radial.
  • Answer (C) is correct. The tail of the RMI indicator tells you what radial you are on. RMI 2 shows the tail of the indicator on 315°, which means that you are on the 315° radial.

[171]

In which publication can the VOR receiver ground checkpoint(s) for a particular airport be found?
  1. Airman's Information Manual.
  2. En Route Low Altitude Chart.
  3. Airport/Facility Directory.
  • Answer (A) is incorrect because the Aeronautical Information Manual contains general flight information, not data concerning specific airports.
  • Answer (B) is incorrect because En Route Low Altitude Charts do not indicate VOR receiver ground checkpoints, only VOT frequencies.
  • Answer (C) is correct. The Airport/Facility Directory provides a listing of available VOR receiver ground checkpoints.

[172]

Which is the maximum tolerance for the VOR indication when the CDI is centered and the aircraft is directly over the airborne checkpoint?
  1. Plus or minus 4° of the designated radial.
  2. Plus or minus 6° of the designated radial.
  3. Plus 6° or minus 4° of the designated radial.
  • Answer (A) is incorrect because the tolerance for an airborne checkpoint is ±6°, not ±4°, which is the tolerance for a ground checkpoint or a VOT.
  • Answer (B) is correct. Airborne checkpoints consist of certified radials that should be received over specific landmarks while airborne in the immediate vicinity of an airport. The maximum tolerance when the CDI is centered is ±6°.
  • Answer (C) is incorrect because the tolerance for an airborne checkpoint is ±6°, not +6° or -4°.

[173]

When the CDI needle is centered during an airborne VOR check, the omni-bearing selector and the TO/FROM indicator should read
  1. within 6° of the selected radial.
  2. within 4° of the selected radial.
  3. 0° TO, only if you are due south of the VOR.
  • Answer (A) is correct. Airborne VOR checkpoints consist of certified radials that should be received over specific landmarks. If no checkpoint is available, a prominent ground point should be selected more than 20 NM from a VOR station that is along an established VOR airway. Once over this point with the CDI centered, the OBS should indicate within 6° of the published radial.
  • Answer (B) is incorrect because the maximum error for a ground, not airborne, VOR check is ±4°.
  • Answer (C) is incorrect because you should use a certified airborne checkpoint or select a ground reference that is under an established VOR airway, not a randomly selected radial.

[174]

When making an airborne VOR check, what is the maximum allowable tolerance between the two indicators of a dual VOR system (units independent of each other except the antenna)?
  1. 4° between the two indicated radials of a VOR.
  2. Plus or minus 4° when set to identical radials of a VOR.
  3. 6° between the two indicated radials of a VOR.
  • Answer (A) is correct. If a dual system VOR (units independent of each other except for the antenna) is installed in the airplane, one system may be checked against the other in place of other VOR check procedures. The test consists of tuning both systems to the same VOR and centering the CDI needles, then noting the bearing variation between the two VOR units. It should be less than 4°.
  • Answer (B) is incorrect because the CDIs are to be centered, not set to the same radials.
  • Answer (C) is incorrect because it is a maximum tolerance of 4°, not 6°.

[175]

While airborne, what is the maximum permissible variation between the two indicated bearings when checking one VOR system against the other?
  1. Plus or minus 4° when set to identical radials of a VOR.
  2. 4° between the two indicated bearings to a VOR.
  3. Plus or minus 6° when set to identical radials of a VOR.
  • Answer (A) is incorrect because the CDIs must be centered, not set to identical radials.
  • Answer (B) is correct. If a dual system VOR (units independent of each other except for the antenna) is installed in the airplane, one system may be checked against the other in place of other check procedures. The test consists of tuning both systems to the same VOR with the CDI centered and noting the bearing variation between the two VOR units. It should be less than 4°.
  • Answer (C) is incorrect because it is a maximum permissible variation of 4°, not 6°, between the indicated bearings.

[176]

How should the pilot make a VOR receiver check when the aircraft is located on the designated checkpoint on the airport surface?
  1. Set the OBS on the designated radial. The CDI must center within plus or minus 4° of that radial with a FROM indication.
  2. Set the OBS on 180° plus or minus 4°; the CDI should center with a FROM indication.
  3. With the aircraft headed directly toward the VOR and the OBS set to 000°, the CDI should center within plus or minus 4° of that radial with a TO indication.
  • Answer (A) is correct. A VOR receiver check is a checkpoint on the airport surface near a VOR. When the aircraft is on the checkpoint, the designated radial should be set on the OBS. The CDI must then center within 4° of the radial. Also, there will be a FROM indication.
  • Answer (B) is incorrect because the specified radial, not 180°, should be set on the OBS.
  • Answer (C) is incorrect because VOR indications are given the same no matter which heading the aircraft is on. The VOR indication is based upon position, not heading.

[177]

How should the pilot make a VOR receiver check when the aircraft is located on the designated checkpoint on the airport surface?
  1. Set the OBS on the designated radial. The CDI must center within plus or minus 4° of that radial with a FROM indication.
  2. With the aircraft headed directly toward the VOR and the OBS set to 000°, the CDI should center within plus or minus 4° of that radial with a TO indication.
  3. Set the OBS on 180° plus or minus 4°; the CDI should center with a FROM indication.
  • Answer (A) is correct. A VOR receiver check is a checkpoint on the airport surface near a VOR. When the airplane is on the checkpoint, the designated radial should be set on the OBS. The CDI must then center within 4° of the radial. Also, there will be a FROM indication.
  • Answer (B) is incorrect because VOR indications are given the same no matter which heading the aircraft is on. The VOR indication is based upon position, not heading.
  • Answer (C) is incorrect because the specified radial, not 180°, should be set on the OBS.

[178]

Where can the VOT frequency for a particular airport be found?
  1. In the Airport/Facility Directory and on the A/G Voice Communication Panel of the En Route Low Altitude Chart.
  2. On the IAP Chart and in the Airport/Facility Directory.
  3. Only in the Airport/Facility Directory.
  • Answer (A) is correct. Both the Airport/Facility Directory and the A/G voice communication panel of the En Route Low Altitude Chart provide a listing of the VOT frequency for a particular airport.
  • Answer (B) is incorrect because VOT frequencies are not listed on approach charts.
  • Answer (C) is incorrect because VOT frequencies are also published in En Route Low Altitude Charts.

[179]

When using VOT to make a VOR receiver check, the CDI should be centered and the OBS should indicate that the aircraft is on the
  1. 180 radial.
  2. 360 radial.
  3. 090 radial.
  • Answer (A) is incorrect because the VOT transmits only the 360°, not 180°, radial in all directions.
  • Answer (B) is correct. A VOT transmits only the 360° radial. Thus, with the CDI centered, the OBS should indicate 0° with a FROM indication and 180° with a TO indication.
  • Answer (C) is incorrect because the VOT transmits only the 360°, not 090°, radial in all directions.

[180]

Which indications are acceptable tolerances when checking both VOR receivers by use of the VOT?
  1. 176° TO and 003° FROM, respectively.
  2. 360° TO and 003° TO, respectively.
  3. 001° FROM and 005° FROM, respectively.
  • Answer (A) is correct. A VOT transmits a 360° radial in all directions. Thus, with the course deviation indicator (CDI) centered, the omnibearing selector (OBS) should read 0° with the TO-FROM indicator showing FROM, or the OBS should read 180° with the TO-FROM indicator showing TO, with a maximum error of 4°.
  • Answer (B) is incorrect because at 000°, FROM, not TO, should be indicated.
  • Answer (C) is incorrect because it exceeds the 4° maximum error limit.

[181]

(Refer to figure 81.) When checking a dual VOR system by use of a VOT, which illustration indicates the VOR's are satisfactory?
When an aircraft is decelerated some attitude indicators will precess and incorrectly indicate a group of answer choices?

Figure 81
  • Answer (A) is correct. A VOT transmits a 360° radial in all directions. Thus, when using an RMI, the tail of each indicator should point to 360°, ±4°.
  • Answer (B) is incorrect because illustration 2 shows the head, not the tail, of one indicator pointing to 360°.
  • Answer (C) is incorrect because illustration 4 shows the heads, not the tails, of both indicators pointing to 360°.

[182]

(Refer to figure 82.) Which is an acceptable range of accuracy when performing an operational check of dual VOR's using one system against the other?
When an aircraft is decelerated some attitude indicators will precess and incorrectly indicate a group of answer choices?

Figure 82
  • Answer (A) is incorrect because, in illustration 1, the needles have a 180° difference.
  • Answer (B) is correct. When performing an operational check of dual VORs using one system against the other, the difference between the two indicated bearings must be 4° or less. On an RMI, which has two VOR indicators, the VORs should point in the same direction, as in illustration 4 of Fig. 82.
  • Answer (C) is incorrect because, in illustration 2, there is a 10° difference.

[183]

What indication should a pilot receive when a VOR station is undergoing maintenance and may be considered unreliable?
  1. No coded identification, but possible navigation indications.
  2. A voice recording on the VOR frequency announcing that the VOR is out of service for maintenance.
  3. Coded identification, but no navigation indications.
  • Answer (A) is correct. The only positive method of identifying a VOR is by Morse Code identification and/or by the recorded voice identification, which is always indicated by use of the word "VOR" following the VOR name. During periods of maintenance, the facility identification is removed, although navigational signals may still be transmitted.
  • Answer (B) is incorrect because an out-of-service VOR is not announced by a voice recording.
  • Answer (C) is incorrect because the coded identi fication is removed when the station is undergoing maintenance.

[184]

A particular VOR station is undergoing routine maintenance. This is evidenced by
  1. removal of the navigational feature.
  2. broadcasting a maintenance alert signal on the voice channel.
  3. removal of the identification feature.
  • Answer (A) is incorrect because the navigational signals may continue even though they are not accurate.
  • Answer (B) is incorrect because an out-of-service VOR is not announced by a voice recording.
  • Answer (C) is correct. The only positive method of identifying a VOR is by Morse Code identification or by the recorded voice identification, which is always indicated by use of the word "VOR" following the VOR name. During periods of maintenance, the coded and/or voice facility identification is removed, although navigational signals may still be transmitted.

[185]

When a VOR/DME is collocated under frequency pairings and the VOR portion is inoperative, the DME identifier will repeat at an interval of
  1. 60 second intervals at 1350 Hz.
  2. 30 second intervals at 1350 Hz.
  3. 20 second intervals at 1020 Hz.
  • Answer (A) is incorrect because the DME identifier repeats at 30-sec., not 60-sec., intervals.
  • Answer (B) is correct. The DME/TACAN coded identification is transmitted at 1350 Hz once for each three or four times the VOR or localizer coded identification is transmitted. When either the VOR or the DME is operative, but not both, it is important to recognize which identifier is retained for the operative facility. A single-coded identification repeated at intervals of approximately 30 sec. indicates that the DME is operative and the VOR is not.
  • Answer (C) is incorrect because the DME identifier repeats at 30-sec., not 20-sec., intervals at 1350 Hz, not 1020 Hz.

[186]

What is the meaning of a single coded identification received only once approximately every 30 seconds from a VORTAC?
  1. The VOR and DME components are operative.
  2. The DME component is operative and the VOR component is inoperative.
  3. VOR and DME components are both operative, but voice identification is out of service.
  • Answer (A) is incorrect because a constant series of identity codes indicates that both the VOR and DME are operative.
  • Answer (B) is correct. The DME/TACAN coded identification is transmitted at 1350 Hz once for each three or four times the VOR or localizer coded identification is transmitted. When either the VOR or the DME is operative, but not both, it is important to recognize which identifier is retained for the operative facility. A single-coded identification repeated at intervals of approximately 30 sec. indicates that the DME is operative and the VOR is not.
  • Answer (C) is incorrect because voice identification operates independently of the identity codes.

[187]

What angular deviation from a VOR course centerline is represented by a full-scale deflection of the CDI?
  • Answer (A) is incorrect because 5° is indicated by a 2-dot deflection on a 4-dot VOR scale.
  • Answer (B) is incorrect because 4° is indicated by a 2-dot deflection on a 5-dot VOR scale.
  • Answer (C) is correct. On VORs, full needle deflection from the center position to either side of the dial indicates that the aircraft is 10° or more off course, assuming normal needle sensitivity.

[188]

Full scale deflection of a CDI occurs when the course deviation bar or needle
  1. deflects from half scale left to half scale right.
  2. deflects from the center of the scale to either far side of the scale.
  3. deflects from left side of the scale to right side of the scale.
  • Answer (A) is incorrect because it indicates moving from a left half deflection to a right half deflection, i.e., 5° left of course to 5° right of course.
  • Answer (B) is correct. Full-scale deflection of a CDI occurs when the needle deflects from the center of the scale to either far side of the scale. This indicates that the aircraft is 10° or more off course, assuming normal needle sensitivity.
  • Answer (C) is incorrect because it indicates moving from a left full deflection to a right full deflection, i.e., 10° left of course to 10° right of course.

[189]

For operations off established airways at 17,000 feet MSL in the contiguous U.S., (H) Class VORTAC facilities used to define a direct route of flight should be no farther apart than
  • Answer (A) is incorrect because 100 NM is the range of an (H) Class VORTAC at 17,000 ft. MSL; thus the distance between two (H) Class VORTAC facilities can be 200 NM.
  • Answer (B) is correct. (H) Class VORTAC facilities have a range of 100 NM from 14,500 ft. AGL up to 18,000 ft. Thus, (H) Class VORTAC facilities should be no farther apart than 200 NM.
  • Answer (C) is incorrect because 75 NM is the range of an (HH) Class NDB facility, not an (H) Class VORTAC.

[190]

For IFR operations off of established airways below 18,000 feet, VOR navigational aids used to describe the "route of flight'' should be no more than
  1. 70 NM apart.
  2. 40 NM apart.
  3. 80 NM apart.
  • Answer (A) is incorrect because VORs should be no more than 80 NM, not 70 NM, apart to define a route of flight off of established airways below 14,500 ft. AGL.
  • Answer (B) is incorrect because 40 NM is the range of an (H) Class VOR from 1,000 ft. AGL to 14,500 ft. AGL. VORs used to describe a route of flight off of established airways below 14,500 ft. AGL should be no more than 80 NM apart.
  • Answer (C) is correct. (H) Class VOR facilities have a range of 40 NM from 1,000 ft. AGL up to 14,500 ft. AGL, and a range of 100 NM from 14,500 ft. AGL up to 18,000 ft. Thus, in general, VOR navigational aids used to describe a route of flight off of established airways below 18,000 ft. (more-specifically, below 14,500 ft. AGL) should be no more than 80 NM apart.

[191]

When using VOR for navigation, which of the following should be considered as station passage?
  1. The first movement of the CDI as the airplane enters the zone of confusion.
  2. The moment the TO-FROM indicator becomes blank.
  3. The first positive, complete reversal of the TO-FROM indicator.
  • Answer (A) is incorrect because it indicates flight into the zone of confusion over the VOR, not station passage.
  • Answer (B) is incorrect because it is an indication that you are in the zone of confusion over the VOR, not an indication of station passage.
  • Answer (C) is correct. When approaching a VOR, the TO-FROM indicator and the CDI flicker as the airplane flies into the zone of confusion (no signal area). Station passage is shown by complete reversal of the TO-FROM indicator.

[192]

Which of the following should be considered as station passage when using VOR?
  1. The first complete reversal of the TO-FROM indicator.
  2. The first full-scale deflection of the CDI.
  3. The first flickering of the TO-FROM indicator and CDI as the station is approached.
  • Answer (A) is correct. When approaching a VOR, the TO-FROM indicator and the CDI flicker as the airplane flies into the zone of confusion (no signal area). Station passage is shown by complete reversal of the TO-FROM indicator.
  • Answer (B) is incorrect because it is an indication that you are in the zone of confusion over the VOR, not an indication of station passage.
  • Answer (C) is incorrect because it indicates flight into the zone of confusion over the VOR, not station passage.

[193]

When checking the sensitivity of a VOR receiver, the number of degrees in course change as the OBS is rotated to move the CDI from center to the last dot on either side should be between
  1. 10° and 12°.
  2. 8° and 10°.
  3. 5° and 6°.
  • Answer (A) is correct. In addition to VOR receiver checks, course sensitivity may be checked by noting the number of degrees of change in the course selected as you rotate the OBS to move the CDI from center to the last dot on either side. This range should be between 10° and 12°.
  • Answer (B) is incorrect because 8° to 10° of course change should result in a ¾-scale, not full-scale, needle deflection.
  • Answer (C) is incorrect because 5° to 6° of course change should result in a ½-scale, not full-scale, needle deflection.

[194]

A VOR receiver with normal five-dot course sensitivity shows a three-dot deflection at 30 NM from the station. The aircraft would be displaced approximately how far from the course centerline?
  • Answer (A) is correct. Airplane displacement from a course is approximately 200 ft. per dot per nautical mile for VORs. For example, at 30 NM from the station, a one-dot deflection indicates approximately 1 NM displacement of the airplane from the course centerline. A full course deflection is 5 dots. With a 3-dot deflection, one would be about 3 NM from the course centerline.
  • Answer (B) is incorrect because two dots, not three dots, indicate 2 NM off course.
  • Answer (C) is incorrect because five dots, not three dots, indicate 5 NM off course.

[195]

An aircraft which is located 30 miles from a VOR station and shows a ½ scale deflection on the CDI would be how far from the selected course centerline?
  1. 2½ miles.
  2. 1½ miles.
  3. 3½ miles.
  • Answer (A) is correct. Airplane displacement from a course is approximately 200 ft. per dot per nautical mile for VORs. For example, at 30 NM from the station, a 1-dot deflection indicates approximately 1 NM displacement of the airplane from the course centerline. A full course deflection is 5 dots. Since a ½-scale deflection on the CDI would be 2½ dots, the airplane would be about 2½ mi. from the course centerline.
  • Answer (B) is incorrect because 1½ mi. would be indicated by a 1½-dot deflection.
  • Answer (C) is incorrect because 3½ mi. would be indicated by a 3½-dot deflection.

[196]

What angular deviation from a VOR course centerline is represented by a ½ scale deflection of the CDI?
  • Answer (A) is incorrect because a full-scale deflection is 10°, not 4°.
  • Answer (B) is correct. A full course deflection is 5 dots, which is approximately 10°. Since rotation of the OBS to move the CDI from the center to the last dot is approximately 10°, a ½-scale deflection would be approximately 5°.
  • Answer (C) is incorrect because a full-scale deflection is 10°, not 8°.

[197]

After passing a VORTAC, the CDI shows ½ scale deflection to the right. What is indicated if the deflection remains constant for a period of time?
  1. The airplane is flying away from the radial.
  2. The OBS is erroneously set on the reciprocal heading.
  3. The airplane is getting closer to the radial.
  • Answer (A) is correct. If the CDI shows a ½-scale deflection to the right, the airplane is flying 5° to the left of course. If it is constant, it means the airplane is flying away from the radial because the 5° off course increases in actual distance as one gets farther away from the VORTAC.
  • Answer (B) is incorrect because, if you use the reciprocal heading, you get reverse indications from the CDI.
  • Answer (C) is incorrect because a steady deflection would indicate the airplane is getting closer to the radial if it were flying TO, not FROM, the station.

[198]

Determine the approximate time and distance to a station if a 5° wingtip bearing change occurs in 1.5 minutes with a true airspeed of 95 knots.
  1. 18 minutes and 33.0 NM.
  2. 18 minutes and 28.5 NM.
  3. 16 minutes and 14.3 NM.
  • Answer (A) is incorrect because 18 min. is less than 1/3 hr., and 1/3 of 95 kt. is less than 33 NM.
  • Answer (B) is correct. Use the following formula to compute the time to station: 
    Time to station = 60 x Min. between bearings
    Degrees of bearing change
     
    =(60 x 1.5)/5 = 18 min.

    Thus, it is 18 min. to the station, which is less than 1/3 of an hour. One-third of 95 is less than 33, and thus must be 28.5 rather than 33.0. On the computer side of your flight computer, put 95 kt. on the outer scale over 60 on the inner scale. Find 18 min. on the inner scale; 28.5 NM is on the outer scale.

  • Answer (C) is incorrect because the time is 18 min., not 16 min.

[199]

(Refer to figure 106.) The course selector of each aircraft is set on 360°. Which aircraft would have a FROM indication on the ambiguity meter and the CDI pointing left of center?
When an aircraft is decelerated some attitude indicators will precess and incorrectly indicate a group of answer choices?

Figure 106
  • Answer (A) is correct. If airplane 2 were heading 360°, the course would be to the left, and the airplane would fly away FROM the VOR. See discussion of VOR orientation presented at the end of the HSI outline.
  • Answer (B) is incorrect because, as airplane 3 is heading 360°, it is flying closer TO, not further FROM, the station, and the CDI would be pointing right, not left, of center.
  • Answer (C) is incorrect because, if airplane 1 were heading 360°, the course would be to the right, not left.

[200]

(Refer to figure 95.) What is the lateral displacement of the aircraft in NM from the radial selected on the No. 1 NAV?
When an aircraft is decelerated some attitude indicators will precess and incorrectly indicate a group of answer choices?

Figure 95
  • Answer (A) is incorrect because 7.5 NM would be indicated by a ¾ deflection.
  • Answer (B) is correct. On VORs, the displacement from course is approximately 200 ft. per dot per nautical mile. At 30 NM from the station, one-dot deflection indicates approximately 1 NM displacement of the airplane from the course centerline. At 60 NM, it would be 2 NM for every dot of displacement. Since here displacement is 2½ dots, the airplane would be 5 NM from the centerline.
  • Answer (C) is incorrect because 10 NM would be indicated by a full deflection.

[201]

(Refer to figure 95.) On which radial is the aircraft as indicated by the No. 1 NAV?
When an aircraft is decelerated some attitude indicators will precess and incorrectly indicate a group of answer choices?

Figure 95
  • Answer (A) is incorrect because R-175 would require a TO indicator and a left deflection.
  • Answer (B) is correct. The course selector in Fig. 95 is set on 350° with a FROM reading, indicating that, if the course deviation bar were centered, the airplane would be on R-350. Since a total deflection is approximately 10° to 12°, one-half deflection is 5° to 6°. Here, deflection is less than one-half, so it is about 5°. The course deviation bar indicates that this airplane is to the west of R-350, which would be R-345.
  • Answer (C) is incorrect because R-165 would require a TO indicator.

[202]

(Refer to figure 95.) Which OBS selection on the No. 1 NAV would center the CDI and change the ambiguity indication to a TO?
When an aircraft is decelerated some attitude indicators will precess and incorrectly indicate a group of answer choices?

Figure 95
  • Answer (A) is incorrect because the airplane is currently on R-345 with a FROM, not TO, indication.
  • Answer (B) is incorrect because the airplane is currently on R-345, not R-355.
  • Answer (C) is correct. The course selector in Fig. 95 is set on 350°, resulting in a FROM reading and a ½-scale needle deflection. Thus, the airplane is 5° or 6° west of R-350, i.e., R-345. Setting the OBS to the reciprocal course of 165° would center the CDI with a TO indication.

[203]

(Refer to figure 95.) What is the lateral displacement in degrees from the desired radial on the No. 2 NAV?
When an aircraft is decelerated some attitude indicators will precess and incorrectly indicate a group of answer choices?

Figure 95
  • Answer (A) is incorrect because each dot is 2°, not 1°.
  • Answer (B) is correct. Since on a standard 5-dot VOR indicator a full deflection of 5 dots is about 10°, 2 dots means a 4° deflection.
  • Answer (C) is incorrect because each dot is 2°, not ½°.

[204]

(Refer to figure 95.) Which OBS selection on the No. 2 NAV would center the CDI?
When an aircraft is decelerated some attitude indicators will precess and incorrectly indicate a group of answer choices?

Figure 95
  • Answer (A) is correct. The course selector in Fig. 95 is set to 170° (it is not an HSI; it is a VOR), and the TO-FROM indicator indicates FROM, which means the airplane would be on R-170 if the course deviation bar were centered. Since the bar indicates a left 2-dot deflection, the airplane is 4° to the west of the radial, or on R-174.
  • Answer (B) is incorrect because, on R-335 with an OBS setting of 170°, there would be a TO, not FROM, indication.
  • Answer (C) is incorrect because a right, not left, deflection would indicate R-166.

[205]

(Refer to figure 95.) Which OBS selection on the No. 2 NAV would center the CDI and change the ambiguity indication to a TO?
When an aircraft is decelerated some attitude indicators will precess and incorrectly indicate a group of answer choices?

Figure 95
  • Answer (A) is incorrect because the airplane is on R-174, not R-166.
  • Answer (B) is incorrect because the airplane is on R-174, not R-346.
  • Answer (C) is correct. The course selector in Fig. 95 is set to 170° (it is not an HSI; it is a VOR), and the TO-FROM indicator indicates FROM, which means the airplane would be on R-170 if the course deviation bar were centered. Since the bar indicates a 2-dot left deflection, the airplane is 4° to the west of the radial, or on R-174. To obtain a TO indication, one would have to change the OBS selection by 180° from 174° to 354°.

[206]

(Refer to figure 109.) In which general direction from the VORTAC is the aircraft located?
  1. Northeast.
  2. Southeast.
  3. Southwest.
When an aircraft is decelerated some attitude indicators will precess and incorrectly indicate a group of answer choices?

Figure 109
  • Answer (A) is correct. The course indicating arrow (OBS) is set to 180°, and the TO-FROM indicator indicates TO (triangle pointing TO arrowhead), which means the airplane is north of the VORTAC. Since the course deviation bar indicates that the airplane needs to be flown to the right, the airplane is to the east of the 360° radial of the VORTAC. Thus, the airplane is northeast of the VORTAC.
  • Answer (B) is incorrect because, if the airplane were southeast of the VORTAC, there would be a FROM, not TO, indication.
  • Answer (C) is incorrect because, if the airplane were southwest of the VORTAC, there would be a FROM, not TO, indication and a left, not right, bar deflection.

[207]

(Refer to figures 98 and 99.) To which aircraft position does HSI presentation "A" correspond?
When an aircraft is decelerated some attitude indicators will precess and incorrectly indicate a group of answer choices?

Figure 98

When an aircraft is decelerated some attitude indicators will precess and incorrectly indicate a group of answer choices?

Figure 99

  • Answer (A) is incorrect because airplane 11 is to the right of the 360/180 radials and is south of the 270/090 radials, which would require a FROM, not TO, indication and a left, not right, bar deflection.
  • Answer (B) is correct. On Figs. 98 and 99, HSI "A" has a VOR course selection of 090°, with a TO indication, meaning the airplane is to the left of the 360/180 radials. It has a right deflection, which means it is north of the 270/90 radials. The airplane heading is 205°, which means airplane 1 is described.
  • Answer (C) is incorrect because airplane 8 is to the right of the 360/180 radials, which would require a FROM, not TO, indication.

[208]

(Refer to figures 98 and 99.) To which aircraft position does HSI presentation "B" correspond?
When an aircraft is decelerated some attitude indicators will precess and incorrectly indicate a group of answer choices?

Figure 98

When an aircraft is decelerated some attitude indicators will precess and incorrectly indicate a group of answer choices?

Figure 99

  • Answer (A) is correct. On Figs. 98 and 99, HSI "B" has a VOR course selection of 270° with a FROM indication, meaning that the airplane is to the left of the 360/180 radials. Since it has a right deflection, the airplane is south of R-270. Given a heading of 135°, airplane 19 is described.
  • Answer (B) is incorrect because airplane 13 is to the right of the 360/180 radials, which would require a TO, not FROM, indication.
  • Answer (C) is incorrect because airplane 9 would require a TO, not FROM, indication and a left, not right, bar deflection.

[209]

(Refer to figures 98 and 99.) To which aircraft position does HSI presentation "C" correspond?
When an aircraft is decelerated some attitude indicators will precess and incorrectly indicate a group of answer choices?

Figure 98

When an aircraft is decelerated some attitude indicators will precess and incorrectly indicate a group of answer choices?

Figure 99

  • Answer (A) is correct. On Figs. 98 and 99, HSI "C" has a VOR course selection of 360° with a TO indication, meaning the airplane is south of the 270/090 radials. Since the course deflection bar is to the left, the airplane is to the east of the 180° radial. Given a 310° heading, airplane 12 is described.
  • Answer (B) is incorrect because airplane 7 is north of the 270/090 radials, which would require a FROM, not TO, indication.
  • Answer (C) is incorrect because airplane 6 is north of the 270/090 radials, which would require a FROM, not TO, indication and has a north, not 310°, heading.

[210]

(Refer to figures 98 and 99.) To which aircraft position does HSI presentation "D" correspond?
When an aircraft is decelerated some attitude indicators will precess and incorrectly indicate a group of answer choices?

Figure 98

When an aircraft is decelerated some attitude indicators will precess and incorrectly indicate a group of answer choices?

Figure 99

  • Answer (A) is correct. On Figs. 98 and 99, HSI "D" has a VOR course selection (OBS) of 180°. Its FROM indication means the airplane is south of R-270/90. Since the course deflection bar is to the left, the airplane is west of R-180. Given the heading of 180°, the position describes airplane 17.
  • Answer (B) is incorrect because airplane 15 would have a centered deflection bar and a north, not 180°, heading.
  • Answer (C) is incorrect because airplane 4 is north of the 270/090 radials, which would have a TO, not FROM, indication.

[211]

(Refer to figures 98 and 99.) To which aircraft position does HSI presentation "E" correspond?
When an aircraft is decelerated some attitude indicators will precess and incorrectly indicate a group of answer choices?

Figure 98

When an aircraft is decelerated some attitude indicators will precess and incorrectly indicate a group of answer choices?

Figure 99

  • Answer (A) is incorrect because airplane 15 is south of the 270/090 radials, which would require a centered deflection bar and a TO, not FROM, indication.
  • Answer (B) is correct. On Figs. 98 and 99, HSI "E" has a VOR course selection of 360°. Its FROM indication means the airplane is north of R-270/90. Given the course deflection bar to the left, the airplane is to the east of the 360° radial. Given the 360° heading, the position describes airplane 6.
  • Answer (C) is incorrect because airplane 5 would require a centered deflection bar, and has a south, not 360°, heading.

[212]

(Refer to figure 98 and 99.) To which aircraft position does HSI presentation "F" correspond?
When an aircraft is decelerated some attitude indicators will precess and incorrectly indicate a group of answer choices?

Figure 98

When an aircraft is decelerated some attitude indicators will precess and incorrectly indicate a group of answer choices?

Figure 99

  • Answer (A) is incorrect because airplane 14 is east of the 360/180 radials, which would require a bar deflection, not a centered bar.
  • Answer (B) is correct. On Figs. 98 and 99, HSI "F" has a VOR course selection of 180° with a FROM indication, meaning that the airplane is south of the 270/90 radials. Since the course deflection bar is centered, the airplane is on R-180. Given a heading of 045° (at the top of the HSI), airplane 16 is described.
  • Answer (C) is incorrect because airplane 10 is east of the 360/180 radials and north of the 270/090 radials, which would require a bar deflection, not a centered bar, and a TO, not FROM, indication.

[213]

(Refer to figure 110.) In which general direction from the VORTAC is the aircraft located?
  1. Southwest.
  2. Northwest.
  3. Northeast.
When an aircraft is decelerated some attitude indicators will precess and incorrectly indicate a group of answer choices?

Figure 110
  • Answer (A) is incorrect because southwest would require a TO, not FROM, indication.
  • Answer (B) is incorrect because northwest would require a full right, not a ½-scale, deflection.
  • Answer (C) is correct. The course indicator arrow (OBS) is set to 60°, and the TO-FROM indicates FROM (opposite the head of the arrow), which means the airplane is northeast of the VORTAC. Since the course deviation bar indicates a deflection of 3 dots to the right, the airplane is to the north of the 60° radial by 6°, which is in the northeast.

[214]

(Refer to figure 111.) In which general direction from the VORTAC is the aircraft located?
  1. Northeast.
  2. Northwest.
  3. Southeast.
When an aircraft is decelerated some attitude indicators will precess and incorrectly indicate a group of answer choices?

Figure 111
  • Answer (A) is incorrect because a left, not right, deflection would indicate the airplane is in the northeast.
  • Answer (B) is correct. The course indicator arrow (OBS) is set to 360°, and the TO-FROM indicator is FROM, which means the airplane is north of the VORTAC. Since the course deviation bar is to the right, the airplane is to the west of the 360° radial of the VORTAC. Accordingly, the airplane is to the northwest of the VORTAC.
  • Answer (C) is incorrect because a TO, not FROM, indication would indicate the airplane is south of the VORTAC.

[215]

(Refer to figures 96 and 97.) To which aircraft position(s) does HSI presentation "A" correspond?
When an aircraft is decelerated some attitude indicators will precess and incorrectly indicate a group of answer choices?

Figure 96

When an aircraft is decelerated some attitude indicators will precess and incorrectly indicate a group of answer choices?

Figure 97

  • Answer (A) is incorrect because the indication will be the same on either localizer.
  • Answer (B) is correct. On Figs. 96 and 97, HSI "A" has a heading of 360° with no localizer deviation, which means the airplane is on the localizer. Airplanes 6 and 9 are on the localizer with a 360° heading.
  • Answer (C) is incorrect because the indication will be the same on either localizer.

[216]

(Refer to figures 96 and 97.) To which aircraft position(s) does HSI presentation "B" correspond?
When an aircraft is decelerated some attitude indicators will precess and incorrectly indicate a group of answer choices?

Figure 96

When an aircraft is decelerated some attitude indicators will precess and incorrectly indicate a group of answer choices?

Figure 97

  • Answer (A) is correct. On Figs. 96 and 97, HSI "B" has a heading of 090°. It has localizer course setting of 090° with a right deflection, meaning the airplane is south of the localizer. Both airplanes 5 and 13 are described. Note the backcourse setting. If the front course 270° (instead of 90°) had been set, normal (rather than reverse) sensing would be indicated.
  • Answer (B) is incorrect because airplanes 7 and 11 have 270°, not 090°, headings.
  • Answer (C) is incorrect because airplane 11 has a 270°, not 090°, heading.

[217]

(Refer to figures 96 and 97.) To which aircraft position does HSI presentation "C" correspond?
When an aircraft is decelerated some attitude indicators will precess and incorrectly indicate a group of answer choices?

Figure 96

When an aircraft is decelerated some attitude indicators will precess and incorrectly indicate a group of answer choices?

Figure 97

  • Answer (A) is incorrect because airplane 9 has a 360°, not 090°, heading.
  • Answer (B) is incorrect because airplane 4 has a 270°, not 090°, heading.
  • Answer (C) is correct. On Figs. 96 and 97, HSI "C" has a heading of 090° with a centered course deviation bar; thus the airplane is on the localizer with a 090° heading, which describes airplane 12. The backcourse setting (090) has no effect because the deviation bar is centered.

[218]

(Refer to figures 96 and 97.) To which aircraft position does HSI presentation "D" correspond?
When an aircraft is decelerated some attitude indicators will precess and incorrectly indicate a group of answer choices?

Figure 96

When an aircraft is decelerated some attitude indicators will precess and incorrectly indicate a group of answer choices?

Figure 97

  • Answer (A) is incorrect because airplane 10 has a heading of 135°, not 315°.
  • Answer (B) is incorrect because airplane 1 has a heading of 215°, not 315°.
  • Answer (C) is correct. On Figs. 96 and 97, HSI "D" has a heading of 315°. Airplane 2 is the only one with a northwest heading.

[219]

(Refer to figures 96 and 97.) To which aircraft position(s) does HSI presentation "E" correspond?
When an aircraft is decelerated some attitude indicators will precess and incorrectly indicate a group of answer choices?

Figure 96

When an aircraft is decelerated some attitude indicators will precess and incorrectly indicate a group of answer choices?

Figure 97

  • Answer (A) is correct. On Figs. 96 and 97, HSI "E" has a heading of 045°. It has a right deflection with a backcourse HSI setting of 090. This results in reverse sensing, meaning the airplane is south of the localizer. Thus, airplanes 3 and 8 are described.
  • Answer (B) is incorrect because airplane 3 is also south of the localizer with a 045° heading.
  • Answer (C) is incorrect because airplane 8 also has a 045° heading and is south of the localizer.

[220]

(Refer to figures 96 and 97.) To which aircraft position does HSI presentation "F" correspond?
When an aircraft is decelerated some attitude indicators will precess and incorrectly indicate a group of answer choices?

Figure 96

When an aircraft is decelerated some attitude indicators will precess and incorrectly indicate a group of answer choices?

Figure 97

  • Answer (A) is incorrect because airplane 11 would have a left, not centered, course deviation bar.
  • Answer (B) is correct. On Figs. 96 and 97, HSI "F" has a setting of 270° with a centered bar and a 270° heading, which corresponds to airplane 4.
  • Answer (C) is incorrect because airplane 5 has a heading of 090°, not 270°.

[221]

(Refer to figures 96 and 97.) To which aircraft position does HSI presentation "H" correspond?
When an aircraft is decelerated some attitude indicators will precess and incorrectly indicate a group of answer choices?

Figure 96

When an aircraft is decelerated some attitude indicators will precess and incorrectly indicate a group of answer choices?

Figure 97

  • Answer (A) is correct. On Figs. 96 and 97, HSI "H" has a heading of 215°. Airplane 1 is the only one with a southwest heading.
  • Answer (B) is incorrect because airplane 8 has a heading of 045°, not 215°.
  • Answer (C) is incorrect because airplane 2 has a heading of 315°, not 215°.

[222]

(Refer to figures 96 and 97.) To which aircraft position(s) does HSI presentation "G" correspond?
  1. 7 only.
  2. 5 and 13.
  3. 7 and 11.
When an aircraft is decelerated some attitude indicators will precess and incorrectly indicate a group of answer choices?

Figure 96

When an aircraft is decelerated some attitude indicators will precess and incorrectly indicate a group of answer choices?

Figure 97

  • Answer (A) is incorrect because airplane 11 is also north of the localizer with a 270° heading.
  • Answer (B) is incorrect because airplanes 5 and 13 have a heading of 090°, not 270°, and are also south, not north, of the localizer.
  • Answer (C) is correct. On Figs. 96 and 97, HSI "G" has a localizer setting at 270° with a left deviation, meaning the airplane is north of the localizer. With a 270° heading, it corresponds to airplanes 7 and 11.

[223]

(Refer to figures 96 and 97.) To which aircraft position does HSI presentation "I" correspond?
When an aircraft is decelerated some attitude indicators will precess and incorrectly indicate a group of answer choices?

Figure 96

When an aircraft is decelerated some attitude indicators will precess and incorrectly indicate a group of answer choices?

Figure 97

  • Answer (A) is incorrect because airplane 4 would have a centered CDI, not a right deviation.
  • Answer (B) is correct. On Figs. 96 and 97, HSI "I" has a left deviation with a backcourse HSI setting of 090 resulting in reverse sensing. Thus, the airplane is north of the localizer. Airplane 11 has a 270° heading and is north of the localizer.
  • Answer (C) is incorrect because airplane 12 would have a centered CDI, not a right deviation, and has a heading of 090°, not 270°.

[224]

How can a pilot determine if a Global Positioning System (GPS) installed in an aircraft is approved for IFR en route and IFR approaches?
  1. Flight manual supplement.
  2. Aircraft owner's handbook.
  3. GPS operator's manual.
  • Answer (A) is correct. A supplement to the aircraft's POH (flight manual) is required for newly installed equipment. A supplement should state that the GPS is approved for IFR operations (en route and approaches).
  • Answer (B) is incorrect because the aircraft owner's handbook (i.e., information manual or POH) may not have any information on a GPS unit unless the unit was installed as a standard feature.
  • Answer (C) is incorrect because the GPS operator's manual will describe how to operate the GPS unit but will not provide approval for its use in a particular aircraft.

[225]

Hand-held GPS systems, and GPS systems certified for VFR operation, may be used during IFR operations as
  1. the primary source of navigation.
  2. an aid to situational awareness.
  3. the principal reference to determine en route waypoints.
  • Answer (A) is incorrect because hand-held and VFR-only GPS systems are not approved as the primary source of navigation under IFR; they may only be used as an aid to situational awareness.
  • Answer (B) is correct. In order to use a GPS as the primary means of navigation under IFR or to perform instrument approaches, both the GPS receiver and the aircraft installation must FAA-approved. Additionally, the receiver must have a current database of waypoints and instrument approach procedures. Hand-held GPS systems and VFR- only GPS systems are not FAA-approved for IFR operations, and may only be used during IFR operations as an aid to situational awareness.
  • Answer (C) is incorrect because hand-held and VFR-only GPS systems are not approved for IFR operations, and may only be used as an aid to situational awareness. Passage of en route reporting points and fixes (i.e., waypoints) must be verified by some other means (e.g., VOR, ADF, radar, etc.).

[226]

During IFR en route and terminal operations using an approved GPS system for navigation, ground based navigational facilities
  1. must be operational only if RAIM predicts an outage.
  2. are only required during the approach portion of the flight.
  3. must be operational along the entire route.
  • Answer (A) is incorrect because ground-based navigational facilities appropriate to the route must be available and operational regardless of Receiver Autonomous Integrity Monitoring (RAIM) status. However, unless RAIM predicts an outage, active monitoring of these facilities is not necessary.
  • Answer (B) is incorrect because ground-based navigational facilities appropriate to the route must be available and operational along the entire route of flight, not just the approach portion.
  • Answer (C) is correct. In order to conduct IFR en route and terminal operations using an approved GPS system, the avionics necessary to receive all of the ground-based facilities that are appropriate for the route to the destination airport and to any alternate airport must be installed and operational, and the ground-based facilities necessary for these routes must be operational.

[227]

During IFR operation using an approved GPS system for navigation,
  1. active monitoring of an alternate navigation system is always required.
  2. no other navigation system is required.
  3. the aircraft must have an approved and operational alternate navigation system appropriate for the route.
  • Answer (A) is incorrect because active monitoring of an alternate navigation system is only required when GPS RAIM capability is lost or when an outage is predicted, not at all times.
  • Answer (B) is incorrect because aircraft using an approved GPS system for IFR operations must be equipped with an approved and operational alternate means of navigation appropriate to the flight.
  • Answer (C) is correct. Aircraft using GPS under IFR must be equipped with an approved and operational alternate means of navigation appropriate for the route. Active monitoring of alternative navigation equipment is not required if the GPS receiver uses Receiver Autonomous Integrity Monitoring (RAIM). However, active monitoring of an alternate means of navigation is required when the RAIM capability of the GPS equipment is lost, or if RAIM predicts an outage.

[228]

Under which condition must the pilot in command of a civil aircraft have at least an instrument rating?
  1. When operating in Class E airspace.
  2. For a flight in VFR conditions while on an IFR flight plan.
  3. For any flight above an altitude of 1,200 feet AGL, when the visibility is less than 3 miles.
  • Answer (A) is incorrect because an instrument rating is required at all times when operating in Class A, not Class E, airspace.
  • Answer (B) is correct. No person may act as pilot in command of a civil aircraft under IFR or in weather conditions less than the minimums prescribed for VFR flight unless (s)he holds an instrument rating.
  • Answer (C) is incorrect because VFR is permitted in uncontrolled airspace during the day with visibilities of as little as 1 SM when more than 1,200 ft. AGL but below 10,000 ft. MSL.

[229]

The pilot in command of a civil aircraft must have an instrument rating only when operating
  1. under IFR, in weather conditions less than the minimum for VFR flight or in Class A airspace.
  2. in weather conditions less than the minimum prescribed for VFR flight.
  3. under IFR in positive control airspace.
  • Answer (A) is correct. No person may act as pilot in command of a civil aircraft under IFR, in weather conditions less than the minimums prescribed for VFR flight, or in Class A airspace unless (s)he holds an instrument rating.
  • Answer (B) is incorrect because it omits flying in VFR conditions with an IFR clearance.
  • Answer (C) is incorrect because you must have an instrument rating when operating under IFR, in weather conditions less than the minimum for VFR flight, or in Class A airspace, not only under IFR in positive control airspace. Positive control airspace means that ATC will provide separation to all aircraft within that airspace.

[230]

Which flight time may be logged as instrument time when on an instrument flight plan?
  1. Only the time you controlled the aircraft solely by reference to flight instruments.
  2. All of the time the aircraft was not controlled by ground references.
  3. Only the time you were flying in IFR weather conditions.
  • Answer (A) is correct. A pilot may log as instrument flight time only that time during which (s)he operates the aircraft solely by reference to instruments, under actual or simulated instrument flight conditions.
  • Answer (B) is incorrect because VFR flight can be conducted above a cloud layer without visual ground references, i.e., VFR-on-top.
  • Answer (C) is incorrect because time under the hood (i.e., simulated IFR) as well as actual IFR conditions counts as instrument time.

[231]

What portion of dual instruction time may a certificated instrument flight instructor log as instrument flight time?
  1. Only the time during which the instructor flies the aircraft by reference to instruments.
  2. All time during which the instructor acts as instrument instructor in actual instrument weather conditions.
  3. All time during which the instructor acts as instrument instructor, regardless of weather conditions.
  • Answer (A) is incorrect because instructing in (as well as flying in) actual IFR weather conditions can be logged as instrument time by the instructor.
  • Answer (B) is correct. An instrument flight instructor may log as instrument time that time during which (s)he acts as instrument flight instructor in actual instrument weather conditions.
  • Answer (C) is incorrect because the flight conditions must be IMC for the instructor to log flight instruction as instrument time.

[232]

To meet instrument experience requirements of CFR Part 61, section 61.57(c), a pilot enters the condition of flight in the pilot logbook as simulated instrument conditions, what other qualifying information must also be entered?
  1. Location and type of each instrument approach completed and name of safety pilot.
  2. Name and pilot certificate number of safety pilot and type of approaches completed.
  3. Number and type of instrument approaches completed and route of flight.
  • Answer (A) is correct. A pilot may log as instrument flight time only that time during which (s)he operates the aircraft solely by reference to instruments, under actual or simulated instrument flight conditions. Each entry must include the location and type of each instrument approach completed and the name of the safety pilot for each simulated instrument flight.
  • Answer (B) is incorrect because the location is required, but the safety pilot's certificate number is not.
  • Answer (C) is incorrect because the location and type of instrument approaches must be entered along with the safety pilot's name, not the route of flight.

[233]

How long does a pilot meet the recency of experience requirements for IFR flight after successfully completing an instrument proficiency check if no further IFR flights are made?
  1. 6 calendar months.
  2. 90 days.
  3. 12 calendar months.
  • Answer (A) is correct. No pilot may act as pilot in command when operating under IFR or in weather conditions less than the minimums prescribed for VFR unless (s)he has, within the past 6 months, logged instrument time under actual or simulated IFR conditions in the category of aircraft involved or in an appropriate flight simulator or flight training device and has performed at least six instrument approaches, holding procedures, and intercepting and tracking courses through the use of navigation systems. An alternative way to remain current is to pass an instrument proficiency check in the category of aircraft involved.
  • Answer (B) is incorrect because 90 days refers to takeoff and landing currency to carry passengers.
  • Answer (C) is incorrect because 12 months is the time whereafter another instrument proficiency check is required.

[234]

No pilot may act as pilot-in-command of an aircraft, under IFR or in weather conditions less than the minimums prescribed for VFR unless that pilot has, within the preceding 6 calendar months, completed at least
  1. six instrument approaches, holding procedures, intercepting and tracking courses using navigational systems, or passed an instrument proficiency check.
  2. three instrument approaches and logged 3 hours.
  3. six instrument flights under actual IFR conditions.
  • Answer (A) is correct. No person may act as pilot in command under IFR or in weather conditions less than the minimums prescribed for VFR unless, within the preceding 6 calendar months, that person has performed and logged under actual or simulated instrument conditions, either in flight in the appropriate category of aircraft for the instrument privileges sought or in a flight simulator or flight training device that is representative of the aircraft category for the instrument privileges sought, at least six instrument approaches, holding procedures, and intercepting and tracking courses through the use of navigation systems. Alternatively, the pilot may pass an instrument proficiency check in the category of aircraft involved.
  • Answer (B) is incorrect because a pilot must complete at least six, not three, instrument approaches, holding procedures, and intercepting and tracking courses through the use of navigation systems. There is no required minimum number of hours to be logged.
  • Answer (C) is incorrect because a pilot must complete, under actual or simulated instrument conditions, at least six instrument approaches, holding procedures, and intercepting and tracking courses through the use of navigation systems, not six instrument flights under actual IFR conditions.

[235]

To meet the minimum required instrument flight experience to act as pilot in command of an aircraft under IFR, you must have logged within the preceding 6 calendar months in the same category of aircraft: six instrument approaches,
  1. and 6 hours of instrument time in any aircraft.
  2. three of which must be in the same category and class of aircraft to be flown, and 6 hours of instrument time in any aircraft.
  3. holding procedures, intercepting and tracking courses through the use of navigation systems.
  • Answer (A) is incorrect because you must have also accomplished holding procedures and intercepting and tracking courses through the use of navigation systems in the same category and class of aircraft, not accumulated 6 hr. of instrument time in any aircraft.
  • Answer (B) is incorrect because all six, not three, approaches must be in the same category and aircraft. Additionally, you must have accomplished holding procedures and intercepting and tracking courses through the use of navigation systems, not accumulated 6 hr. of instrument time.
  • Answer (C) is correct. No person may act as pilot in command under IFR or in weather conditions less than VFR minimums unless, within the preceding 6 calendar months, (s)he has performed and logged, under actual or simulated instrument conditions, at least six instrument approaches, holding procedures, and intercepting and tracking courses through the use of navigation systems, in the category of aircraft to be flown.

[236]

To meet the minimum instrument experience requirements, within the last 6 calendar months you need
  1. six hours in the same category aircraft.
  2. six hours in the same category aircraft, and at least 3 of the 6 hours in actual IFR conditions.
  3. six instrument approaches, holding procedures, and intercepting and tracking courses in the appropriate category of aircraft.
  • Answer (A) is incorrect because, to be current for IFR, you must have performed six instrument approaches, holding procedures, and intercepting and tracking courses in the same category of aircraft, not accumulated 6 hr. in the same category of aircraft.
  • Answer (B) is incorrect because, to be current for IFR, you must have performed six instrument approaches, holding procedures, and intercepting and tracking courses in the same category of aircraft, not accumulated 6 hr. in the same category aircraft with at least 3 hr. in actual IFR conditions.
  • Answer (C) is correct. No person may act as pilot in command under IFR or in weather conditions less than VFR minimums unless, within the preceding 6 calendar months, (s)he has performed and logged, under actual or simulated instrument conditions, at least six instrument approaches, holding procedures, and intercepting and tracking courses through the use of navigation systems, in the category of aircraft to be flown.

[237]

What recent instrument flight experience requirements must be met before you may act as pilot in command of an airplane under IFR?
  1. A minimum of six instrument approaches, at least three of which must be in an aircraft within the preceding 6 calendar months.
  2. A minimum of six instrument approaches in an airplane, or an approved simulator (airplane) or ground trainer, within the preceding 6 calendar months.
  3. A minimum of six instrument approaches in an aircraft, at least three of which must be in the same category within the preceding 6 calendar months.
  • Answer (A) is incorrect because all six approaches may be done in an approved flight simulator or flight training device; none are required to be done in an aircraft.
  • Answer (B) is correct. No person may act as pilot in command under IFR or in weather conditions less than VFR minimums unless, within the preceding 6 calendar months, (s)he has performed and logged, under actual or simulated instrument conditions, at least six instrument approaches, holding procedures, and intercepting and tracking courses through the use of navigation systems, in the category of aircraft to be flown, in an approved simulator or flight training device, or in any combination of these.
  • Answer (C) is incorrect because all six approaches may be done in an approved flight simulator or flight training device; none are required to be done in an aircraft. In addition, all six approaches, not just three, must be done in the same category of aircraft or a simulator or flight training device representative of that aircraft category.

[238]

An instrument rated pilot, who has not logged any instrument experience in 1 year or more, cannot serve as pilot in command under IFR, unless the pilot
  1. passes an instrument proficiency check in the category of aircraft involved, followed by 6 hours and six instrument approaches, 3 of those hours in the category of aircraft involved.
  2. completes the required 6 hours and six approaches, followed by an instrument proficiency check given by an FAA-designated examiner.
  3. passes an instrument proficiency check in the category of aircraft involved, given by an approved FAA examiner, instrument instructor, or FAA inspector.
  • Answer (A) is incorrect because an instrument proficiency check by itself provides currency. Additional time and approaches are not required.
  • Answer (B) is incorrect because an instrument proficiency check by itself provides currency. Additional time and approaches are not required.
  • Answer (C) is correct. A pilot who does not meet the recent instrument experience requirements during the prescribed time or 6 months thereafter may not serve as pilot in command under IFR or in weather conditions less than the minimums prescribed for VFR until (s)he passes an instrument proficiency check in the category of aircraft involved, given by an FAA inspector, a member of an armed force of the U.S. authorized to conduct flight tests, an FAA-approved check pilot, or a certificated instrument flight instructor. The proficiency check may be completed in an appropriate flight simulator or flight training device.

[239]

After your recent IFR experience lapses, how much time do you have before you must pass an instrument proficiency check to act as pilot in command under IFR?
  1. 6 months.
  2. 12 months.
  3. 90 days.
  • Answer (A) is correct. A pilot who does not meet the recent instrument experience requirements during the prescribed time or 6 months thereafter may not serve as pilot in command under IFR or in weather conditions less than the minimums prescribed for VFR until (s)he passes an instrument proficiency check.
  • Answer (B) is incorrect because 12 months is the time from when you gain IFR currency to when you are required to pass another instrument proficiency check (assuming you have not maintained currency since you gained proficiency).
  • Answer (C) is incorrect because 90 days refers to the takeoff and landing currency requirements to carry passengers.

[240]

What minimum conditions are necessary for the instrument approaches required for IFR currency?
  1. The approaches may be made in an aircraft, approved instrument ground trainer, or any combination of these.
  2. At least three approaches must be made in the same category of aircraft to be flown.
  3. The approaches must be made in the same category and class of aircraft to be flown.
  • Answer (A) is correct. For IFR currency, the six instrument approaches must be made either in the same category of aircraft to be flown or in a flight simulator or training device representative of the aircraft category to be flown, or in any combination of these.
  • Answer (B) is incorrect because all six approaches may be done in an approved flight simulator or training device, not necessarily in an airplane.
  • Answer (C) is incorrect because all six approaches may be done in an approved flight simulator or training device, not necessarily in an airplane.

[241]

A pilot's recent IFR experience expires on July 1 of this year. What is the latest date the pilot can meet the IFR experience requirement without having to take an instrument proficiency check?
  1. December 31, this year.
  2. July 31, this year.
  3. June 30, next year.
  • Answer (A) is correct. A pilot who does not meet the recent instrument experience requirements during the prescribed time or 6 months thereafter may not serve as pilot in command under IFR or in weather conditions less than the minimums prescribed for VFR until (s)he passes an instrument proficiency check. If the 6 months' recency experience period expires on July 1, the 6 months thereafter would expire on December 31 this year.
  • Answer (B) is incorrect because this date represents 1 month instead of 6 months.
  • Answer (C) is incorrect because this date represents 12 months instead of 6 months.

[242]

How may a pilot satisfy the recent flight experience requirement necessary to act as pilot in command in IMC in powered aircraft? Within the previous 6 calendar months, logged
  1. six instrument approaches, holding procedures, and intercepting and tracking courses using navigational systems.
  2. six instrument approaches and 3 hours under actual or simulated IFR conditions within the last 6 months; three of the approaches must be in the category of aircraft involved.
  3. 6 hours of instrument time under actual or simulated IFR conditions within the last 3 months, including at least six instrument approaches of any kind. Three of the 6 hours must be in flight in any category aircraft.
  • Answer (A) is correct. No pilot may act as pilot in command of a powered aircraft under IFR or in weather conditions less than VFR minimums unless, within the preceding 6 calendar months, (s)he has performed and logged, under actual or simulated instrument conditions, at least six instrument approaches, holding procedures, and intercepting and tracking courses through the use of navigation systems, in the category of aircraft to be flown or in an airplane flight simulator or training device.
  • Answer (B) is incorrect because there is no instrument flight time requirement. In addition to the six instrument approaches, you must also log holding procedures and intercepting and tracking courses using navigational systems.
  • Answer (C) is incorrect because there is no instrument flight time requirement. You must log six instrument approaches, holding procedures, and intercepting and tracking courses using navigational systems within the past 6 months, not 3 months.

[243]

Which additional IFR experience is required for you to meet the recent flight experience requirements to act as pilot in command of an airplane under IFR?

Your present instrument experience within the preceding 6 calendar months is

1. three hours with holding, intercepting and tracking courses in an approved airplane flight simulator.

2. two instrument approaches in an airplane.

  1. Four instrument approaches in an airplane, or an approved airplane flight simulator or training device.
  2. Three hours of simulated or actual instrument flight time in a helicopter and two instrument approaches in an airplane or helicopter.
  3. Three instrument approaches in an airplane.
  • Answer (A) is correct. No person may act as pilot in command under IFR or in weather conditions less than the minimums prescribed for VFR, unless within the preceding 6 calendar months that person has performed and logged under actual or simulated instrument conditions, either in flight in the appropriate category of aircraft for the instrument privileges being sought or in a flight simulator or flight training device that is representative of the aircraft category for the instrument privileges sought; at least six instrument approaches, holding procedures, and intercepting and tracking courses through the use of navigation systems. Having logged only two instrument approaches, the pilot must log four more.
  • Answer (B) is incorrect because no minimum time is required; four, not two, more approaches are required; and all approaches must be in an airplane or flight simulator or flight training device representative of an airplane, not a helicopter.
  • Answer (C) is incorrect because four, not three, more approaches are required to have the required minimum of six.

[244]

To carry passengers for hire in an airplane on cross-country flights of more than 50 NM from the departure airport, the pilot in command is required to hold at least
  1. a First-Class Medical certificate.
  2. a Commercial Pilot Certificate with an instrument rating.
  3. a Category II pilot authorization.
  • Answer (A) is incorrect because a first-class medical certificate is required for operations requiring an airline transport pilot certificate.
  • Answer (B) is correct. To carry passengers for hire, the pilot in command is required to hold at least a commercial pilot certificate. Additionally, to carry those passengers for hire in an airplane on cross-country flights of more than 50 NM (or at night), (s)he must also hold an instrument rating on the commercial certificate.
  • Answer (C) is incorrect because Category II refers to an authorization for reduced ILS approach minimums.

[245]

Which limitation is imposed on the holder of a Commercial Pilot Certificate if that person does not hold an instrument rating?
  1. The carrying of passengers for hire on cross-country flights is limited to 50 NM and the carrying of passengers for hire at night is prohibited.
  2. The carrying of passengers or property for hire on cross-country flights at night is limited to a radius of 50 NM.
  3. That person is limited to private pilot privileges at night.
  • Answer (A) is correct. The applicant for a commercial pilot certificate must hold an instrument rating (airplane), or the commercial pilot certificate must be endorsed with a limitation prohibiting the carriage of passengers for hire in airplanes on cross-country flights of more than 50 NM or at night.
  • Answer (B) is incorrect because no passengers may be carried at night without an instrument rating.
  • Answer (C) is incorrect because that person may exercise commercial pilot privileges at night, but with limitations.

[246]

What limitation is imposed on a newly certificated commercial airplane pilot if that person does not hold an instrument pilot rating?
  1. The carrying of passengers for hire on cross-country flights is limited to 50 NM and the carrying of passengers for hire at night is prohibited.
  2. The carrying of passengers for hire on cross-country flights is limited to 50 NM for night flights, but not limited for day flights.
  3. The carrying of passengers or property for hire on cross-country flights at night is limited to a radius of 50 nautical miles (NM).
  • Answer (A) is correct. The applicant for a commercial pilot certificate must hold an instrument rating (airplane), or the commercial pilot certificate must be endorsed with a limitation prohibiting the carriage of passengers for hire in airplanes on cross-country flights of more than 50 NM or at night.
  • Answer (B) is incorrect because no passengers may be carried at night and the flight is limited to 50 NM, not unlimited, for day flights without an instrument rating.
  • Answer (C) is incorrect because the carriage of property (freight) is not limited at night.

[247]

A certificated commercial pilot who carries passengers for hire at night or in excess of 50 NM is required to have at least
  1. an instrument rating in the same category and class of aircraft.
  2. a First-Class Medical Certificate.
  3. an associated type rating if the airplane is of the multiengine class.
  • Answer (A) is correct. A certificated commercial pilot who carries passengers for hire at night or in excess of 50 NM is required to have an instrument rating.
  • Answer (B) is incorrect because only a second-class medical certificate is required of commercial pilots. First-class medical certificates are required of airline transport pilots.
  • Answer (C) is incorrect because, even if the airplane requires a type rating, the commercial pilot must have at least an instrument rating to carry passengers for hire at night or in excess of 50 NM.

[248]

You intend to carry passengers for hire on a night VFR flight in a single-engine airplane within a 25-mile radius of the departure airport. You are required to possess at least which rating(s)?
  1. A Commercial Pilot Certificate with a single-engine land rating.
  2. A Private Pilot Certificate with a single-engine land and instrument airplane rating.
  3. A Commercial Pilot Certificate with a single-engine and instrument (airplane) rating.
  • Answer (A) is incorrect because, to carry passengers for hire at night, one must have an instrument rating as well as a commercial pilot certificate.
  • Answer (B) is incorrect because a commercial, not private, pilot certificate is required to carry passengers for hire.
  • Answer (C) is correct. A commercial pilot certificate with a single-engine airplane rating is required for a pilot to carry passengers for hire and to operate that class of aircraft. Also, an applicant for a commercial pilot certificate must hold an instrument rating (airplane), or the commercial pilot certificate will be endorsed with a limitation prohibiting carrying passengers for hire on cross-country flights of more than 50 NM or at night.

[249]

Who is responsible for determining that the altimeter system has been checked and found to meet 14 CFR Part 91 requirements for a particular instrument flight?
  1. Operator.
  2. Pilot in command.
  3. Owner.
  • Answer (A) is incorrect because the operator is primarily responsible for maintaining the aircraft, but the pilot in command is responsible for determining that the aircraft is airworthy.
  • Answer (B) is correct. The pilot in command of an aircraft is directly responsible for, and is the final authority as to, the airworthiness and operation of that aircraft.
  • Answer (C) is incorrect because the owner is primarily responsible for maintaining the aircraft, but the pilot in command is responsible for determining that the aircraft is airworthy.

[250]

The use of certain portable electronic devices is prohibited on aircraft that are being operated under
  • Answer (A) is incorrect because portable electronic devices are not prohibited in aircraft operated under VFR.
  • Answer (B) is incorrect because portable electronic devices are not prohibited in aircraft operated under DVFR (defense VFR).
  • Answer (C) is correct. The use of portable electronic devices in carrier aircraft and other aircraft operated under IFR is prohibited. This prohibition does not apply to portable voice recorders, hearing aids, heart pacemakers, electric shavers, and other devices which do not interfere with the aircraft's navigation and communication systems.

[251]

Before beginning any flight under IFR, the pilot in command must become familiar with all available information concerning that flight. In addition, the pilot must
  1. be familiar with the runway lengths at airports of intended use, and the alternatives available if the flight cannot be completed.
  2. list an alternate airport on the flight plan and confirm adequate takeoff and landing performance at the destination airport.
  3. list an alternate airport on the flight plan and become familiar with the instrument approaches to that airport.
  • Answer (A) is correct. Each pilot in command shall, before beginning a flight, familiarize him/herself with all available information concerning that flight. For a flight under IFR or a flight not in the vicinity of an airport, this information should include weather reports and forecasts, fuel requirements, alternatives available if the planned flight cannot be completed, and any known traffic delays of which (s)he has been advised by ATC. For any flight, the preflight information should include runway lengths at airports of intended use and takeoff and landing distance data.
  • Answer (B) is incorrect because listing an alternate airport is not required for all IFR flights, i.e., when the destination is forecast to have ceilings above 2,000 ft. and visibility of at least 3 SM.
  • Answer (C) is incorrect because listing an alternate airport is not required for all IFR flights, i.e., when the destination is forecast to have ceilings above 2,000 ft. and visibility of at least 3 SM.

[252]

Before beginning any flight under IFR, the pilot in command must become familiar with all available information concerning that flight including:
  1. the runway lengths at airports of intended use, and the aircraft's takeoff and landing data.
  2. an alternate airport and adequate takeoff and landing performance at the destination airport.
  3. all instrument approaches at the destination airport.
  • Answer (A) is correct. Each pilot in command shall, before beginning a flight, familiarize him/herself with all available information concerning that flight. For a flight under IFR or a flight not in the vicinity of an airport, this information should include weather reports and forecasts, fuel requirements, alternatives available if the planned flight cannot be completed, and any known traffic delays of which (s)he has been advised by ATC. For any flight, the preflight information should include runway lengths at airports of intended use and takeoff and landing distance information.
  • Answer (B) is incorrect because a pilot must be familiar with the airplane's takeoff and landing performance at all airports of intended use, not just the destination airport.
  • Answer (C) is incorrect because, while knowing what approaches are available is a good operating procedure, it is not a required preflight action.

[253]

What are the minimum qualifications for a person who occupies the other control seat as safety pilot during simulated instrument flight?
  1. Private pilot with appropriate category, class, and instrument ratings.
  2. Private pilot certificate with appropriate category and class ratings for the aircraft.
  3. Private pilot with instrument rating.
  • Answer (A) is incorrect because the safety pilot does not need to be instrument rated.
  • Answer (B) is correct. No person may operate a civil aircraft in simulated instrument flight unless the other control seat is occupied by a safety pilot who possesses at least a private pilot certificate with category and class ratings appropriate to the aircraft being flown.
  • Answer (C) is incorrect because the safety pilot's certificate must carry an appropriate category and class (but not instrument) rating, e.g., a private pilot (helicopter) may not act as safety pilot in an airplane.

[254]

When may ATC request a detailed report of an emergency even though a rule has not been violated?
  1. Any time an emergency occurs.
  2. When priority has been given.
  3. When the emergency occurs in controlled airspace.
  • Answer (A) is incorrect because a written report may be requested when priority is given in an emergency, not any time an emergency occurs.
  • Answer (B) is correct. Each pilot in command who is given priority by ATC in an emergency (even though no FAR has been violated) shall, if requested by ATC, submit a detailed report of that emergency within 48 hr. to the manager of that ATC facility.
  • Answer (C) is incorrect because a written report may be requested when priority is given in an emergency, regardless of where the emergency occurs.

[255]

While on an IFR flight, a pilot has an emergency which causes a deviation from an ATC clearance. What action must be taken?
  1. Squawk 7700 for the duration of the emergency.
  2. Notify ATC of the deviation as soon as possible.
  3. Submit a detailed report to the chief of the ATC facility within 48 hours.
  • Answer (A) is incorrect because, in an emergency, you must report a deviation from an ATC clearance as soon as possible, not just squawk 7700 during the emergency.
  • Answer (B) is correct. Each pilot in command who, in an emergency, deviates from an ATC clearance or instruction shall notify ATC of that deviation as soon as possible.
  • Answer (C) is incorrect because a report in 48 hr. is required only if you are given priority during the emergency and ATC requests such a report.

[256]

During an IFR flight in IMC, a distress condition is encountered, (fire, mechanical, or structural failure). The pilot should
  1. wait until the situation is immediately perilous before declaring an emergency.
  2. not hesitate to declare an emergency and obtain an amended clearance.
  3. contact ATC and advise that an urgency condition exists and request priority consideration.
  • Answer (A) is incorrect because a distress condition is perilous and you should not hesitate to declare an emergency.
  • Answer (B) is correct. Distress is a condition of being threatened by serious and/or imminent danger and of requiring immediate assistance. Thus, during an IFR flight in IMC, if a distress condition is encountered, you should immediately declare an emergency and obtain an amended clearance.
  • Answer (C) is incorrect because you should contact ATC and declare that an emergency, not urgency, condition exists and obtain an amended clearance, not a request for consideration.

[257]

The aircraft's transponder fails during flight within Class D airspace.
  1. No deviation is required because a transponder is not required in Class D airspace.
  2. Pilot must immediately request priority handling to proceed to destination.
  3. The pilot should immediately request clearance to depart the Class D airspace.
  • Answer (A) is correct. If an aircraft's transponder fails during flight within Class D airspace, no deviation is required because a transponder is not required in Class D airspace.
  • Answer (B) is incorrect because, since a transponder is not required in Class D airspace, a pilot does not need priority handling to proceed to his/her destination.
  • Answer (C) is incorrect because, since a transponder is not required in Class D airspace, the pilot does not need to depart the Class D airspace.

[258]

In addition to a VOR receiver and two-way communications capability, which additional equipment is required for IFR operation in Class B airspace?
  1. An operable coded transponder having Mode C capability.
  2. DME and an operable coded transponder having Mode C capability.
  3. Standby communications receiver, DME, and coded transponder.
  • Answer (A) is correct. Unless otherwise authorized by ATC, no person may operate an aircraft within Class B airspace unless that aircraft is equipped with

1. An operable two-way radio,

2. An operable 4096-code transponder having Mode C capability, and

3. For IFR operations, an operable VOR or TACAN receiver.

  • Answer (B) is incorrect because DME is not required.
  • Answer (C) is incorrect because a standby radio receiver and DME are not required, and the transponder must have Mode C capability.
  • [259]

    Which of the following is required equipment for operating an aircraft within Class B airspace?
    1. A 4096 code transponder with automatic pressure altitude reporting equipment.
    2. A VOR receiver with DME.
    3. A 4096 code transponder.
    • Answer (A) is correct. Unless otherwise authorized by ATC, no person may operate an aircraft within Class B airspace unless that aircraft is equipped with

    1. An operable two-way radio,

    2. An operable 4096-code transponder having Mode C capability, and

    3. For IFR operations, an operable VOR or TACAN receiver.

  • Answer (B) is incorrect because DME is not required and a VOR receiver is only required when operating IFR.
  • Answer (C) is incorrect because the 4096-code trans ponder must also have Mode C capability.
  • [260]

    When are you required to have an instrument rating for flight in VMC?
    1. Flight into an ADIZ.
    2. Flight into Class A airspace.
    3. Flight through an MOA.
    • Answer (A) is incorrect because an instrument rating is not required for flight into an ADIZ (air defense identification zone) in VMC.
    • Answer (B) is correct. No person may operate an aircraft within Class A airspace at any time unless (s)he is rated for instrument flight and is on an instrument flight plan.
    • Answer (C) is incorrect because an instrument rating is not required for flight through an MOA in VMC.

    [261]

    When is an IFR clearance required during VFR weather conditions?
    1. When operating in airspace above 14,500 feet.
    2. When operating in a Class A airspace.
    3. When operating in the Class E airspace.
    • Answer (A) is incorrect because an IFR clearance is not required in VMC in Class E airspace from 14,500 ft. MSL up to but not including 18,000 ft. MSL.
    • Answer (B) is correct. No person may operate an aircraft within Class A airspace unless the aircraft is operated under an IFR clearance, regardless of the weather conditions. Class A airspace includes the airspace from 18,000 ft. MSL up to and including FL 600.
    • Answer (C) is incorrect because an IFR clearance is not required in VMC in Class E airspace.

    [262]

    When is an IFR flight plan required?
    1. When less than VFR conditions exist in either Class E or Class G airspace and in Class A airspace.
    2. In all Class E airspace when conditions are below VFR, in Class A airspace, and in defense zone airspace.
    3. In Class E airspace when IMC exists or in Class A airspace.
    • Answer (A) is incorrect because, while an instrument rating is required, an IFR flight plan is not required in Class G airspace.
    • Answer (B) is incorrect because VFR flights are permitted when VFR weather conditions exist in air defense identification zones (ADIZ).
    • Answer (C) is correct. No person may operate an aircraft in Class E airspace in IMC unless (s)he has filed an IFR flight plan and received an appropriate ATC clearance. Furthermore, under FAR 91.135, no one may operate in Class A airspace unless the aircraft is operated under IFR at a specific flight level assigned by ATC. This implies having filed an IFR flight plan for Class A airspace also.

    [263]

    Operation in which airspace requires filing an IFR flight plan?
    1. Class E airspace with IMC and Class A airspace.
    2. Any airspace when the visibility is less than 1 mile.
    3. Positive control area, Continental Control Area, and all other airspace, if the visibility is less than 1 mile.
    • Answer (A) is correct. No person may operate an aircraft in Class E airspace in IMC unless (s)he has filed an IFR flight plan and received an appropriate ATC clearance. Furthermore, under FAR 91.135, no one may operate in Class A airspace unless the aircraft is operated under IFR at a specific flight level assigned by ATC. This implies having filed an IFR flight plan for Class A airspace also.
    • Answer (B) is incorrect because an IFR flight plan is not required in Class G airspace.
    • Answer (C) is incorrect because an IFR flight plan is not required in uncontrolled Class G airspace.

    [264]

    What is the required flight visibility and distance from clouds if you are operating in Class E airspace at 9,500 feet MSL with a VFR-on-Top clearance during daylight hours?
    1. 3 SM, 1,000 feet above, 500 feet below, and 2,000 feet horizontal.
    2. 3 SM, 500 feet above, 1,000 feet below, and 2,000 feet horizontal.
    3. 5 SM, 500 feet above, 1,000 feet below, and 2,000 feet horizontal.
    • Answer (A) is correct. In Class E airspace below 10,000 ft. MSL, the basic VFR weather minimums are flight visibility of 3 SM and a distance from clouds of 500 ft. below, 1,000 ft. above, and 2,000 ft. horizontal.
    • Answer (B) is incorrect because the distances from clouds above and below are reversed. It should be 1,000 ft. above and 500 ft. below.
    • Answer (C) is incorrect because the visibility requirement is 3, not 5, SM and the distances from clouds above and below are reversed. It should be 1,000 ft. above and 500 ft. below.

    [265]

    What is the minimum flight visibility and distance from clouds for flight at 10,500 feet with a VFR-on-Top clearance during daylight hours? (Class E airspace.)
    1. 3 SM, 1,000 feet above, 500 feet below, and 2,000 feet horizontal.
    2. 5 SM, 1,000 feet above, 1,000 feet below, and 1 mile horizontal.
    3. 5 SM, 1,000 feet above, 500 feet below, and 1 mile horizontal.
    • Answer (A) is incorrect because 3 SM, 1,000 ft. above, 500 ft. below, and 2,000 ft. horizontal are the minimum flight visibility and distance from clouds in Class E airspace below, not at or above, 10,000 ft. MSL.
    • Answer (B) is correct. In Class E airspace at or above 10,000 ft. MSL, the basic VFR weather minimums are flight visibility of 5 SM and a distance from clouds of 1,000 ft. above or below and 1 SM horizontal.
    • Answer (C) is incorrect because the vertical separation from clouds is 1,000 ft. both above and below.

    [266]

    (Refer to figure 92.) What is the minimum in-flight visibility and distance from clouds required for an airplane operating less than 1,200 feet AGL during daylight hours in area 6?
    1. 1 mile; (I) clear of clouds; (K) clear of clouds; (L) clear of clouds.
    2. 1 mile; (I) 500 feet; (K) 1,000 feet; (L) 500 feet.
    3. 3 miles; (I) 1,000 feet; (K) 2,000 feet; (L) 500 feet.
    When an aircraft is decelerated some attitude indicators will precess and incorrectly indicate a group of answer choices?

    Figure 92
    • Answer (A) is correct. In Class G airspace at or below 1,200 ft. AGL (Area 6 in Fig. 92), the basic VFR weather minimums during daylight hours are in-flight visibility of 1 SM and clear of clouds.
    • Answer (B) is incorrect because no such combination of requirements exists in any airspace.
    • Answer (C) is incorrect because 3 SM visibility, 1,000 ft. above, 500 ft. below, and 2,000 ft. horizontal are the minimum visibility and distance from clouds in VFR flight in Area 6 at night, not in daylight.

    [267]

    (Refer to figure 92.) What is the minimum in-flight visibility and distance from clouds required in VFR conditions above clouds at 13,500 feet MSL (above 1,200 feet AGL) in Class G airspace during daylight hours for area 2?
    1. 5 miles; (A) 1,000 feet; (C) 2,000 feet; (D) 500 feet.
    2. 3 miles; (A) 1,000 feet; (C) 1 mile; (D) 1,000 feet.
    3. 5 miles; (A) 1,000 feet; (C) 1 mile; (D) 1,000 feet.
    When an aircraft is decelerated some attitude indicators will precess and incorrectly indicate a group of answer choices?

    Figure 92
    • Answer (A) is incorrect because 1,000 ft. above, 2,000 ft. horizontal, and 500 ft. below are the minimum cloud distances for VFR in Class G airspace above 1,200 ft. AGL and below, not at or above, 10,000 ft. MSL.
    • Answer (B) is incorrect because visibility minimum is 5 SM, not 3 SM.
    • Answer (C) is correct. In Class G airspace at more than 1,200 ft. AGL and at or above 10,000 ft. MSL (Area 2 in Fig. 92), the basic VFR weather minimums are in-flight visibility of 5 SM and a distance from clouds of 1,000 ft. above or below and 1 SM horizontal.

    [268]

    (Refer to figure 92.) What in-flight visibility and distance from clouds is required for a flight at 8,500 feet MSL (above 1,200 feet AGL) in Class G airspace in VFR conditions during daylight hours in area 4?
    1. 3 miles; (E) 1,000 feet; (G) 2,000 feet; (H) 500 feet.
    2. 5 miles; (E) 1,000 feet; (G) 1 mile; (H) 1,000 feet.
    3. 1 mile; (E) 1,000 feet; (G) 2,000 feet; (H) 500 feet.
    When an aircraft is decelerated some attitude indicators will precess and incorrectly indicate a group of answer choices?

    Figure 92
    • Answer (A) is incorrect because the in-flight visibility of 3 SM is required for a night, not day, flight.
    • Answer (B) is incorrect because in-flight visibility of 5 SM and a distance from clouds of 1,000 ft. above or below and 1 SM horizontal are the VFR weather minimums in Class G airspace above 1,200 ft. AGL and at or above, not below, 10,000 ft. MSL.
    • Answer (C) is correct. In Class G airspace at more than 1,200 ft. AGL but less than 10,000 ft. MSL (Area 4 in Fig. 92), the basic VFR weather minimums during daylight hours are in-flight visibility of 1 SM and a distance from clouds of 500 ft. below, 1,000 ft. above, and 2,000 ft. horizontal.

    [269]

    (Refer to figure 92.) What is the minimum in-flight visibility and distance from clouds required for a VFR-on-Top flight at 9,500 feet MSL (above 1,200 feet AGL) during daylight hours for Area 3?
    1. 2,000 feet; (E) 1,000 feet; (F) 2,000 feet; (H) 500 feet.
    2. 3 miles; (E) 1,000 feet; (F) 2,000 feet; (H) 500 feet.
    3. 5 miles; (E) 1,000 feet; (F) 2,000 feet; (H) 500 feet.
    When an aircraft is decelerated some attitude indicators will precess and incorrectly indicate a group of answer choices?

    Figure 92
    • Answer (A) is incorrect because the visibility required is 3 SM, not 2,000 ft.
    • Answer (B) is correct. In Class E airspace at less than 10,000 ft. MSL (Area 3 in Fig. 92), the basic VFR weather minimums are in-flight visibility of 3 SM and a distance from clouds of 500 ft. below, 1,000 ft. above, and 2,000 ft. horizontal.
    • Answer (C) is incorrect because the visibility required is 3 SM, not 5 SM.

    [270]

    (Refer to figure 92.) A flight is to be conducted in VFR-on-Top conditions at 12,500 feet MSL (above 1,200 feet AGL). What is the in-flight visibility and distance from clouds required for operation in Class E airspace during daylight hours for Area 1?
    1. 3 miles; (A) 1,000 feet; (B) 2,000 feet; (D) 1,000 feet.
    2. 5 miles; (A) 1,000 feet; (B) 2,000 feet; (D) 500 feet.
    3. 5 miles; (A) 1,000 feet; (B) 1 mile; (D) 1,000 feet.
    When an aircraft is decelerated some attitude indicators will precess and incorrectly indicate a group of answer choices?

    Figure 92
    • Answer (A) is incorrect because the visibility requirement is 5 SM, not 3 SM, and the horizontal separation requirement from clouds is 1 SM, not 2,000 ft.
    • Answer (B) is incorrect because the distance-from-clouds requirements listed are for below, not at or above, 10,000 ft. MSL.
    • Answer (C) is correct. In Class E airspace at or above 10,000 ft. MSL (Area 1 in Fig. 92), the basic VFR weather minimums are in-flight visibility of 5 SM and a distance from clouds of 1,000 ft. above or below and 1 SM horizontal.

    [271]

    (Refer to figure 92.) What is the minimum in-flight visibility and distance from clouds required for an airplane operating less than 1,200 feet AGL under special VFR during daylight hours in area 5?
    1. 1 mile; (I) 2,000 feet; (J) 2,000 feet; (L) 500 feet.
    2. 3 miles; (I) clear of clouds; (J) clear of clouds; (L) 500 feet.
    3. 1 mile; (I) clear of clouds; (J) clear of clouds; (L) clear of clouds.
    When an aircraft is decelerated some attitude indicators will precess and incorrectly indicate a group of answer choices?

    Figure 92
    • Answer (A) is incorrect because special VFR permits operation just clear of clouds.
    • Answer (B) is incorrect because special VFR permits operation just clear of clouds and with a minimum visibility of 1 SM, not 3 SM.
    • Answer (C) is correct. In Class E airspace when an airplane is operating under special VFR, the distance-from-clouds requirement is clear of clouds. No one may take off or land an airplane under special VFR unless ground visibility is at least 1 SM. If ground visibility is not reported, the in-flight visibility during takeoff or landing must be at least 1 SM.

    [272]

    What are the minimum fuel requirements in IFR conditions, if the first airport of intended landing is forecast to have a 1,500-foot ceiling and 3 miles visibility at flight-planned ETA? Fuel to fly to the first airport of intended landing,
    1. fly to the alternate, and fly thereafter for 45 minutes at normal cruising speed.
    2. and fly thereafter for 45 minutes at normal cruising speed.
    3. fly to the alternate, and fly thereafter for 30 minutes at normal cruising speed.
    • Answer (A) is correct. In general, no person may operate a civil aircraft in IFR conditions unless it carries enough fuel (considering weather reports, forecasts, and conditions) to complete the flight to the first airport of intended landing, fly from that airport to the alternate airport, and fly after that for 45 min. at normal cruising speed.
    • Answer (B) is incorrect because an alternate airport is required because the destination airport, from 1 hr. before to 1 hr. after ETA, has a forecast ceiling of less than 2,000 ft. AGL.
    • Answer (C) is incorrect because the fuel requirement after the alternate is 45 min., not 30 min.

    [273]

    During your preflight planning for an IFR flight, you determine that the first airport of intended landing has no instrument approach prescribed in 14 CFR part 97. The weather forecast for one hour before through one hour after your estimated time of arrival is 3,000 ft. scattered with 5 miles visibility. To meet the fuel requirements for this flight, you must be able to fly to the first airport of intended landing,
    1. and then fly for 45 minutes at normal cruising speed.
    2. then to the alternate airport, and then for 45 minutes at normal cruising speed.
    3. then to the alternate airport, and then for 30 minutes at normal cruising speed.
    • Answer (A) is incorrect because you are required to file an alternate airport and meet the applicable fuel requirements because your destination airport has no IAP.
    • Answer (B) is correct. You may not operate an aircraft (other than a helicopter) under IFR unless it carries sufficient fuel to fly to the first airport of intended landing, fly from that airport to the alternate airport, and fly thereafter for 45 minutes at normal cruising speed. An alternate airport is not required if the forecast weather conditions from one hour before to one hour after your estimated time of arrival call for a 2,000-ft. ceiling and 3 SM visibility, and your destination airport has at least one approved instrument approach procedure (IAP). Because your destination airport (first airport of intended landing) has no IAP, you must file an alternate airport and meet the applicable fuel requirements.
    • Answer (C) is incorrect because you would be required to carry fuel sufficient for only 30 minutes at normal cruising speed after flying to the alternate airport if you were flying a helicopter, not an airplane. Airplanes must be able to fly for 45 minutes at normal cruising speed.

    [274]

    For aircraft other than helicopters, is an alternate airport required for an IFR flight to ATL (Atlanta Hartsfield) if the proposed ETA is 1930Z?  

    TAF KATL 121720Z 121818 20012KT 5SM HZ BKN030 FM2000 3SM TSRA OVC025CB FM2200 33015G20KT P6SM BKN015 OVC040 BECMG 0608 02008KT BKN040 BECMG 1012 00000KT P6SM

    CLR=

    1. No, because the ceiling and visibility are forecast to remain at or above 1,000 feet and 3 miles, respectively.
    2. Yes, because the ceiling could fall below 2,000 feet within 2 hours before to 2 hours after the ETA.
    3. No, because the ceiling and visibility are forecast to be at or above 2,000 feet and 3 miles within 1 hour before to 1 hour after the ETA.
    • Answer (A) is incorrect because the ceiling must remain at least 2,000 ft., not 1,000 ft., for an alternate not to be required.
    • Answer (B) is incorrect because the time frame of concern is 1 hr., not 2 hr., before and after the ETA.
    • Answer (C) is correct. Since the ETA is 1930Z, you must check the forecast from 1830Z to 2030Z. In the TAF given, you will use the first forecast period (which is valid from 1800 to 2000Z) and the second forecast period (which is valid from 2000 to 2200Z). The lowest visibility and ceiling are forecast from 2000 to 2200Z. The visibility is 3 SM, and the ceiling is 2,500 ft. Thus, no alternate airport is required because the ceiling and visibility are forecast to be at or above 2,000 ft. and 3 SM within 1 hr. before to 1 hr. after the ETA.

    [275]

    When a pilot elects to proceed to the selected alternate airport, which minimums apply for landing at the alternate?
    1. Ceiling 200 feet above the published minimum; visibility 2 miles.
    2. The landing minimums for the approach to be used.
    3. 600-1 if the airport has an ILS.
    • Answer (A) is incorrect because the published approach minimums, not some adjustment thereof, should be used.
    • Answer (B) is correct. When one goes to an alternate airport to land, the landing minimums for the particular approach, not the minimums for listing the airport as an alternate, are the minimums to be used for the approach.
    • Answer (C) is incorrect because, to be listed on the flight plan as an alternate airport, the weather conditions at the estimated time of arrival at the alternate must be a 600-ft. ceiling and 2 SM, not 1 SM, visibility.

    [276]

    If a pilot elects to proceed to the selected alternate, the landing minimums used at that airport should be the
    1. alternate minimums shown on the approach chart.
    2. minimums shown for that airport in a separate listing of "IFR Alternate Minimums."
    3. minimums specified for the approach procedure selected.
    • Answer (A) is incorrect because alternate minimums shown on the approach chart refer to the weather conditions required to list that airport as an alternate on your IFR flight plan, not to land there.
    • Answer (B) is incorrect because alternate minimums shown on the approach chart refer to the weather conditions required to list that airport as an alternate on your IFR flight plan, not to land there.
    • Answer (C) is correct. When one goes to an alternate airport to land, the landing minimums for the particular approach, not the minimums for listing the airport as an alternate, are the approach minimums to be used for the approach.

    [277]

    What minimum weather conditions must be forecast for your ETA at an alternate airport, that has only a VOR approach with standard alternate minimums, for the airport to be listed as an alternate on the IFR flight plan?
    1. 1,000-foot ceiling and visibility to allow descent from minimum en route altitude (MEA), approach, and landing under basic VFR.
    2. 800-foot ceiling and 1 statute mile visibility.
    3. 800-foot ceiling and 2 statute miles visibility.
    • Answer (A) is incorrect because, if no instrument approach procedure is available at an airport, the ceiling and visibility minimums are those allowing descent from the MEA, approach, and landing under basic VFR.
    • Answer (B) is incorrect because the visibility requirement is 2 SM, not 1 SM.
    • Answer (C) is correct. Unless otherwise authorized, no one may include an alternate airport with only a nonprecision approach in an IFR flight plan unless current weather forecasts indicate that, at the ETA at the alternate airport, the ceiling will be at least 800 ft. and 2 SM visibility.

    [278]

    For aircraft other than helicopters, what minimum conditions must exist at the destination airport to avoid listing an alternate airport on an IFR flight plan when a standard IAP is available?
    1. From 2 hours before to 2 hours after ETA, forecast ceiling 2,000, and visibility 2 and ½ miles.
    2. From 1 hour before to 1 hour after ETA, forecast ceiling 2,000, and visibility 3 miles.
    3. From 2 hours before to 2 hours after ETA, forecast ceiling 3,000, and visibility 3 miles.
    • Answer (A) is incorrect because the destination weather condition forecast is from 1 hr., not 2 hr., before and after ETA, and the visibility must be at least 3 SM, not 2½ SM.
    • Answer (B) is correct. An alternate airport is not required to be listed on an IFR flight plan if the destination airport has a standard instrument approach procedure available and, for at least 1 hr. before and 1 hr. after the estimated time of arrival, the weather reports or forecasts, or any combination of them, indicate

    1. The ceiling will be at least 2,000 ft. above the airport elevation; and

    2. The visibility will be at least 3 SM.

  • Answer (C) is incorrect because the destination weather condition forecast is from 1 hr., not 2 hr., before and after ETA, and the ceiling must be at least 2,000 ft. AGL, not 3,000 ft. AGL.
  • [279]

    What are the minimum weather conditions that must be forecast to list an airport as an alternate when the airport has no approved IAP?
    1. The ceiling and visibility from 2 hours before until 2 hours after ETA, 2,000 feet and 3 miles, respectively.
    2. The ceiling and visibility at ETA, 2,000 feet and 3 miles, respectively.
    3. The ceiling and visibility at ETA must allow descent from MEA, approach, and landing, under basic VFR.
    • Answer (A) is incorrect because an alternate must be listed on your IFR flight plan, unless the weather at your destination is forecast, from 1 hr., not 2 hr., before to 1 hr. after ETA, to have at least a 2,000-ft. ceiling and visibility of 3 SM.
    • Answer (B) is incorrect because an alternate must be listed on your IFR flight plan unless the weather at your destination is forecast, from 1 hr. before to 1 hr. after ETA, to have a ceiling of 2,000 ft. and visibility of 3 SM.
    • Answer (C) is correct. Unless otherwise authorized, no one may include an alternate airport that has no instrument approach in an IFR flight plan unless current weather forecasts indicate that, at the ETA at the alternate airport, the ceiling and visibility will allow descent from the MEA, approach, and landing under basic VFR.

    [280]

    What standard minimums are required to list an airport as an alternate on an IFR flight plan if the airport has VOR approach only?
    1. Ceiling and visibility at ETA, 600 feet and 2 miles, respectively.
    2. Ceiling and visibility from 2 hours before until 2 hours after ETA, 800 feet and 2 miles, respectively.
    3. Ceiling and visibility at ETA, 800 feet and 2 miles, respectively.
    • Answer (A) is incorrect because 600 ft. and 2 SM are the alternate airport weather minimums for a precision approach, i.e., ILS.
    • Answer (B) is incorrect because the alternate airport weather minimums apply to the ETA, not 2 hr. plus or minus.
    • Answer (C) is correct. Unless otherwise authorized, no person may include an alternate airport that has only a VOR (i.e., nonprecision) approach in an IFR flight plan unless current weather forecasts indicate that at the ETA at the alternate airport the ceiling is at least 800 ft. and visibility is 2 SM.

    [281]

    For aircraft other than helicopters, what minimum weather conditions must be forecast for your ETA at an alternate airport that has a precision approach procedure, with standard alternate minimums, in order to list it as an alternate for the IFR flight?
    1. 600-foot ceiling and 2 SM visibility from 2 hours before to 2 hours after your ETA.
    2. 800-foot ceiling and 2 SM visibility at your ETA.
    3. 600-foot ceiling and 2 SM visibility at your ETA.
    • Answer (A) is incorrect because the alternate airport weather minimums apply to the ETA, not 2 hr. plus or minus.
    • Answer (B) is incorrect because 800 ft. and 2 SM are the alternate airport minimums for a nonprecision approach.
    • Answer (C) is correct. Unless otherwise authorized, no person may include an alternate airport that has a precision (ILS) approach in an IFR flight plan unless current weather forecasts indicate that at the ETA at the alternate airport the ceiling is at least 600 ft. and visibility is 2 SM.

    [282]

    What are the alternate minimums that must be forecast at the ETA for an airport that has a precision approach procedure?
    1. 800-foot ceiling and 2 miles visibility.
    2. 600-foot ceiling and 2 miles visibility.
    3. 400-foot ceiling and 2 miles visibility.
    • Answer (A) is incorrect because 800 ft. and 2 SM are the alternate airport minimums for nonprecision approaches.
    • Answer (B) is correct. Unless otherwise authorized, no person may include an alternate airport that has a precision (ILS) approach in an IFR flight plan unless current weather forecasts indicate that at the ETA at the alternate airport the ceiling is at least 600 ft. and visibility is 2 SM.
    • Answer (C) is incorrect because 400 ft. is not a standard minimum ceiling used as an alternative airport minimum.

    [283]

    An airport without an authorized IAP may be included on an IFR flight plan as an alternate, if the current weather forecast indicates that the ceiling and visibility at the ETA will
    1. allow for a descent from the MEA, approach, and a landing under basic VFR conditions.
    2. allow for descent from the IAF to landing under basic VFR conditions.
    3. be at least 1,000 feet and 1 mile.
    • Answer (A) is correct. Unless otherwise authorized, no person may include an alternate airport that does not have a standard instrument approach on an IFR flight plan unless current weather forecasts indicate that at the ETA at the alternate airport the ceiling and visibility will allow for a descent from the MEA, approach, and landing under basic VFR conditions.
    • Answer (B) is incorrect because descent must be possible from the MEA, not the IAF (initial approach fix), under basic VFR conditions.
    • Answer (C) is incorrect because 800 ft., not 1,000 ft., and 2 SM, not 1 SM, are the standard alternate airport weather minimums at ETA for a nonprecision approach procedure.

    [284]

    When an alternate airport is required, what are the weather minimums that must be forecast at the ETA for an alternate airport that has a precision approach procedure?
    1. Ceiling 200 feet above field elevation and visibility 1 statute mile, but not less than the minimum visibility for the approach.
    2. 600 foot ceiling and 2 statute miles visibility.
    3. Ceiling 200 feet above the approach minimums and at least 1 statute mile visibility, but not less than the minimum visibility for the approach.
    • Answer (A) is incorrect because 600 ft., not 200 ft., and 2 SM, not 1 SM, are the alternate minimums for airports with precision approaches.
    • Answer (B) is correct. Unless otherwise authorized, no person may include an alternate airport that has a precision (ILS) approach in an IFR flight plan unless current weather forecasts indicate that at the ETA at the alternate airport the ceiling is at least 600 ft. and visibility is 2 SM.
    • Answer (C) is incorrect because 600 ft., not 200 ft. above minimums, is the minimum ceiling; and 2 SM, not 1 SM, is the minimum visibility required to list an airport with a precision approach as an alternate.

    [285]

    What record shall be made in the aircraft log or other permanent record by the pilot making the VOR operational check?
    1. The date, place, bearing error, aircraft total time, and signature.
    2. The date, frequency of VOR or VOT, number of flight hours since last check, and signature.
    3. The date, place, bearing error, and signature.
    • Answer (A) is incorrect because the aircraft's total time is not required.
    • Answer (B) is incorrect because VOR frequency and number of flight hr. since last check are not required.
    • Answer (C) is correct. Each person making the VOR operational check shall enter the date, place, and bearing error and sign the aircraft log or other record.

    [286]

    When must an operational check on the aircraft VOR equipment be accomplished when used to operate under IFR?
    1. Within the preceding 10 days or 10 hours of flight time.
    2. Within the preceding 30 days or 30 hours of flight time.
    3. Within the preceding 30 days.
    • Answer (A) is incorrect because it must be checked every 30 days, not 10 days or 10 hr.
    • Answer (B) is incorrect because there is no time requirement regarding hours of flight time.
    • Answer (C) is correct. No person may operate a civil aircraft under IFR using the VOR system of radio navigation unless the VOR equipment of that aircraft is maintained, checked, and inspected under an approved procedure, or has been operationally checked within the preceding 30 days and was found to be within the limits of the permissible indicated bearing error.

    [287]

    Which data must be recorded in the aircraft log or other appropriate log by a pilot making a VOR operational check for IFR operations?
    1. VOR name or identification, date of check, amount of bearing error, and signature.
    2. Date of check, VOR name or identification, place of operational check, and amount of bearing error.
    3. Place of operational check, amount of bearing error, date of check, and signature.
    • Answer (A) is incorrect because the place of operational check rather than the VOR name or identification is required.
    • Answer (B) is incorrect because a signature is required, but the VOR name is not required.
    • Answer (C) is correct. Each person making the VOR operational check shall enter the date, place, and bearing error and sign the aircraft log or other record.

    [288]

    When making an airborne VOR check, what is the maximum allowable tolerance between the two indicators of a dual VOR system (units independent of each other except the antenna)?
    1. 6° between the two indicated radials of a VOR.
    2. Plus or minus 4° when set to identical radials of a VOR.
    3. 4° between the two indicated bearings of a VOR.
    • Answer (A) is incorrect because the maximum allowable difference between the two VORs is 4°, not 6°, of the indicated radial.
    • Answer (B) is incorrect because you center the CDI and note the bearing, not set both VORs to the same radial.
    • Answer (C) is correct. If a dual VOR system (units independent of each other except the antenna) is installed in the aircraft, you may check one system against the other. You tune both systems to the same VOR station and note the indicated bearings to that station. The maximum permissible variation between the two indicated bearings is 4°.

    [289]

    Which checks and inspections of flight instruments or instrument systems must be accomplished before an aircraft can be flown under IFR?
    1. VOR within 30 days, altimeter systems within 24 calendar months, and transponder within 24 calendar months.
    2. VOR within 24 calendar months, transponder within 24 calendar months, and altimeter system within 12 calendar months.
    3. ELT test within 30 days, altimeter systems within 12 calendar months, and transponder within 24 calendar months.
    • Answer (A) is correct. No person may operate a civil aircraft under IFR using the VOR system of radio navigation unless the VOR equipment of that aircraft is maintained, checked, and inspected under an approved procedure, or has been operationally checked within the preceding 30 days and was found to be within the limits of the permissible indicated bearing error. Also, within the preceding 24 calendar months, each altimeter system and transponder must be tested, inspected, and found to comply with the regulations.
    • Answer (B) is incorrect because VORs must be checked within 30 days, not 24 months, and altimeter systems must be inspected within 24 months, not 12 months.
    • Answer (C) is incorrect because check and inspection of the altimeter system is required every 24 months, not 12 months, and ELTs must be inspected every 12 months, not tested within 30 days.

    [290]

    What is the maximum tolerance allowed for an operational VOR equipment check when using a VOT?
    1. Plus or minus 6°.
    2. Plus or minus 8°.
    3. Plus or minus 4°.
    • Answer (A) is incorrect because plus or minus 6° is the maximum error allowed when using an airborne checkpoint.
    • Answer (B) is incorrect because plus or minus 8° is not an acceptable error for any type of VOR equipment check.
    • Answer (C) is correct. When using a VOT for an operational VOR equipment check, the maximum permissible indicated bearing error is plus or minus 4°.

    [291]

    When departing from an airport located outside controlled airspace during IMC, you must file an IFR flight plan and receive a clearance before
    1. takeoff.
    2. entering Class E airspace.
    3. entering IFR conditions.
    • Answer (A) is incorrect because an IFR flight plan and clearance are not required until you enter controlled airspace.
    • Answer (B) is correct. No person may operate an aircraft in controlled airspace under IFR unless (s)he has filed an IFR flight plan and received an appropriate ATC clearance.
    • Answer (C) is incorrect because an IFR flight plan and clearance are not required until you enter controlled airspace.

    [292]

    To operate an aircraft under IFR, a flight plan must have been filed and an ATC clearance received prior to
    1. entering controlled airspace.
    2. controlling the aircraft solely by use of instruments.
    3. entering weather conditions in any airspace.
    • Answer (A) is correct. No person may operate an aircraft in controlled airspace under IFR unless (s)he has filed an IFR flight plan and received an appropriate ATC clearance.
    • Answer (B) is incorrect because an IFR flight plan and clearance are not required until you enter controlled airspace.
    • Answer (C) is incorrect because an IFR flight plan and clearance are not required until you enter controlled airspace.

    [293]

    To operate under IFR below 18,000 feet, a pilot must file an IFR flight plan and receive an appropriate ATC clearance prior to
    1. takeoff.
    2. entering controlled airspace.
    3. entering weather conditions below VFR minimums.
    • Answer (A) is incorrect because an IFR flight plan and clearance are not required until you enter controlled airspace.
    • Answer (B) is correct. No person may operate an aircraft in controlled airspace under IFR unless (s)he has filed an IFR flight plan and received an appropriate ATC clearance.
    • Answer (C) is incorrect because an IFR flight plan and clearance are not required until you enter controlled airspace.

    [294]

    Prior to which operation must an IFR flight plan be filed and an appropriate ATC clearance received?
    1. Flying by reference to instruments in controlled airspace.
    2. Takeoff when IFR weather conditions exist.
    3. Entering controlled airspace when IMC exists.
    • Answer (A) is incorrect because you may fly by reference to instruments with a safety pilot in VFR weather conditions without an IFR flight plan or an IFR clearance.
    • Answer (B) is incorrect because an IFR flight plan is not required until you enter controlled airspace.
    • Answer (C) is correct. No person may operate an aircraft in controlled airspace under IFR unless (s)he has filed an IFR flight plan and received an appropriate ATC clearance.

    [295]

    No person may operate an aircraft in controlled airspace under IFR unless he/she files a flight plan
    1. prior to takeoff and requests the clearance upon arrival on an airway.
    2. and receives a clearance prior to entering controlled airspace.
    3. and receives a clearance by telephone prior to takeoff.
    • Answer (A) is incorrect because a person must file an IFR flight plan and receive clearance before operating in controlled airspace, e.g., a federal airway.
    • Answer (B) is correct. No person may operate an aircraft in controlled airspace under IFR unless (s)he has filed an IFR flight plan and received an appropriate ATC clearance.
    • Answer (C) is incorrect because it does not matter how the clearance is obtained.

    [296]

    Except when necessary for takeoff or landing or unless otherwise authorized by the Administrator, the minimum altitude for IFR flight is
    1. 3,000 feet over designated mountainous terrain; 2,000 feet over terrain elsewhere.
    2. 2,000 feet above the highest obstacle over designated mountainous terrain; 1,000 feet above the highest obstacle over terrain elsewhere.
    3. 3,000 feet over all terrain.
    • Answer (A) is incorrect because the minimum IFR altitude is 2,000 ft., not 3,000 ft., above the highest obstacle over mountainous terrain, or 1,000 ft., not 2,000 ft., above the highest obstacle over terrain elsewhere.
    • Answer (B) is correct. Except when necessary for takeoff or landing, no person may operate an aircraft under IFR below 2,000 ft. above the highest obstacle within a horizontal distance of 4 NM from the course to be flown over designated mountainous terrain, or 1,000 ft. above the highest obstacle within a horizontal distance of 4 NM from the course to be flown over terrain elsewhere.
    • Answer (C) is incorrect because the minimum IFR altitude is 2,000 ft. above the highest obstacle over mountainous terrain or 1,000 ft. over the highest obstacle over terrain elsewhere, not 3,000 ft. over all terrain.

    [297]

    What minimum navigation equipment is required for IFR flight?
    1. VOR receiver and, if in ARTS III environment, a coded transponder equipped for altitude reporting.
    2. Navigation equipment appropriate to the ground facilities to be used.
    3. VOR/LOC receiver, transponder, and DME.
    • Answer (A) is incorrect because a VOR is required only if using VOR stations for navigation. If using NDB stations, a VOR is not required. A transponder is not a navigation system.
    • Answer (B) is correct. The minimum navigation equipment requirement for IFR flight is navigation equipment that is appropriate to the ground facilities to be used.
    • Answer (C) is incorrect because a VOR/LOC receiver and DME are required only if VORTAC stations will be used for navigation and DME fixes need to be identified. If you are using alternative means of navigation (e.g., LORAN), this equipment is not needed. A transponder is not a navigation system.

    [298]

    Where is DME required under IFR?
    1. Above 18,000 feet MSL.
    2. At or above 24,000 feet MSL if VOR navigational equipment is required.
    3. In positive control airspace.
    • Answer (A) is incorrect because Class A airspace begins at 18,000 ft. MSL and DME is required at or above FL 240, if VOR navigational equipment is required.
    • Answer (B) is correct. If VOR navigational equipment is required, no person may operate a U.S.-registered civil aircraft within the 50 states and the District of Columbia, at or above 24,000 ft. MSL (FL 240), unless that aircraft is equipped with approved distance measuring equipment (DME).
    • Answer (C) is incorrect because, if VOR navigational equipment is required, DME is required at or above FL 240, not only in positive control airspace. Positive control airspace is that airspace in which ATC separates all aircraft, e.g., Class A, Class B, Class C.

    [299]

    An aircraft operated during IFR under 14 CFR Part 91 is required to have which of the following?
    1. Radar altimeter.
    2. Gyroscopic direction indicator.
    3. Dual VOR system.
    • Answer (A) is incorrect because only a sensitive altimeter, not a radar altimeter, is required.
    • Answer (B) is correct. An aircraft operated during IFR under FAR Part 91 is required to have a gyroscopic direction indicator (directional gyro or equivalent).
    • Answer (C) is incorrect because, if VOR navigation is to be used, only one, not two, VOR is required under FAR Part 91.

    [300]

    Aircraft being operated under IFR are required to have, in addition to the equipment required for VFR and night, at least
    1. distance measuring equipment.
    2. dual VOR receivers.
    3. a slip skid indicator.
    • Answer (A) is incorrect because DME is required only above 24,000 ft. MSL when VOR navigational equipment is required.
    • Answer (B) is incorrect because the requirement is for navigational equipment appropriate to the ground facilities to be used, not necessarily dual VOR receivers.
    • Answer (C) is correct. An aircraft operated under FAR Part 91 under IFR is required to have a slip-skid indicator (e.g., the ball of the turn coordinator).

    [301]

    To meet the requirements for flight under IFR, an aircraft must be equipped with certain operable instruments and equipment. One of those required is
    1. a clock with sweep second pointer or digital presentation.
    2. a radar altimeter.
    3. a transponder with altitude reporting capability.
    • Answer (A) is correct. An aircraft operated under FAR Part 91 under IFR is required to have a clock displaying hours, minutes, and seconds with a sweep-second pointer or digital presentation.
    • Answer (B) is incorrect because a radio (radar) altimeter is required under some circumstances for Category II operations with decision heights below 150 ft. AGL, not for all flights under IFR.
    • Answer (C) is incorrect because a transponder with altitude encoding capability is required in certain airspace areas, not for all flights under IFR.

    [302]

    What is the oxygen requirement for an unpressurized aircraft at 15,000 feet?
    1. Crew must use oxygen for the entire time above 14,000 feet and passengers must be provided supplemental oxygen only above 15,000 feet.
    2. Crew must start using oxygen at 12,000 feet and passengers at 15,000 feet.
    3. All occupants must use oxygen for the entire time at this altitude.
    • Answer (A) is correct. No one may operate a U.S. civil aircraft at cabin pressure altitudes above 14,000 ft. MSL unless the required minimum flight crew is provided with and uses supplemental oxygen during the entire flight time at those altitudes. At cabin pressure altitudes above 15,000 ft. MSL, each passenger must be provided with supplemental oxygen.
    • Answer (B) is incorrect because the crew must start using oxygen at 14,000 ft. MSL or after 30 min. above 12,500 ft. MSL.
    • Answer (C) is incorrect because the required minimum flight crew must use oxygen above 14,000 ft. MSL and others must be provided with oxygen above 15,000 ft. MSL.

    [303]

    What is the maximum IFR altitude you may fly in an unpressurized aircraft without providing passengers with supplemental oxygen?
    1. 14,000 feet.
    2. 15,000 feet.
    3. 12,500 feet.
    • Answer (A) is incorrect because, at cabin pressure altitudes above 14,000 ft. MSL, only the required minimum flight crew must use supplemental oxygen.
    • Answer (B) is correct. At cabin pressure altitudes above 15,000 ft. MSL, each occupant must be provided with supplemental oxygen.
    • Answer (C) is incorrect because, at cabin pressure altitudes above 12,500 ft. MSL up to and including 14,000 ft. MSL, only the minimum flight crew must use supplemental oxygen for that part of the flight at those altitudes that is more than 30 min. duration.

    [304]

    What is the maximum cabin pressure altitude at which a pilot can fly for longer than 30 minutes without using supplemental oxygen?
    1. 12,000 feet.
    2. 10,500 feet.
    3. 12,500 feet.
    • Answer (A) is incorrect because supplemental oxygen is not required at any time at 12,000 ft.
    • Answer (B) is incorrect because supplemental oxygen is not required at any time at 10,500 ft.
    • Answer (C) is correct. No one may operate a U.S. civil aircraft at cabin pressure altitudes above 12,500 ft. MSL up to and including 14,000 ft. MSL unless the required minimum flight crew uses supplemental oxygen for that part of the flight at those altitudes that is of more than 30 min. duration.

    [305]

    If an unpressurized aircraft is operated above 12,500 feet MSL, but not more than 14,000 feet MSL, for a period of 2 hours 20 minutes, how long during that time is the minimum flightcrew required to use supplemental oxygen?
    1. 2 hours 20 minutes.
    2. 1 hour 20 minutes.
    3. 1 hour 50 minutes.
    • Answer (A) is incorrect because one may fly for 30 min. without supplemental oxygen between 12,500 ft. MSL and 14,000 ft. MSL.
    • Answer (B) is incorrect because 30 min. of flight, not 1 hr., is permitted without supplemental oxygen between 12,500 ft. MSL up to and including 14,000 ft. MSL.
    • Answer (C) is correct. No one may operate a U.S. civil aircraft at cabin pressure altitudes above 12,500 ft. MSL up to and including 14,000 ft. MSL unless the required minimum flight crew uses supplemental oxygen for that part of the flight at those altitudes that is of more than 30 min. duration. If the flight lasts 2 hr. and 20 min., the crew must use supplemental oxygen for all but 30 min., or 1 hr. and 50 min.

    [306]

    In the 48 contiguous states, excluding the airspace at or below 2,500 feet AGL, an operable coded transponder equipped with Mode C capability is required in all controlled airspace at and above
    1. 10,000 feet MSL.
    2. 12,500 feet MSL.
    3. Flight level (FL) 180.
    • Answer (A) is correct. Unless otherwise authorized or directed by ATC, no person may operate an aircraft in the 48 contiguous states at and above 10,000 ft. MSL, excluding the airspace at or below 2,500 ft. AGL, unless the aircraft is equipped with an operable Mode C transponder.
    • Answer (B) is incorrect because 12,500 ft. MSL pertains to supplemental oxygen, not Mode C, requirements.
    • Answer (C) is incorrect because FL 180 is the floor of Class A airspace.

    [307]

    A coded transponder equipped with altitude reporting capability is required in all controlled airspace
    1. at and above 10,000 feet MSL, excluding at and below 2,500 feet AGL.
    2. below 10,000 feet MSL, excluding at and below 2,500 feet AGL.
    3. at and above 2,500 feet above the surface.
    • Answer (A) is correct. Unless otherwise authorized or directed by ATC, no person may operate an aircraft in the 48 contiguous states at and above 10,000 ft. MSL, excluding the airspace at or below 2,500 ft. AGL, unless the aircraft is equipped with an operable Mode C transponder.
    • Answer (B) is incorrect because the limit is at and above, not below, 10,000 ft. MSL.
    • Answer (C) is incorrect because the airspace above 2,500 ft. AGL must also be at or above 10,000 ft. MSL.

    [308]

    Prior to operating an aircraft not equipped with a transponder in Class B airspace, a request for a deviation must be submitted to the
    1. FAA Administrator at least 24 hours before the proposed operation.
    2. nearest FAA General Aviation District Office 24 hours before the proposed operation.
    3. controlling ATC facility at least 1 hour before the proposed flight.
    • Answer (A) is incorrect because a request for a deviation to operate in Class B airspace in an airplane not equipped with a transponder must be submitted to the controlling ATC facility at least 1 hr. before the proposed flight, not to the FAA Administrator at least 24 hr. before the operation.
    • Answer (B) is incorrect because a request for a deviation to operate in Class B airspace in an airplane not equipped with a transponder must be submitted to the controlling ATC facility at least 1 hr. before the proposed flight, not to the nearest FSDO (formerly called GADO) 24 hr. before the proposed operation.
    • Answer (C) is correct. ATC may authorize deviations on a continuing basis, or for individual flights, for operations of aircraft without a transponder. The request for a deviation must be submitted to the ATC facility having jurisdiction over the airspace concerned at least 1 hr. before the proposed operation.

    [309]

    When an aircraft is not equipped with a transponder, what requirement must be met before ATC will authorize a flight within Class B airspace?
    1. The proposed flight must be conducted in visual meteorological conditions (VMC).
    2. The proposed flight must be conducted when operating under instrument flight rules.
    3. A request for the proposed flight must be made to ATC at least 1 hour before the flight.
    • Answer (A) is incorrect because you must request a deviation from the transponder equipment requirement from ATC. The requirement is not that the proposed flight be conducted in VMC.
    • Answer (B) is incorrect because you must request a deviation from the transponder equipment requirement from ATC. The requirement is not that the proposed flight be conducted under IFR.
    • Answer (C) is correct. Requests for ATC authorized deviations must be made to the ATC facility having jurisdiction over the concerned airspace. A request for a deviation from the transponder equipment requirement in Class B airspace must be made to the controlling ATC facility at least 1 hr. before the flight.

    [310]

    If the aircraft's transponder fails during flight within Class B airspace,
    1. aircraft must immediately descend below 1,200 feet AGL and proceed to destination.
    2. the pilot should immediately request clearance to depart the Class B airspace.
    3. ATC may authorize deviation from the transponder requirement to allow aircraft to continue to the airport of ultimate destination.
    • Answer (A) is incorrect because a pilot may descend only if clearance from ATC is obtained.
    • Answer (B) is incorrect because ATC can immediately authorize a deviation from the transponder requirement without requiring the pilot to request clearance to depart the Class B airspace area.
    • Answer (C) is correct. If an aircraft's transponder fails during flight within Class B airspace, ATC may authorize deviation from the transponder requirement to allow the aircraft to continue to the airport of ultimate destination, including any intermediate stops, or to proceed to a place where suitable repairs can be made, or both.

    [311]

    Your aircraft had the static pressure system and altimeter tested and inspected on January 5, of this year, and was found to comply with FAA standards. These systems must be reinspected and approved for use in controlled airspace under IFR by
    1. January 5, 2 years hence.
    2. January 5, next year.
    3. January 31, 2 years hence.
    • Answer (A) is incorrect because these tests must be completed within the preceding 24 calendar months, not 2 years from the date of the last inspection.
    • Answer (B) is incorrect because these tests must be completed every 24 calendar months, not 1 year from the date of the last inspection.
    • Answer (C) is correct. Within the preceding 24 calendar months, each static pressure system, each altimeter instrument, and each automatic pressure altitude reporting system must be tested, inspected, and found to comply with the regulations. The 24-calendar-month period following January of this year begins February 1, this year, and ends on January 31, 2 years hence.

    [312]

    An aircraft altimeter system test and inspection must be accomplished within
    1. 18 calendar months.
    2. 24 calendar months.
    3. 12 calendar months.
    • Answer (A) is incorrect because the aircraft's altimeter system must be tested and inspected within 24, not 18, calendar months.
    • Answer (B) is correct. Within the preceding 24 calendar months, each static pressure system, each altimeter instrument, and each automatic pressure altitude reporting system must be tested, inspected, and found to comply with the regulations.
    • Answer (C) is incorrect because an annual inspection, not the altimeter system, must be accomplished within the preceding 12 calendar months.

    [313]

    Which publication covers the procedures required for aircraft accident and incident reporting responsibilities for pilots?
    1. FAR Part 61.
    2. NTSB Part 830.
    3. FAR Part 91.
    • Answer (A) is incorrect because FAR Part 61 concerns certification of pilots, flight instructors, and ground instructors.
    • Answer (B) is correct. NTSB Part 830 contains rules pertaining to the following:

    1. Notification and reporting aircraft accidents and incidents and certain other occurrences in the operation of aircraft when they involve civil aircraft of the U.S. wherever they occur, or foreign civil aircraft when such events occur in the U.S., its territories, or possessions.

    2. Reporting aircraft accidents and listed incidents in the operation of aircraft when they involve certain public aircraft.

    3. Preservation of aircraft wreckage, mail, cargo, and records involving all civil aircraft in the U.S., its territories, or possessions.

  • Answer (C) is incorrect because FAR Part 91 concerns general operating and flight rules.